SlideShare a Scribd company logo
1 of 61
ĐẠI HỌC QUỐC GIA HÀ NỘI
TRƯỜNG ĐẠI HỌC KHOA HỌC TỰ NHIÊN
Đào Thị Anh Phương
PHÉP BIẾN ĐỔI PHÂN TUYẾN TÍNH
VÀ ÁP DỤNG GIẢI MỘT SỐ BÀI TOÁN PHỔ THÔNG
LUẬN VĂN THẠC SỸ KHOA HỌC
Hà Nội - 2011
ĐẠI HỌC QUỐC GIA HÀ NỘI
TRƯỜNG ĐẠI HỌC KHOA HỌC TỰ NHIÊN
Đào Thị Anh Phương
PHÉP BIẾN ĐỔI PHÂN TUYẾN TÍNH
VÀ ÁP DỤNG GIẢI MỘT SỐ BÀI TOÁN PHỔ THÔNG
Chuyên ngành : Phương pháp toán sơ cấp
Mã số : 60 46 40
LUẬN VĂN THẠC SỸ KHOA HỌC
Người hướng dẫn khoa học
PGS.TS. NGUYỄN MINH TUẤN
Hà Nội - 2011
Mục lục
LỜI NÓI ĐẦU . . . . . . . . . . . . . . . . . . . . . . . . . . . . . . . . . . . . . . . . . . . . 2
Chương 1. Một lớp phương trình hàm sinh bởi hàm phân tuyến tính.. . . 4
1.1. Kiến thức chuẩn bị . . . . . . . . . . . . . . . . . . . . . . . . . . . . . . . . . . . . . . . . . . . . . 4
1.1.1. Hàm số. . . . . . . . . . . . . . . . . . . . . . . . . . . . . . . . . . . . . . . . . . . . . . . . . . . . . . . . . . . 4
1.1.2. Hàm số đơn điệu . . . . . . . . . . . . . . . . . . . . . . . . . . . . . . . . . . . . . . . . . . . . . . . . . . 5
1.1.3. Hàm phân tuyến tính . . . . . . . . . . . . . . . . . . . . . . . . . . . . . . . . . . . . . . . . . . . . . . 6
1.2. Phép biến đổi phân tuyến tính trong phương trình hàm . . . . . . . . . . 6
1.2.1. Hàm số xác định bởi các phép biến đổi phân tuyến tính. . . . . . . . . . . . . . . . 6
1.2.2. Một số bài toán khác về hàm phân tuyến tính. . . . . . . . . . . . . . . . . . . . . . . . 23
1.2.3. Bài tập tham khảo . . . . . . . . . . . . . . . . . . . . . . . . . . . . . . . . . . . . . . . . . . . . . . . . 34
Chương 2. Một số bài toán về dãy số. . . . . . . . . . . . . . . . . . . . . . . . . . . . . . . . . 36
2.1. Phương trình và hệ phương trình sai phân. . . . . . . . . . . . . . . . . . . . . . 36
2.1.1. Phương trình sai phân tuyến tính với hệ số hằng.. . . . . . . . . . . . . . . . . . . . . 36
2.1.2. Hệ phương trình sai phân tuyến tính với hệ số hằng . . . . . . . . . . . . . . . . . 40
2.2. Phương trình sai phân dạng phân tuyến tính với hệ số hằng . . . . . 41
2.3. Giới hạn của một số dãy truy hồi dạng phân tuyến tính. . . . . . . . . . 50
2.4. Bài tập tham khảo. . . . . . . . . . . . . . . . . . . . . . . . . . . . . . . . . . . . . . . . . . . . . 56
KẾT LUẬN . . . . . . . . . . . . . . . . . . . . . . . . . . . . . . . . . . . . . . . . . . . . . 58
Tài liệu tham khảo . . . . . . . . . . . . . . . . . . . . . . . . . . . . . . . . . . . . . . . 59
1
LỜI NÓI ĐẦU
Phép biến đổi phân tuyến tính có nhiều ứng dụng trong môn Toán ở bậc phổ
thông. Đặc biệt là ở trường chuyên, lớp chọn và trong các kỳ thi học sinh giỏi Toán
trong nước, trong các kỳ thi Olympic các nước trên thế giới thông qua các bài toán
về phương trình hàm, các bài toán về dãy số.
Để đáp ứng nhu cầu học tập và giảng dạy môn toán ở bậc phổ thông, luận văn
Phép biến đổi phân tuyến tính và áp dụng giải một số bài toán phổ thông với
mục tiêu tổng hợp và chọn lọc các kiến thức về phép biến đổi phân tuyến tính để
giải quyết các bài toán về phương trình hàm và các bài toán về dãy số. Luận văn
được chia thành hai chương.
Chương 1: Một lớp phương trình hàm sinh bởi hàm phân tuyến tính.
Chương này nêu lên một số kiến thức cơ bản về hàm số nói chung và hàm
phân tuyến tính nói riêng. Phần trọng tâm của chương là giải quyết các bài
toán về phép biến đổi phân tuyến tính trong phương trình hàm.
Chương 2: Một số bài toán về dãy số.
Chương này nêu lên các kiến thức cơ bản về phương trình và hệ phương trình
sai phân tuyến tính với hệ số hằng. Phần trọng tâm của chương là giải quyết
các bài toán về hai mảng kiến thức của dãy số:
• Phương trình sai phân dạng phân tuyến tính với hệ số hằng.
• Giới hạn của một số dãy sai phân dạng phân tuyến tính.
Để hoàn thành được luận văn này, trước nhất tác giả xin được gửi lời cảm ơn
sâu sắc tới PGS.TS Nguyễn Minh Tuấn đã dành thời gian hướng dẫn, đánh giá, chỉ
bảo tận tình giúp đỡ trong quá trình xây dựng đề tài cũng như hoàn thiện luận văn.
Tiếp theo, tác giả cũng xin gửi lời cảm ơn chân thành các thầy cô đã đọc, kiểm tra
đánh giá và cho những ý kiến quý báu để luận văn được đầy đủ hơn, phong phú
hơn. Qua đây, tác giả cũng xin được gửi lời cảm ơn tới Ban giám hiệu, phòng sau
Đại học, khoa Toán-Cơ-Tin học trường Đại học Khoa học Tự nhiên và các bạn đồng
2
nghiệp đã tạo điều kiện thuận lợi trong suốt quá trình học tập tại trường.
Tuy có nhiều cố gắng nhưng do thời gian và khả năng có hạn nên các vấn đề
trong luận văn chưa được trình bày sâu sắc và không thể tránh khỏi có những sai sót
trong cách trình bày. Rất mong được sự đóng góp ý kiến thêm nữa của thầy cô và
các bạn.
Tác giả xin chân thành cảm ơn !
Hà Nội, Tháng 02 năm 2011
Đào Thị Anh Phương
3
Chương 1
Một lớp phương trình hàm
sinh bởi hàm phân tuyến tính.
1.1. Kiến thức chuẩn bị
1.1.1. Hàm số
Định nghĩa Cho tập hợp D ⊂ R. Một ánh xạ f : D → R được gọi là một hàm
số từ tập D đến tập R và ký hiệu là f : D → R hoặc y = f(x).
•D được gọi là tập xác định của hàm số.
• f(x0) là giá trị của hàm số tại điểm x0 ∈ D.
• Tập hợp T = { f(x)|x ∈ D} được gọi là tập giá trị của hàm số f.
Chú ý
1)t ∈ T khi và chỉ khi phương trình f(x) = t có nghiệm x ∈ D.
2)t ∈ T, suy ra t có thể viết dưới dạng t = f(x) với x ∈ D.
• Điểm x0 ∈ Dđược gọi là điểm bất động của hàm f nếu như f(x0) = x0.
Ví dụ:
Ánh xạ x → f(x) =
ax+b
cx+d
c = 0 và ad − bc = 0 xác định một hàm ( gọi là
phân tuyến tính trên tập D = R{
−d
c
}).
4
1.1.2. Hàm số đơn điệu
Định nghĩa
1) Hàm số f(x) được gọi là tăng trên khoảng (a;b) nếu với x1 ,x2 ∈ (a;b) mà
x1 ≤ x2 thì f(x1) ≤ f(x2).
2) Hàm số f(x) được gọi là giảm trên khoảng (a;b) nếu như với
x1 ,x2 ∈ (a;b) mà x1 ≤ x2 thì f(x1) ≥ f(x2).
Hàm số tăng hoặc giảm trên một khoảng gọi là hàm số đơn điệu trên khoảng
đó.
3) Hàm số f(x) được gọi là tăng thực sự trên khoảng (a;b) nếu như với
x1 ,x2 ∈ (a;b) mà x1 < x2 thì f(x1) < f(x2).
4) Hàm số f(x) được gọi là giảm thực sự trên khoảng (a;b) nếu như với
x1 ,x2 ∈ (a;b) mà x1 < x2 thì f(x1) > f(x2).
5) Hàm số tăng thực sự hoặc giảm thực sự trên khoảng (a;b) gọi là hàm số đơn
điệu thực sự trên khoảng đó.
Tính chất:
1) Mọi hàm đơn điệu thật sự trên một khoảng đều là đơn ánh trên khoảng đó.
2) Nếu f : D → R; g : D → R là hai hàm tăng thì f +g tăng.
3) Nếu f : D → R; g : D → R là hai hàm tăng và không âm thì f(x) · g(x) là
hàm tăng.
4) Nếu hàm f đơn điệu trên khoảng (a,b) thì phương trình f(x) = m có nhiều
nhất một nghiệm trên khoảng đó.
5) Nếu f : Df → R và g : Dg → R tăng và Tf ⊂ Dg thì hàm số hợp g◦ f tăng.
Chú ý: Từ kết quả trên suy ra:
Nếu hàm f tăng thì hàm số hợp f(f(x)) (nếu được xác định) cũng tăng.
Nếu hàm f giảm thì hàm số hợp f(f(x)) (nếu được xác định) cũng giảm.
5
1.1.3. Hàm phân tuyến tính
Định nghĩa: Hàm phân tuyến tính là ánh xạ x −→ y = f(x) =
ax+b
cx+d
(1)
trong đó a,b,c,d ∈ R và ad −bc = 0, c = 0.
Điều kiện ad − bc = 0 để loại trường hợp của vế phải của (1) suy biến thành
hằng số vì
f (x) =
a(cx+d)−c(ax+b)
(cx+d)2
=
ad −bc
(cx+d)2
= 0, ∀x ∈ D.
Điều kiện c = 0 để loại trường hợp của vế phải của (1) suy biến thành (mx+n).
Tập xác định: D = R{
−d
c
}.
Tập giá trị: T = R{
a
c
}.
Một vài tích chất của hàm phân tuyến tính
Định lý 3.1:
(i) Hàm ngược của một hàm phân tuyến tính là hàm phân tuyến tính.
(ii) Hợp thành của hai hàm phân tuyến tính là hàm phân tuyến tính.
Định lý 3.2:
Hàm phân tuyến tính luôn tăng thực sự ( hoặc giảm thực sự ) trên mỗi khoảng
xác định của nó.
1.2. Phép biến đổi phân tuyến tính trong phương
trình hàm
1.2.1. Hàm số xác định bởi các phép biến đổi phân tuyến tính
Cho hàm số
w(x) =
ax+b
cx+d
, c = 0 ,ad −bc = 0.
Trong chương trình này, ta sẽ nghiên cứu các phương trình hàm dạng:
f(w(x)) = pf(x)+q, ∀x ∈ R
−d
c
trong đó a, b, c, d, p, q là các hằng số thực
và p = 0
6
Bài toán 1.2.1. Tìm f : R
−d
c
→ R sao cho
f
ax+b
cx+d
= p f(x) + q, ∀x ∈ R 
−d
c
, ad − bc = 0 ,c = 0 trong đó phương
trình w(x) = x có nghiệm.
Giải. a) Trường hợp w(x) = x có hai nghiệm phân biệt x1 ,x2.
Với x = x2 ta có f(w(x2)) = p f(x2)+q ⇒ f(x2) = p f(x2)+q
=⇒ f(x2)(1− p) = q.
+)Nếu p = 1thì f(x2) =
q
1− p
,
+)Nếu p = 1thì


Phương trình vô nghiệm nếu q = 0.
Phương trình có nghiệm f(x2) bất kỳ nếu q = 0.
Với x = x2, đặt
x−x1
x−x2
= t ⇒ x = x2 +
x2 −x1
t −1
, ∀t = 1,
và
ax+b
cx+d
= x2 +
x2 −x1
cx2 +d
cx1 +d
t −1
,∀t = 1.
Khi đó theo giả thiết thì
f x2 +
x2 −x1
cx2 +d
cx1 +d
t −1
= p f x2 +
x2 −x1
t −1
+q ∀t = 1.
Đặt
f x2 +
x2 −x1
t −1
= g(t).
Suy ra
g
cx2 +d
cx1 +d
t = pg(t)+q ,∀t = 1.
b) Trường hợp w(x) = x có nghiệm kép ( lúc đó (d −a)2 +4ac = 0)
và x0 =
a−d
2c
.
Với x = x0 ta có f(x0) = pf(x0)+q hay f(x0)(1− p) = q.
+) Nếu p = 1 thì f(x) =
q
1−q
.
7
+) Nếu p = 1 thì


Phương trình vô nghiệm nếu q = 0
Có nghiệmf(x0) tùy ý nếu q = 0
Với x = x0 đặt:
1
x−x0
= t ⇒ x = x0 +
1
t
, ∀t = 0,
ax+b
cx+d
= x0 +
1
t +
c
cx0 +d
.
Khi đó giả thiết đã cho trở thành
f


x0 +
1
t +
c
cx0 +d


 = p f x0 +
1
t
+q với t = 0.
Đặt f(x0 +
1
t
) = g(t) ta được
g t +
c
cx0 +d
= pg(t)+ q, ∀t = 0.
Bài toán 1.2.2. Tìm hàm
f : R
−d
c
→ R sao cho f
ax+b
cx+d
= pf(x) + q,
∀x ∈ R 
−d
c
, ad − bc = 0, c = 0, trong đó k ∈ Z+ sao cho wk(x) = x có
nghiệm, ở đó w1(x) = w(x) =
ax+b
cx+d
.
Giải. Gọi k ∈ Z+ là số bé nhất sao cho wk(x) = x có nghiệm
+) Nếu k = 1 thì chính là bài toán 1.2.1
+) Nếu k = 1 thì:
f(w(x)) = p· f(x) + q,
⇒ f(w2(x)) = p· f(w(x)) + q = p2 · f(x)+ pq+q,
···
⇒ f(wk(x)) = pk · f(x)+(pk−1 + pk−2 +···+ p+1)q.
Vì phương trình
wk(x) =
akx+bk
ckx+dk
= x,
8
(với ak ·dk −bk ·ck = 0) có nghiệm. Nếu ck = 0 hoặc dk = 0 thì bài toán sẽ quay
về bài toán 1.2.1 vừa xét.
Sau đây, ta minh họa cách giải ứng với các trường hợp thông qua bài toán cụ
thể.
Ta chỉ cần xét các phương trình hàm sinh bởi hàm phân tuyến tính
w(x) =
ax+b
cx+d
, c = 0, ad −bc > 0.
Bài toán 1.2.3. Tìm tất cả các hàm số f : R → R sao cho
f
−1
x+2
= 2 f(x)−3, ∀x = −2. (1.2.1)
Giải. Nhận xét rằng, phương trình x =
−1
x+2
có nghiệm duy nhất x = −1
Thay x = −1 vào (1.2.1), ta được f(−1) = 3.
Xét x = −1.
Đặt
1
x+1
= t thì t = 0 ,t = −1 và
x = −1+
1
t
,
−1
x+2
= −1+
1
t +1
.
Khi đó (1.2.1) có dạng
f −1+
1
t +1
= 2 f −1+
1
t
−3, ∀t ∈ R{−1;0}, (1.2.2)
hay g(t +1) = 2g(t), ∀t ∈ R{−1;0},
trong đó
g(t) = f(−1+
−1
t
)−3, ∀t ∈ R{−1;0}. (1.2.3)
Suy ra g(t) = 2th(t), với h(t) là hàm tùy ý sao cho
h(t +1) = h(t), ∀t ∈ R{−1;0}.
9
Từ (1.2.2) và (1.2.3) ta có
Kết luận
f(x) =



3, khi x = −1
g
1
x+1
+3, khi x = −1,
(1.2.4)
trong đó g(t) = 2th(t), với h(t) là hàm tùy ý thỏa mãn
h(t +1) = h(t), ∀t ∈ R{−1;0}.
Bài toán 1.2.4.[1] Cho q ∈ R và cho hàm số
w(x) =
ax+b
x−1
, a = 1, b =
−1
4
(a+1)2
.
Tìm tất cả các hàm số f : R{1} → R sao cho
f(w(x)) = −f(x)+q, ∀x = 1. (1.2.5)
Giải. Theo giả thiết thì phương trình w(x) = x có nghiệm thực duy nhất
x =
a+1
2
.
Thay x =
a+1
2
vào (1.2.5) ta được:
f
a+1
2
=
q
2
.
Xét x = x0, x0 =
a+1
2
.
Đặt t =
1
x−x0
, khi đó t = 0, và
x = x0 +
1
t
,
w(x) = x0 +
1
t +
2
a−1
.
Khi đó có thể viết (1.2.5) dưới dạng
f x0 +
1
t +
2
a−1
= −f(x0 +
1
t
)+q, ∀t ∈ R{0 ;
2
1−a
}, (1.2.6)
10
hay
g t +
2
a−1
= −g(t), ∀t ∈ R{0 ;
2
1−a
},
trong đó
g(t) = f(x0 +
1
t
)−
q
2
, ∀t ∈ R{0 ;
2
1−a
}. (1.2.7)
Suy ra
g(t) = h t +
2
a−1
−h(t) ,
với h(t) là hàm tùy ý thỏa mãn:
h t +
4
a−1
= h(t), ∀t ∈ R{0 ;
2
1−a
}.
Từ (1.2.6) và (1.2.7) ta có
Kết luận:
f(x) =



q
2
, khi x =
a+1
2
g



1
x−
a+1
2


+
q
2
, khi∀x ∈ R{1;
a+1
2
},
(1.2.8)
trong đó
g(t) = h t +
2
a−1
−h(t), ∀t ∈ R{0 ;
2
1−a
},
với h(t) là hàm tùy ý thỏa mãn
h t +
4
a−1
= h(t), ∀t ∈ R{0 ;
2
1−a
}.
Từ hai bài toán 1.2.3 và 1.2.4 ta xây dựng bài toán khái quát hóa trong trường
hợp phương trình hàm sinh bởi hàm phân tuyến tính
w(x) =
ax+b
cx+d
, c = 0, ad −bc = 0
thỏa mãn w (x) = x có nghiệm kép x = x0.
11
Bài toán 1.2.5.[1] Cho hàm số:
w(x) =
ax+b
cx+d
, ad −bc = 0, c = 0,
sao cho phương trình w(x) = x có nghiệm kép x = x0. Tìm tất cả các hàm số
f : R{
−d
c
} → R sao cho
f(w(x)) = −2f(x)+3, ∀x =
−d
c
. (1.2.9)
Giải. +) Theo giả thiết thì phương trình w(x) = x có nghiệm duy nhất x = x0.
Thay x = x0 vào (1.2.9), ta được f(x0) = 1.
+) Xét x = x0
Đặt
1
x−x0
= t thì t = 0 và x = x0 +
1
t
,
w(x) = x0 +
1
t +
1
x0 +
d
c
.
Khi đó có thể viết (1.2.9) dưới dạng
f







x0 +
1
t +
1
x0 +
d
c







= −2 f(x0 +
1
t
)+3, ∀t ∈ R{0}, (1.2.10)
hay g t +
1
x0 +
d
c
= −2g(t), ∀t ∈ R{0}, trong đó
g(t) = f(x0 +
1
t
)−1, ∀t ∈ R{0}. (1.2.11)
Suy ra
g(t) = 2
t
t0 h(t), trong đó hàm h(t) tùy ý thỏa mãn
h t +
1
x0 +
d
c
= −h(t), ∀t = 0.
12
Từ (1.2.10) và (1.2.11), ta có:
Kết luận
f(x) =



1 khi x = x0
g
1
x−x0
+1 khi x ∈ R{x0;
−d
c
},
(1.2.12)
trong đó
g(t) = 2
t
t0 h(t), ∀t = 0,
với h(t) là hàm tùy ý thỏa mãn:
h t +
1
x0 +
d
c
= −h(t), ∀t = 0.
Bài toán 1.2.6. Cho hàm số:
w(x) =
2
3−x
.
Tìm tất cả các hàm số f : R{3} → R sao cho
f(w(x)) = 2f(x)−3, ∀x = 3. (1.2.13)
Giải. Nhận xét rằng phương trình w(x) = x có hai nghiệm phân biệt x = 1 và
x = 2.
Thay x = 1 hoặc x = 2 vào (1.2.13) ta được f(1) = f(2) = 3
Xét x = 1 và x = 2
Đặt t =
x−1
x−2
thì t /∈ {2,1,0}.Do đó
x =
2t −1
t −1
= 2+
1
t −1
,
2
3−x
= 2+
1
t
2
−1
.
Viết (1.2.13) dưới dạng sau
f 2+
1
t
2
−1
= 2 f 2+
1
t −1
−3, ∀t /∈ {2,1,0},
hay
g
t
2
= 2g(t)−3
13
g(t) = f 2+
1
t −1
, ∀t /∈ {2,1,0}. (1.2.14)
Đặt
g(t) = 3+t−1
h(t), ∀t /∈ {2,1,0},
và viết (1.2.14) dưới dạng
3+
t
2
−1
h
t
2
= 2[3+t−1
h(t)]−3, ∀t /∈ {2,1,0}.
Vậy
h
t
2
= h(t), ∀t /∈ {2,1,0}. (1.2.15)
Từ (1.2.14) và (1.2.15), ta có
Kết luận
f(x) =



3, khi x ∈ {1;2}
g
x−1
x−2
, khi x /∈ {1;2;3},
(1.2.16)
trong đó
g(t) = 3+t−1
h(t),
với h(t) là hàm tùy ý thỏa mãn
h
t
2
= h(t), ∀t /∈ {2,1,0}.
Bài toán 1.2.7. Xác định hàm số f(x) thỏa mãn điều kiện sau:
f
2
x+2
= −3 f(x)+7, ∀x ∈ R{−2}. (1.2.17)
Giải. Nhận xét phương trình
2
x−2
= x có hai nghiệm phân biệt
x1 = −1−
√
3 và x2 = −1+
√
3.
+)Thay x = x1 hoặc x = x2 vào (2.1.17) ta được f(x1) = f(x2) =
7
4
.
+) Xét x = x1 và x = x2.
Đặt
x−x1
x−x2
= t thì t /∈ {
1
α
,0,1}, α =
1+
√
3
1−
√
3
.
Và
x = −1+
√
3+
2
√
3
t −1
,
14
2
x+2
= −1+
√
3+
2
√
3
αt −1
.
Khi đó ta có thể viết (1.2.17) dưới dạng sau:
f −1+
√
3+
2
√
3
αt −1
= −3 f −1+
√
3+
2
√
3
t −1
+7, ∀t /∈ {
1
α
,0,1}.
Hay
g(αt) = −3g(t)+7, ∀t /∈ {
1
α
,0,1} ,
trong đó
g(t) = f −1+
√
3+
2
√
3
t −1
, ∀t /∈ {
1
α
,0,1}. (1.2.18)
Vì α =
1+
√
3
1−
√
3
nên α = 0, |α| = 1.
Đặt
g(t) = |t|log|α|3
h(t)+
7
4
, ∀t /∈ {
1
α
,0,1}.
Khi đó ta có
|αt|log|α|3
h(αt)+
7
4
= −3|t|log|α|3
h(t)−
21
4
+7.
Do đó
h(αt) = −h(t), ∀t /∈ {
1
α
,0,1}. (1.2.19)
Từ (1.2.18) và (1.2.19) ta có
Kết luận
f(x) =



7
4
, khi x ∈ {−1−
√
3;−1+
√
3}
g
x+1+
√
3
x+1−
√
3
, khi x /∈ {−1−
√
3;−1+
√
3;−2},
(1.2.20)
trong đó 


t =
x+1+
√
3
x+1−
√
3
g(t) = |t|log|α|3
h(t)+
7
4
,
(1.2.21)
15
với h(t) là hàm tùy ý thỏa mãn
h(αt) = −h(t), ∀t /∈ {
1
α
,0,1}.
Từ hai bài 1.2.6 và 1.2.7 ta xây dựng bài toán khái quát hóa trong trường hợp
phương trình hàm sinh bởi phương trình phân tuyến tính
w(x) =
ax+b
cx+d
, c = 0, ad −bc = 0
thỏa mãn w(x) = x có hai nghiệm phân biệt x = x1, x = x2.
Bài toán 1.2.8.[1] Cho hàm số:
w(x) =
ax+b
cx+d
, ad −bc = 0, c = 0,
sao cho phương trình w(x) = x có hai nghiệm phân biệt x1, x2.
Tìm tất cả các hàm số f : R{
−d
c
} → R sao cho
f(w(x)) = 2 f(x)−3, ∀x =
−d
c
. (1.2.22)
Giải. Theo giả thiết thì phương trình w(x) = x có hai nghiệm phân biệt x =
x1, x = x2.
+)Thay x = x1 hoặc x = x2 vào (1.2.22),ta được:
f(x1) = f(x2) = 3.
+)Xét x = x1, x = x2,
Đặt
x−x1
x−x2
= t thì t /∈ {
1
α
,0,1}, α =
cx2 +d
cx1 +d
,
và x = x2 +
x2 −x1
t −1
,
ax+b
cx+d
= x2 +
x2 −x1
αt −1
.
Khi đó ta có thể viết (1.2.22) dưới dạng sau:
f x2 +
x2 −x1
αt −1
= 2 f x2 +
x2 −x1
t −1
−3, ∀t /∈ {
1
α
,0,1},
hay
g(αt) = 2g(t)−3, ∀t /∈ {
1
α
,0,1},
16
trong đó
g(t) = f x2 +
x2 −x1
t −1
, ∀t /∈ {
1
α
,0,1}. (1.2.23)
Do x1 = x2 và c = 0 nên α = 1.
Nếu α = −1 thì (1.2.23) cho ta f(x) = g(t) ≡ 3.
Do x2 =
−d
c
nên α = 0.
•) Xét trường hợp α = 0 và |α| = 1.
Đặt g(t) = 3+|t|log|α|2
h(t), ∀t /∈ {
1
α
,0,1},
khi đó ta có
−3+(|α||t|)log|α|2
·h(αt) = 2[3+|t|log|α|2
·h(t)]−3.
Do đó
h(αt) = h(t), ∀t /∈ {
1
α
,0,1}. (1.2.24)
•) Xét −1 = α < 0.
Đặt
g(t) = 3+|t|log|α|2
·h(t), ∀t /∈ {
1
α
,0,1},
khi đó ta có
3+(|α||t|)log|α|2
·h(αt) = 2[3+|t|log|α|2
·h(t)]−3.
Do đó
h(αt) = h(t), ∀t /∈ {
1
α
,0,1}. (1.2.25)
Từ (1.2.23), (1.2.24) và (1.2.25), ta có
Kết luận:
Nếu α = −1 thì f(x) = 3.
Nếu α = 0 và |α| = 1 thì
f(x) =



3, khix = x1, x = x2,
g
x−x1
x−x2
, khi x /∈ {x1,x2,
−d
c
},
(1.2.26)
17
trong đó
g(t) = 3+|t|log|α|2
·h(t),
với h(t) là hàm tùy ý thỏa mãn
h(αt) = h(t), ∀t /∈ {
1
α
,0,1}.
Tiếp theo, ta minh họa một số bài toán cụ thể trong trường hợp phương trình
hàm sinh bởi hàm phân tuyến tính
w(x) =
ax+b
cx+d
, c = 0, ad −bc = 0
thỏa mãn w(x) = x không có nghiệm thực .
Bài toán 1.2.9. Cho hàm số w(x) =
x−3
x−1
.
Tìm tất cả các hàm số f : R{1} → R sao cho
f(w(x))+ f(x) = 5, ∀x = 1. (1.2.27)
Giải. Nhận xét rằng phương trình w(x) = x không có nghiệm thực và
w(w(x)) ≡ x.
Ta chứng minh mọi hàm dạng
f(x) =
1
2
[g(w(x))−g(x)]+
5
2
, (1.2.28)
với g(x) tùy ý xác định trên R{1}, đều là nghiệm của (1.2.27).
Thật vậy, nếu f(x) có dạng (1.2.27) thì
f(w(x))+ f(x) =
1
2
[g(x)−g(w(x))]+
5
2
+
1
2
[g(w(x))−g(x)]+
5
2
= 5, ∀x = 1.
Ngược lại, khi f(x) thỏa mãn điều kiện (1.2.27) thì chỉ cần chọn g(x) = f(w(x))
ta có ngay biểu diễn (1.2.28).
Kết luận:
f(x) =
1
2
[g(w(x))−g(x)]+
5
2
,
với g(x) là hàm tùy ý xác định trên R{1}.
Bài toán 1.2.10.[1] Cho các hàm số h(x), x ∈ R và w(x) =
2x−5
x−2
.
Tìm tất cả các hàm số f : R{2} → R sao cho
f(w(x)) = f(x)+h(x), ∀x = 2. (1.2.29)
18
Giải. Nhận xét rằng phương trình w(x) = x không có nghiệm thực và
w(w(x)) ≡ x.
Thay x bởi w(x), từ (1.2.29) ta được
h(w(x)) = −h(x), (1.2.30)
Vậy điều kiện cần để (1.2.29) có nghiệm là điều kiện (1.2.30) được thỏa mãn
Giả sử điều kiện (1.2.30) được thỏa mãn. Khi đó
h(x) =
1
2
[h(x)−h(w(x))], ∀x = 2.
Ta chứng minh rằng mọi hàm dạng
f(x) =
1
2
[g(w(x))+g(x)−h(x)], (1.2.31)
với g(x) tùy ý xác định trên R{2}, là nghiệm của (1.2.29).
Thật vậy, nếu f(x) có dạng (1.2.31) thì
f(w(x)) =
1
2
[g(w(x))+g(x)−h(w(x))]
=
1
2
[g(w(x))+g(x)+h(x)]
=
1
2
[g(w(x))+g(x)−h(x)+2h(x)] = f(x)+h(x).
Ngược lại, khi f(x) thỏa mãn điều kiện (1.2.29) thì chỉ cần chọn g(x) = f(x), ta
có ngay biểu diễn (1.2.31).
Kết luận
f(x) =
1
2
[g(w(x))+g(x)−h(x)],
với g(x) là hàm tùy ý xác định trên R{2}.
Bài toán1.2.11.[1] Cho hàm số w(x) =
−1
x+1
.
Tìm tất cả các hàm số f : R{−1;0} → R sao cho
f(w(w(x)))+ f(w(x))+ f(x) = 3, ∀x = −1; x = 0. (1.2.32)
Giải. Nhận xét rằng phương trình w(x) = x không có nghiệm thực
Ta có đẳng thức sau đây:
w2(x) := w(w(x)) =
−1
−1
x+1
+1
= −
x+1
x
,
19
w3(x) := w(w(w(x))) =
−1
−
x+1
x
+1
= x, ∀x ∈ R{−1;0}.
Từ (1.2.32) ta thấy f(x) ≡ 1 là một nghiệm của bài toán.
Đặt f(x) = 1+g(x). Khi đó có thể viết (1.2.32) dưới dạng
g(w2(x))+g(w(x))+g(x) = 0, ∀x = −1; x = 0. (1.2.33)
Ta chứng minh rằng mọi nghiệm của (1.2.33) đều có dạng:
g(x) =
1
3
[2h(x)−h(w2(x))−h(w(x))], (1.2.34)
với h(x) là hàm tùy ý xác định trên R{−1;0}.
Thật vậy, khi g(x) có dạng (1.2.34) thì
g(w2(x))+g(w(x))+g(x)
=
1
3
[2h(w2(x))−h(w(x))−h(x)]+
1
3
[2h(w(x))−h(x)−h(w2(x))]+
+
1
3
[2h(x)−h(w2(x))−h(w(x))] = 0 ,∀x ∈ R{−1;0}.
Ngược lại, khi g(x) thỏa mãn (1.2.33) thì chỉ cần chọn h(x) = g(x) ta có ngay
công thức biểu diễn (1.2.34).
Kết luận:


f(x) ≡ 1, ∀x ∈ R{−1;0},
f(x) = 1+
1
3
[2h(x)−h(w2(x))−h(w(x))],
với h(x) là hàm tùy ý xác định trên R{−1;0}.
Bài toán 1.2.12.[1] Cho hàm số q(x) xác định trên R và w(x) =
−1
x+1
.
Tìm tất cả các hàm số f : R{−1;0} → R sao cho
f(w(w(x)))+ f(w(x))+ f(x) = q(x), ∀x ∈ R{−1;0}. (1.2.35)
Giải. Nhận xét rằng phương trình w(x) = x không có nghiệm thực và w(x) có
những tính chất
w2(x) := w(w(x)) =
−1
−1
x+1
+1
= −
x+1
x
,
w3(x) := w(w(w(x))) = −
−1
x+1
+1
−1
x+1
= x, ∀x ∈ R{−1;0}.
20
Từ tính chất này của hàm w(x), suy ra điều kiện cần để phương trình (1.2.35) có
nghiệm là
q(w(x)) = q(x), ∀x ∈ R{−1;0}. (1.2.36)
Giả sử điều kiện (1.2.36) thỏa mãn. Khi đó có thể viết:
q(x) =
1
3
[q(w2(x))+q(w(x))+q(x)], ∀x ∈ R{−1;0}. (1.2.37)
Từ (1.2.37) ta thấy f(x) ≡
1
3
q(x) là một nghiệm.
Đặt
f(x) =
1
3
q(x)+g(x).
Khi đó có thể viết (1.2.35) dưới dạng:
g(w2(x))+g(w(x))+g(x) = 0, ∀x ∈ R{−1;0}. (1.2.38)
Ta chứng minh rằng mọi nghiệm của (1.2.38) đều có dạng:
g(x) =
1
3
[2h(x)−h(w2(x))−h(w(x))], (1.2.39)
với h(x) là hàm tùy ý xác định trên R{−1;0}.
Thật vậy, khi g(x) có dạng (1.2.39) thì
g(w2(x))+g(w(x))+g(x)
=
1
3
[2h(w2(x))−h(w(x))−h(x)]+
+
1
3
[2h(w(x))−h(x)−h(w2(x))]+
1
3
[2h(x)−h(w2(x))−h(w(x))] = 0.
Ngược lại, khi g(x) thỏa mãn điều kiện (1.2.38) thì ta chỉ việc chọn h(x) = g(x)
ta có ngay biểu diễn (1.2.39).
Kết luận:
Điều kiện cần để phương trình (1.2.35) có nghiệm là:
q(w(x)) = q(x), ∀x ∈ R{−1;0}.
Khi đó mọi nghiệm của (1.2.35) có dạng:
f(x) =
1
3
q(x)+
1
3
[2h(x)−h(w2(x))−h(w(x))],
21
với h(x) là hàm tùy ý xác định trên R{−1;0}.
Bài toán 1.2.13.[1] Cho hàm số p(x) và q(x) xác định trên R và w(x) =
−1
x
.
Tìm tất cả các hàm số f : R{0} → R sao cho
p(x)f(w(x))+ f(x) = q(x), ∀x = 0. (1.2.40)
Giải. Nhận xét rằng, phương trình w(x) = x không có nghiệm thực và w(x) có
tính chất
w2(x) := w(w(x)) = x, ∀x = 0.
Thay x bởi w(x) vào (1.2.40), ta được
p(w(x))f(x)+ f(w(x)) = q(w(x)), ∀x = 0. (1.2.41)
Nhận thấy rằng (1.2.40) và (1.2.41) là hệ hai phương trình tuyến tính đối với hai
ẩn là f(w(x)) và f(x).
Nếu [1− p(w(x))p(x)] = 0, ∀x = 0 thì:
f(x) =
q(x)−q(w(x))p(x)
1− p(x)p(w(x))
, (1.2.42)
f(w(x)) =
q(w(x))− p(w(x))q(x)
1− p(x)p(w(x))
, ∀x = 0. (1.2.43)
Các công thức (1.2.42) và (1.2.43) xác định cùng một hàm số f(x) thỏa mãn
phương trình (1.2.40).
Nếu tồn tại x0 = 0 sao cho [1− p(w(x0))p(x0)] = 0 thì điều kiện cần để phương
trình (1.2.40) có nghiệm là:
q(x0)− p(x0)q(w(x0)) = 0,
và
q(w(x0))− p(w(x0))q(x0) = 0. (1.2.44)
Giả sử điều kiện (1.2.44) được thỏa mãn tại mọi điểm x0 = 0 sao cho
1− p(w(x0))p(x0) = 0.
22
Gọi Zpq là tập hợp các nghiệm x = 0 của phương trình (với ẩn là x )
1− p(w(x))p(x) = 0.
Nhận xét rằng nếu x0 = 0 thuộc Zpq thì w(x0) cũng thuộc Zpq.
Khi đó nghiệm của (1.2.40) được xác định theo cách sau.
a) Nếu x = 0 và x /∈ Zpq thì:
f(x) =
q(x)−q(w(x))p(x)
1− p(x)p(w(x))
.
b) Nếu x = 0 và x ∈ Zpq thì f(x) được chọn tùy ý sao cho (1.2.40) được thỏa
mãn.
Kết luận:
+) Nếu 1− p(x)p(w(x)) = 0, ∀x = 0 thì
f(x) =
q(x)−q(w(x))p(x)
1− p(x)p(w(x))
.
+) Nếu tồn tại x0 = 0 sao cho 1 − p(w(x0))p(x0) = 0 thì điều kiện để (1.2.40)
có nghiệm là q(x)− p(x)p(w(x)) = 0 và q(w(x))− p(w(x))p(x) = 0, ∀x ∈ Zpq với
Zpq là tập nghiệm khác 0 của phương trình 1− p(x)q(w(x)) = 0.
Khi đó:
f(x) =



q(x)−q(w(x))p(x)
1− p(x)p(w(x))
, ∀x ∈ Zpq
tùy ý , ∀x /∈ Zpq,x = 0.
(1.2.45)
1.2.2. Một số bài toán khác về hàm phân tuyến tính
Bài toán 1.2.14.[4] Tìm tất cả các hàm số f : R{0} → R thỏa mãn đồng thời
các điều kiện:
1)f(1) = 1;
2)f
1
x+y
= f
1
x
+f
1
y
;
3)(x+y)f(x+y) = yx f(x)f(y);
Với mọi x,y mà xy(x+y) = 0.
23
Giải. Trước hết chú ý rằng từ các giả thiết ta suy ra f(x) = 0 với mọi x = 0.
Thật vậy, giả sử có x0 = 0 mà f(x0) = 0. Khi đó từ giả thiết 1) ta có x0 = 1.
Thay x = 1−x0, y = x0 vào giả thiết 3) ta có:
(1−x0 +x0)f(1−x0 +x0) = (1−x0)x0 f(1−x0)f(x0) ⇒ f(1) = 0, vô lý .
Vậy với mọi x = 0 thì f(x) = 0.
Từ giả thiết 2) thay x = y ta có:
f
1
2x
= 2 f
1
x
, ∀x = 0. (1.2.46)
Từ (1.2.46) thay x bởi
1
2y
ta có:
f(y) = 2 f(2y) ⇒ f(x) = 2 f(2x) ∀x = 0. (1.2.47)
Mặt khác, từ giả thiết 3) thay x = y ta có:
2x f(2x) = x2
(f(x))2
, ∀x = 0. (1.2.48)
Như vậy (1.2.47) và (1.2.48) cho ta
x f(x) = x2
(f(x))2
, ∀x = 0.
hay f(x) = x(f(x))2 ⇒ f(x) =
1
x
với mọi x = 0 (thỏa mãn).
Kết luận:
f(x) =
1
x
, ∀x = 0.
Bài tập 1.2.15.[4] Tìm tất cả các hàm f : R → R thỏa mãn đồng thời các điều
kiện
1)f(−x) = −f(x), ∀x ∈ R.
2)f(x+1) = f(x)+1, ∀x ∈ R.
3)f
1
x
=
f(x)
x2
, ∀x = 0.
Giải. Từ 1) ta có:
f(0) = −f(0) ⇒ f(0) = 0.
Vì thế f(1) = f(0)+1 = 1.
Từ 2) thay x = −1 ta có:
f(0) = f(−1)+1 ⇒ f(−1) = −1.
24
Bây giờ ta xác định hàm số với x /∈ {−1;0}.
Ta có:
f
x+1
x
= f 1+
1
x
= f(1)+ f
1
x
= 1+ f
1
x
. (1.2.49)
Mặt khác
f
x+1
x
=
x+1
x
2
f
x
x+1
=
x+1
x
2
f 1−
1
x+1
=
x+1
x
2
1− f
1
x+1
=
x+1
x
2
1−
1
(x+1)2
f(x+1)
=
1
x2
(x+1)2 − f(x)−1 .
Do đó ta có:
f(x)+x2
= x2
+2x− f(x) ⇒ f(x) = x.
Thử lại ta thấy f(x) = x thỏa mãn các điều kiện 1), 2), 3).
Kết luận:
f(x) = x, ∀x ∈ R.
Bài tập 1.2.16.[4] Tìm tất cả các hàm f : R+ → R+ thỏa mãn
f(x+y)+ f(x)f(y) = f(xy)+ f(x)+ f(y), ∀x,y ∈ R+
. (1.2.50)
Giải. Cho x = y = 2 ta được:
f(4)+(f(2))2
= f(4)+2 f(2) ⇒ (f(2))2
−2 f(2) = 0 ⇒ f(2) = 2( do f(x) > 0).
Trong (1.2.50) lại cho x =y =1 có f(2)+[f(1)]2 = f(1)+2 f(1)
hay [f(1)]2 −3 f(1)+2 = 0 ⇒ f(1) = 1 hoặc f(1)=2.
a) Xét trường hợp f(1) = 2.
Trong (1.2.50) thay y = 1 ta được:
f(x+1)+2 f(x) = f(x)+ f(x)+ f(1) ⇒ f(x+1) = 2.
Từ đây suy ra f(x) = 2, ∀x > 1.
Với 0 < x < 1 ta chọn y =
1
x
> 1 thì ta có:
f(x+
1
x
)+ f(x)f
1
x
= f(1)+ f(x)+ f
1
x
,
25
⇒ 2+2 f(x) = 2+ f(x)+2 ⇒ f(x) = 2.
Như vậy ta có một hàm số cần tìm là f(x) = 2, ∀x ∈ R+.
b) Xét trường hợp f(1) = 1.
Thay y = 1 vào (1.2.50):
f(x+1)+ f(x) = f(x)+ f(x)+ f(1)
Như thế:
f(x+1) = f(x)+1, ∀x ∈ R+
. (1.2.51)
Từ (1.2.51) bằng phép quy nạp đơn giản ta có f(n) = n, ∀n ∈ N∗ và
f(x+n) = f(x)+n, ∀x ∈ R+
,∀n ∈ N∗
. (1.2.52)
Từ (1.2.50) thay x = n, y =
1
n
ta có:
f n+
1
n
+f(n)f
1
n
= f(1)+ f(n)+ f
1
n
.
Chú ý rằng từ (1.2.52) có:
f n+
1
n
= n+ f
1
n
.
Vậy:
n+ f
1
n
+nf
1
n
= 1+ f(n)+ f
1
n
⇒ f
1
n
=
1
f(n)
=
1
n
,∀n ∈ N∗
.
Với mọi số hữu tỷ
m
n
,∀m,n ∈ N∗, ta thay x = m,y =
1
n
thì được:
f m+
1
n
+f(m)f
1
n
= f
m
n
+f(m)+ f
1
n
⇒ m+
1
n
+
m
n
= f
m
n
+m+
1
n
,
suy ra f
m
n
=
m
n
.
Như thế đã chứng minh được rằng ∀x ∈ Q+ thì f(x) = x.
Ta chứng minh f là hàm đồng biến trên khoảng (0;+∞).
Trước hết chú ý rằng ∀x > 1 chọn
y =
x
x−1
> 0 ⇒ x+y = xy ⇒ f(x+y) = f(xy).
26
Khi đó (1.2.50) trở thành
f(x)f
x
x−1
= f(x)+ f
x
x−1
⇒ f
x
x−1
(f(x)−1) = f(x).
Từ đây suy ra
f(x)−1 > 0 ⇒ f(x) > 1 và f
x
x−1
=
f(x)
f(x)−1
,∀x > 1. (1.2.53)
Mặt khác từ (1.2.53) có:
1+ f
1
x
= f 1+
1
x
= f
x+1
x
=
f(x+1)
f(x+1)−1
=
f(x)+1
f(x)
.
Như vậy:
f
1
x
=
1
f(x)
, ∀x ∈ R+
,
do đó:
Nếu x > 1 thì f(x) > 1, và nếu 0 < x < 1 thì 0 < f(x) < 1.
• Xét 0 < x < y < 1.
Ta có:
f(y−x+x)+ f(y−x)f(x) = f((y−x)x)+ f(y−x)+ f(x).
Hay
f(y) = f((y−x)x)+ f(y−x)+ f(x)− f(y−x)f(x).
Nghĩa là:
f(y) = f(x)+ f((y−x)x)+ f(y−x)(1− f(x)).
Để ý rằng do 0 < x < 1 nên f(x) < 1, vì thế f(y) > f(x).
Do đó hàm f đồng biến trên khoảng (0;1).
• Xét 1 < x < y.
Suy ra:
0 <
1
y
<
1
x
< 1 ⇒ f(
1
y
) < f(
1
x
) ⇒
1
f(y)
<
1
f(x)
⇒ f(y) > f(x).
Như thế f đồng biến trên (1;+∞).
Vậy hàm f là hàm đồng biến trên khoảng (0;+∞).
Cuối cùng với mỗi x > 0 ta chọn hai dãy số hữu tỷ (un),(vn) sao cho:
un ≤ x ≤ vn và limun = limvn = x.
27
Khi đó do f đồng biến nên
f(un) ≤ f(x) ≤ f(vn) ⇒ un ≤ f(x) ≤ vn.
Cho n → +∞ ta được f(x) = x.
Thử lại thấy thỏa mãn.
Kết luận: Có hai hàm số thỏa mãn đầu bài là:
f(x) = 2 và f(x) = x, ∀x ∈ R+
.
Bài toán 1.2.17. (Komal-A328 ) [4] Tìm tất cả các hàm f : R+ → R+ thỏa mãn:
f(f(x)+y) = x f(1+xy), ∀x,y > 0. (1.2.54)
Giải. Trước hết ta chứng minhf là hàm số giảm.
Giả sử có 0 < u < v mà f(u) < f(v).
Chọn w =
v f(v)−uf(u)
v−u
.
Khi đó
uf(u) < uf(v) ⇒ v f(v)−u f(u) > vf(v)−uf(v) = (v−u)f(v).
Vậy:
w =
v f(v)−uf(u)
v−u
> f(v) > f(u).
Ta có:
f(w) = f(f(u)+(w− f(u))) = uf 1+
uv(f(v)− f(u))
v−u
.
Lại có:
f(w) = f(f(v)+(w− f(v))) = v f 1+
uv(f(v)− f(u))
v−u
.
Như thế u = v, vô lý.
Vậy ta phải có f(u) ≥ f(v) nghĩa là f hàm không tăng.
Thay x = 1 vào (1.2.54) có
f(f(1)+y) = f(1+y). (1.2.55)
28
Nếu f(1) > 1 thì từ (1.2.55) suy ra f(y) = f(y + f(1) − 1), ∀y > 1, còn nếu
f(1) < 1 thì lại có f(y) = f(y+1− f(1)), ∀y > 1. Như thế trong cả hai trường hợp
thì hàm f luôn tuần hoàn trên khoảng (1;+∞) với chu kỳ là |f(1)−1|. Như thế f
là hàm đơn điệu tuần hoàn nên phải là hàm hằng với x > 1.
Từ (1.2.54) ta thấy rằng vế trái là hằng số nhưng vế phải lại lớn tùy ý, vô lý. Do
đó f(1) = 1.
Xét x > 1.
Chọn
y =
x−1
x
⇒ xy+1 = x,
khi đó (1.2.54) trở thành
f f(x)+
x−1
x
= x f(x).
Nếu f(x) >
1
x
thì
f(x)+
x−1
x
= f(x)−
1
x
+1 > 1 ⇒ f f(x)−
1
x
+1 ≤ f(1) = 1.
Vậy x f(x) ≤ 1 ⇒ f(x) ≤
1
x
, muân thuẫn. Tương tự cũng không thể có f(x) <
1
x
.
Do đó với x > 1 thì f(x) =
1
x
.
Cuối cùng với x > 0 chọn y = 1 có f(f(x)+1) = x f(1+x).
Nhưng vì f(x)+1 > 1 và x+1 > 1 nên
f(f(x)+1) =
1
f(x)+1
và f(x+1) =
1
x+1
.
Do đó
1
f(x)+1
=
x
x+1
⇒ x f(x)+x = x+1 ⇒ f(x) =
1
x
.
Thử lại thấy f(x) =
1
x
thỏa mãn.
Kết luận: Hàm số cần tìm là:
f(x) =
1
x
,∀x ∈ R+
.
Bài toán1.2.18.[4] Tìm tất cả các hàm xác định trên tập các số thực dương và
nhận giá trị trong tập đó thỏa mãn
f(x f(y))f(y) = f(x+y), ∀x,y > 0. (1.2.56)
29
Giải. Giả sử có y>0 mà f(y) >1.
Khi đó chọn x =
y
f(y)−1
> 0 ⇒ x f(y) = x+y
Ta có f(x f(y)) = f(x+y) = f(x f(y))f(y).
Nhưng f(x f(y)) = f(x+y) > 0 nên ta suy ra f(y)=1, vô lý.
Vậy với mỗi y >0 thì 0 < f(y) ≤ 1. Từ đây suy ra kết quả sau
f(x+y) = f(x f(y))f(y) ≤ f(y), ∀x,y > 0
hay với 0 <x < y ta có:
f(y) = f(y−x+x) = f((y−x)f(x))f(x) ≤ f(x) ⇒ f là hàm số giảm
Nếu có a >0 và f(a)=1 thì
f(y f(a))f(a) = f(a+y) ⇒ f(a+y) = f(y), ∀y > 0
Bằng phương pháp quy nạp dễ có kết quả
f(na+x) = f(x),∀x > 0,∀n ∈ N
Ta chứng minh f là hàm số hằng.
Thật vậy:
Xét 0 < x < y
Đặt
n =
y−x
a
⇒ n ≤
y−x
a
< n+1 ⇒ na+x ≤ y < (n+1)a+x
Vì f giảm nên ta có f(na+x) ≥ f(y) ≥ f((n+1)a+x).
Mặt khác
f(na+x) = f((n+1)a+x) = f(x). Vậy f(x) ≤ f(y) ≤ f(x) ⇒ f(x) = f(y)
Thành thử f là hàm hằng mà f(a) =1, do đó f(x) = 1, ∀x > 0.
Cuối cùng ta xét trường hợp 0 < f(x) < 1, ∀x > 0.
Ta thấy là hàm đơn điệu, giảm thực sự, vì với 0 < x < y ta có:
f(y) = f(y−x+x) = f((y−x)f(x))f(x) < f(x)
Từ hệ thức (1.2.56) cho y=1 ( đặt f(1) = a < 1) ta được
f(ax)a = f(x+1) = f(ax+1+x(1−a)) = f(ax)f((1+x−ax)f(ax))
30
Suy ra
f((1+x−ax)f(ax)) = a = f(1) ⇒ 1 = (1+x−ax)f(ax)( do f giảm thực sự )
Vậy f(ax) =
1
1+x−ax
. Cuối cùng thay x bởi
x
a
ta được
f(x) =
1
1+
x
a
−a
x
a
=
a
a+(1−a)x
, ∀x > 0
Thử lại: Dễ thấy các hàm số f(x) = 1, ∀x > 0 và f(x) =
a
a+(1−a)x
, ∀x > 0
hoàn toàn thỏa mãn điều kiện của bài toán.
Kết luận
Các hàm số cần tìm là


f(x) = 1, ∀x > 0,
f(x) =
a
a+(1−a)x
, ∀x > 0, a tùy ý thuộc khoảng(0;1).
Bài toán 1.2.19. ( IMO -1986 ) [4] Hãy xác định tất cả các hàm f xác định trên
tập hợp các số thực không âm và nhận giá trị trong tập đó thỏa mãn điều kiện:
f(x f(y))f(y) = f(x+y), ∀x,y ≥ 0. (1.2.57)
Giải. Thay x = y = 0 vào (2.2.57), ta được
f(0 f(0))f(0) = f(0) ⇒ [f(0)]2
= f(0) ⇒ f(0)(f(0)−1) = 0 ⇒ f(0) = 0hoặc f(0) = 1.
Nếu f(0) = 0 thì với mọi x > 0:
f(x f(0))f(0) = f(x) ⇒ f(x) = 0, ∀x ≥ 0.
Ta xét trường hợp f(0) = 1. Có hai khả năng sau đây:
1) f(x) > 0,∀x > 0 theo bài toán 1.2.18, các hàm số cần tìm là:
f(x) = 1, ∀x ≥ 0 và f(x) =
a
a+(1−a)x
, ∀x ≥ 0.
2) Tồn tại a,b > 0 sao cho f(a) > 0 và f(b) = 0.
Ta có với 0 < x < a thì
f((a−x)f(x))f(x) = f(a−x+x) = f(a) > 0 ⇒ f(x) > 0.
31
Vì f(b) = 0 nên suy ra b > a.
Lại có với x > b thì
f(x) = f(x−b+b) = f((x−b)f(b))f(b) = 0.
Vậy với 0 ≤ x ≤ a thì f(x) > 0 và với x ≥ b thì f(x) = 0.
Do đó nếu f(x) > 0 thì x < b và nếu f(x) = 0 thì x > a.
Ta xét hai tập hợp:
A = {x ≥ 0|f(x) > 0}; B = {x ≥ 0|f(x) = 0}.
Rõ ràng A,B là hai tập khác rỗng vì a ∈ A và b ∈ B. Mặt khác A là tập bị chặn
trên bởi số b vì ∀x ∈ A thì f(x) > 0 suy ra x < b và B là tập bị chặn dưới bởi số a vì
∀x ∈ B thì f(x) = 0 suy ra x > a.
Thành thử tồn tại α = Sup A, β = inf B,
Ta sẽ chứng minh α = β.
Thật vậy, giả sử α < β suy ra có số ϕ sao cho α < ϕ < β.
Từ α < ϕ, ta thấy nếu f(ϕ) > 0 thì ϕ ∈ A ⇒ ϕ ≤ α trái với α < ϕ < β.
Do đó f(ϕ) = 0 ⇒ ϕ ∈ B ⇒ ϕ ≥ β cũng mâu thuẫn.
Nếu α > β thì có số ϕ sao cho α > ϕ > β, khi đó có a ∈ A và b ∈ B sao cho
a ≥ ϕ ≥ b. Điều này không đúng với mọi a ∈ A và với mọi b ∈ B ta luôn có a < b.
Vậy α = β.
Bây giờ ta tìm công thức của hàm f.
Xét 0 ≤ x < α.
Giả sử
f(x) <
α
α −x
⇒ α −x <
α
f(x)
.
Chọn y sao cho
α −x < y <
α
f(x)
⇒ x+y > α và yf(x) < α.
Vì yf(x) < α nên có a ∈ A sao cho y f(x) ≤ a ⇒ f(yf(x)) > 0. Do đó
f(x+y) = f(yf(x))f(x) > 0 ⇒ x+y ∈ A ⇒ x+y ≤ α, vô lý .
Còn nếu
f(x) >
α
α −x
⇒ α −x >
α
f(x)
,
32
thì ta chọn số y sao cho
α −x > y >
α
f(x)
⇒ x+y < α và yf(x) > α.
Do x+y < α và x < α nên có a ∈ A và a ∈ A mà x+y ≤ a và x ≤ a .
Khi đó ta có f(x+y) > 0 và f(x) > 0.
Vậy f(yf(x)) > 0 ⇒ y f(x) ∈ A ⇒ y f(x) ≤ α, vô lý.
Như vậy ta đã chứng minh được rằng với 0 ≤ x < α thì f(x) =
α
α −x
.
Xét x > α = β, khi đó có b ∈ B mà x ≥ b ⇒ f(x) = 0.
Cuối cùng ta tính giá trị của hàm số tại điểm α. Gọi x, y > 0 là hai số tùy ý thỏa
mãn
x+y = α ⇒ 0 < x < α và 0 < y < α ⇒ f(x) =
α
α −x
và f(y) =
α
α −y
Ta có
x f(y) =
αx
α −y
=
αx
x
= α,
do đó từ f(x f(y))f(y) = f(x+y) ⇒ f(α)f(y) = f(α).
Chú ý rằng f(y) =
α
α −y
= 1 nên f(α) = 0.
Tóm lại, trong trường hợp thứ hai ta có hàm số:
f(x) =



α
α −x
, khi 0 ≤ x < α
0, khi x ≥ α.
(1.2.58)
Thử lại thấy thỏa mãn điều kiện của bài toán.
Kết luận: Các hàm số cần tìm là
f(x) = 0, ∀x ≥ 0,
f(x) = 1, ∀x ≥ 0,
f(x) =
a
a+(1−a)x
, (0 < a < 1), ∀x ≥ 0.
Nếu tồn tại 0 < a < b mà f(a) > 0 và f(b) = 0 thì:
f(x) =



α
α −x
, khi 0 ≤ x < α
0 ,khi x ≥ α,
(1.2.59)
với α = Sup{x ∈ R+|f(x) > 0} = inf {x ∈ R+|f(x) = 0}.
33
1.2.3. Bài tập tham khảo
Bài tập 1.2.20. Cho a,b,c,d, p,q ∈ R, c = 0. Xác định các hàm f(x) sao cho
f
ax+b
cx+d
= p f(x)+q, ∀x ∈ R{
−d
c
}.
Bài tập 1.2.21. Tìm hàm f(x) thỏa mãn điều kiện
f(x)f(
1
x
) = 1, ∀x = 0.
Bài tập 1.2.22. Tìm hàm f(x) thỏa mãn điều kiện
f(x)f
1
1−x
= 1, ∀x = 0, x = 1.
Bài tập 1.2.23. Cho hàm số h(x), x ∈ R{0}.
Tìm hàm f(x) thỏa mãn điều kiện
f
1
x
= x f(x)+h(x), ∀x = 0.
Bài tập 1.2.24. Tìm hàm f(x) thỏa mãn điều kiện
f(x)f
1
1−x
= x−1, ∀x = 0, x = 1.
Bài tập 1.2.25. Tìm hàm f(x) thỏa mãn điều kiện
f(x)+ f
1
1−x
+f
x−1
x
= −1, ∀x = 0, x = 1.
Bài tập 1.2.26. Cho hàm số
w(x) =
−2
x+1
Ký hiệu
w2(x) = w(w(x)) , w3(x) = w(w(w(x)))
Tìm tất cả các hàm số f : R{−1;−3;1} → R sao cho



f(w3(x)) = f(x)
f(w2(x))+ f(w(x))+ f(x) = 3, ∀x /∈ {−1;−3;1}.
(1.2.60)
34
Bài tập 1.2.27. Cho hàm số q(x) xác định trên R và w(x) =
−2
x+1
.
Ký hiệu
w2(x) = w(w(x)) , w3(x) = w(w(w(x))).
Tìm tất cả các hàm số f : R{−1;−3;1} → R sao cho



f(w3(x)) = f(x)
f(w2(x))+ f(w(x))+ f(x) = q(x), ∀x /∈ {−1;−3;1}.
(1.2.61)
Bài tập 1.2.28. Tìm hàm f : R → R thỏa mãn điều kiện
f(x)+
1
2x
f
1
1−x
= 1, ∀x ∈ R{0;1}.
Bài tập 1.2.29. (Iberoamerican - 1987 ) Tìm hàm f(x) thỏa mãn điều kiện
(f(x))2
f
1−x
1+x
= 64x, ∀x ∈ R{0;±1}.
Bài tập 1.2.30. ( Australian- 1992) Tìm tất cả các hàm số f : R{
2
3
} → R thỏa
mãn điều kiện
2f(x)+ f
2x
3x−2
= 996x, ∀x ∈ R{
2
3
}.
Bài tập 1.2.31.( CAMO - 1991) Chứng minh tồn tại đúng một hàm số f :
R{0} → R sao cho
f(x) = x f
1
x
, và f(x+y) = f(x)+ f(y)−1, ∀x,y ∈ R{0} và x+y = 0.
Bài tập 1.2.32. Tìm tất cả các hàm số f xác định trên R{±1} thỏa mãn
f
x−3
x+1
+f
x+3
1−x
= x, ∀x ∈ R{±1}.
Bài tập 1.2.33. Tìm tất cả các hàm số f xác định trên tập hợp các số thực khác
0 sao cho:
1) f(−x) = −f(x), ∀x = 0;
2) f
1
x+y
= f
1
x
+f
1
y
+2(xy−1000), ∀x,y ∈ R{0} và x+y = 0.
Bài tập 1.2.34. Cho a,b,c > 0. Tìm tất cả các hàm f : R+ → R thỏa mãn:
f(x)+af
b
x
= cx.
35
Chương 2
Một số bài toán về dãy số.
2.1. Phương trình và hệ phương trình sai phân.
2.1.1. Phương trình sai phân tuyến tính với hệ số hằng.
• Phương trình sai phân tuyến tính cấp một là phương trình sai phân dạng
u1 = α,aun+1 +bun = fn,n ∈ N∗
.
trong đó a,b,α là các hằng số, a = 0 và fn là biểu thức của n cho trước.
Cách giải.
1) Nếu fn = 0 thì giải phương trình đặc trưng aλ + b = 0 để tìm λ. Khi đó
un = qλn (q là hằng số), trong đó q được xác định khi biết u1 = α.
2) Nếu fn là đa thức theo n thì giải phương trình đặc trưng aλ + b = 0 ta tìm
được λ. Ta có un = un + un, trong đó un là nghiệm của phương trình thuần nhất
aun+1 + bun = 0 và u∗
n là nghiệm riêng tùy ý của phương trình không thuần nhất
aun+1 +bun = fn. Vậy un = qλn, q là hằng số sẽ được xác định sau.
Ta xác định u∗
n như sau:
a) Nếu λ = 1 thì un là đa thức cùng bậc với fn.
b) Nếu λ = 1 thì u∗
n = n.gn với gn là đa thức cùng bậc với fn. Thay u∗
n vào
phương trình, đồng nhất các hệ số, ta tính được các hệ số của u∗
n.
3) Nếu fn = νµn,n ∈ N∗ thì giải phương trình đặc trưng aλ +b = 0 để tìm λ.
36
Ta có un = un + u∗
n, trong đó un = cλn,c là hằng số chưa được xác định, u∗
n được
xác định như sau:
a) Nếu λ = µ thì u∗
n = Aµn.
b) Nếu λ = µ thì u∗
n = Anµn.
Thay u∗
n vào phương trình, đồng nhất các hệ số tính được các hệ số của u∗
n. Biết u1,
từ hệ thức un = un +u∗
n, tính được c.
4) Nếu fn = f1n + f2n,n ∈ N∗, trong đó f1n là đa thức theo n và f2n = νµn
thì ta có un = un + un, trong đó un là nghiệm tổng quát của phương trình thuần
nhất aun+1 + bun = 0,u∗
n là một nghiệm riêng của phương trình không thuần nhất
aun+1 + bun = f1n,u∗∗
n là nghiệm riêng bất kỳ của phương trình không thuần nhất
aun+1 +bun = f2n.
• Phương trình sai phân tuyến tính cấp hai là phương trình sai phân dạng
u1 = α,u2 = β,aun+1 +bun +cun−1 = fn,n ∈ N∗
trong đó a,b,c,α,β là các hằng số, a = 0 và fn là biểu thức chứa n cho trước.
Cách giải.
1) Nếu fn = 0 thì giải phương trình đặc trưng aλ2 +bλ +c = 0, tìm λ.
a) Nếu λ1,λ2 là hai nghiệm thực khác nhau thì un = Aλn
1 +Bλn
2 , trong đó A và
B được xác định khi biết u1 và u2.
b) Nếu λ1,λ2 là nghiệm thực kép, λ1 = λ2 = λ thì un = (A+Bn)λn, trong đó A
và B được xác định khi biết u1 và u2.
c) Nếu λ là nghiệm phức, λ = x+yi, thì ta đặt
r =| λ |= x2 +y2,tanϕ =
y
x
,ϕ ∈
−π
2
,
π
2
Lúc đó λ = r(cosϕ + isinϕ) và un = rn(Acosnϕ + Bsinnϕ), trong đó A và B được
xác định khi biết u1 và u2.
2) Nếu fn là đa thức theo n cho trước thì giải phương trình đặc trưng aλ2 +
bλ + c = 0 để tìm λ. Ta có un = un + u∗
n, trong đó un là nghiệm tổng quát của
phương trình thuần nhất aun+1 +bun +cun−1 = 0 và u∗
n là một nghiệm riêng tùy ý
của phương trình không thuần nhất aun+1 +bun +cun−1 = fn. Theo 1) ta tìm được
un, trong đó các hệ số A,B chưa xác định, u∗
n được xác định như sau:
a) Nếu λ = 1 thì u∗
n là đa thức cùng bậc với fn,
37
b) Nếu λ = 1 là nghiệm đơn thì u∗
n = ngn,gn là đa thức cùng bậc với fn,
c) Nếu λ = 1 là nghiệm kép thì u∗
n = n2gn,gn là đa thức cùng bậc với fn.
Thay u∗
n vào phương trình, đồng nhất các hệ số của u∗
n, biết u1,u2, từ hệ thức un =
un +u∗
n tính được A,B.
3) Nếu fn = νµn,n ≥ 2 thì giải phương trình đặc trưng aλ2 + bλ + c = 0, tìm
λ. Ta có un = un +u∗
n, trong đó un được tìm như ở 1), hệ số A và B chưa được xác
định, u∗
n xác định như sau:
a) Nếu λ = µ thì u∗
n = kµn,
b) Nếu nghiệm đơn λ = µ thì u∗
n = knµn,
c) Nếu nghiệm kép λ = µ thì u∗
n = kn2µn.
Thay u∗
n vào phương trình, dùng phương pháp đồng nhất hệ số sẽ tính được hệ số k.
Biết u1,u2, từ hệ thức un = un +u∗
n tính được A,B.
4) Nếu fn = f1n + f2n,n ≥ 2 và f1n là đa thức theo n, f2n = νµn thì ta có
un = un +u∗
n +u∗∗
n trong đó un là nghiệm tổng quát của phương trình thuần nhất, u∗
n là
nghiệm riêng tùy của phương trình không thuần nhất aun+1 +bun +cun−1 = f1n,u∗∗
n
là nghiệm riêng tùy ý của phương trình không thuần nhất aun+1 +bun +cun−1 = f2n.
5) Nếu fn = νcosn + µsinn (a = 0),n ≥ 2, thì phương trình đặc trưng aλ2 +
bλ + c = 0, tìm λ. Ta có un = un + u∗
n, trong đó un là nghiệm của phương trình
thuần nhất, xác định như ở 1), các hệ số A và B chưa được xác định
u∗
n = kcosn +lsinn. Thay u∗
n vào phương trình, đồng nhất các hệ số, tính được k,l.
Từ hệ thức un = un +u∗
n và u1,u2, ta tính được A và B.
• Phương trình sai phân tuyến tính cấp ba là phương trình sai phân dạng
u1 = α,u2 = β,u3 = γ,aun+2 +bun+1 +cun +dun−1 = fn,n ≥ 2
trong đó a,b,c,d,α,β,γ là các hằng số, a = 0 và fn là biểu thức của n cho trước.
Phương trình sai phân tuyến tính cấp ba luôn giải được vì phương trình bậc ba
luôn giải được. Nghiệm tổng quát của phương trình sai phân tuyến tính cấp ba có
dạng un = un +u∗
n, trong đó u∗
n là nghiệm của phương trình tuyến tính không thuần
nhất, un là nghiệm tổng quát của phương trình tuyến tính thuần nhất.
Cách giải.
38
1) Xét phương trình đặc trưng aλ3 +bλ2 +cλ +d = 0. (∗)
a) Nếu (∗) có 3 nghiệm thực phân biệt λ1,λ2,λ3 thì
un = β1λn
1 +β2λn
2 +β3λn
3 .
b) Nếu (∗) có một nghiệm thực bội 2 và một nghiệm đơn (λ1 = λ2 = λ3) thì
un = (β1 +β2n)λn
1 +β3λn
3 .
c) Nếu (∗) có một nghiệm thực bội 3 (λ1 = λ2 = λ3)thì
un = (β1 +β2n+β3n2
)λn
1 .
d) Nếu (∗) có một nghiệm thực λ1 và 2 nghiệm phức liên hợp λ2,3 = r(cosϕ ±
isinϕ) thì
un = β1λn
1 +rn
(β2cosnϕ +β3sinnϕ).
2) Gọi u∗
n là một nghiệm riêng của phương trình tuyến tính không thuần nhất.
Xét fn là đa thức của n. Ta có
Nếu λ = 1 thì u∗
n là đa thức cùng bậc với fn,
Nếu λ = 1 (nghiệm đơn) thì u∗
n = ngn,gn là đa thức cùng bậc với fn,
Nếu λ = 1 (bội 2) thì u∗
n = n2gn,gn là đa thức cùng bậc với fn,
Nếu λ = 1 (bội 3) thì u∗
n = n3gn,gn là đa thức cùng bậc với fn.
Xét fn = νµn, ta có
Nếu λ = µ thì u∗
n = k.n.µn,
Nếu nghiệm đơn λ = µ thì u∗
n = k.µn,
Nếu nghiệm bội λ = µ (bội s) thì u∗
n = k.ns.µn.
39
2.1.2. Hệ phương trình sai phân tuyến tính với hệ số hằng
Là hệ phương trình sai phân dạng



xn+1 = pxn +qyn ,x1 = a,
yn+1 = rxn +syn ,y1 = b.
Cách giải. Trong hệ phương trình, thay n bởi n+1, ta nhận được
xn+2 = pxn+1 +qyn+1
= pxn+1 +q(rxn +syn) = pxn+1 +qrxn +s(qyn)
= pxn+1 +qrxn +s(xn+1 − pxn).
Suy ra xn+2 −(p+s)xn+1 +(ps−qr)xn = 0, trong đó x1 = a.
Từ hệ phương trình, ta lại có x2 = px1 + qy1 = pa + qb. Như vậy ta được phương
trình sai phân tuyến tính cấp hai thuần nhất
x1 = a,x2 = pa+qb,xn+1 −(p+s)xn +(ps−qr)xn−1 = 0,n ≥ 2.
Giải phương trình này, ta tìm được xn. Thay xn vào hệ phương trình, ta tìm được yn.
40
2.2. Phương trình sai phân dạng phân tuyến tính với
hệ số hằng
Bài toán 2.2.1. Tìm xn, biết x1 = a > 0 và
xn+1 =
xn
2+xn
(n ∈ N) (2.2.1)
Giải. Nhận xét rằng xn > 0 với mọi n ∈ N. Đặt
1
xn
= yn. Khi đó ta có thể viết
(2.2.1) dưới dạng
yn+1 −2yn = 1.
Suy ra
yn =
(a+1)2n−1 −a
a
và
xn =
a
(a+1)2n−1 −a
.
Bài toán 2.2.2.[2] Tìm xn, biết x1 = a > 0 và
xn+1 = anxn +bn (2.2.2)
trong đó an = 0 với mọi n ∈ N.
Giải. Đặt dãy phụ
xn = yn
n−1
∏
k=0
ak.
Khi đó ta có y1 =
a
a0
và (2.2.2) được viết dưới dạng
yn+1 −yn =
bn
∏n
k=0 ak
. (2.2.3)
Từ đẳng thức (2.2.3) ta nhận được
yn = y1 +
n−1
∑
k=1
bk
∏k
i=0 ai
=
a
a0
+
n−1
∑
k=1
bk
∏k
i=0 ai
Vậy nên ta có
xn =
a
a0
+
n−1
∑
k=1
bk
∏k
i=0 ai
n−1
∏
k=0
ak.
41
Bài toán 2.2.3.[2] Xác định số hạng tổng quát của dãy số {xn}, nếu xn thỏa mãn
phương trình dãy sau đây
x0 = a,xn+1 =
pxn +q
rxn +s
,n ∈ N (2.2.4)
trong đó p,q,r,s ∈ R cho trước.
Giải. Giả sử un,vn là một nghiệm của hệ phương trình sai phân



un+1 = pun +qvn ,u0 = a,
vn+1 = run +svn ,v0 = 1,
(2.2.5)
thì xn =
un
vn
là nghiệm của phương trình (2.2.4). Thật vậy, ta chứng minh bằng
quy nạp như sau
x0 =
u0
v0
=
a
1
= a,(đúng).
Giả sử xn =
un
vn
là nghiệm của (2.2.4). Khi đó
xn+1 =
un+1
vn+1
=
pun +qvn
run +svn
=
un
vn
+q
r
un
vn
+s
=
pxn +q
rxn +s
cũng là nghiệm của (2.2.4).
Giải hệ (2.2.5) bằng cách đưa về phương trình sai phân thuần nhất
yn+2 = (p+s)yn+1 +(qr −sp)yn.
Bài toán 2.2.4.[2] Tìm xn thỏa mãn điều kiện
x1 = a, xn+1 =
x2
n +d
2xn
, n ∈ N∗
(2.2.6)
Giải. Khi d =0 thì ta có ngay xn+1 =
1
2
xn và xn = (
1
2
)n−1a
Xét trường hợp d > 0. Giả sử un, vn là nghiệm của hệ phương trình sai phân



un+1 = u2
n +dv2
n,
vn+1 = 2unvn, u1 = a, v1 = a
(2.2.7)
42
thì xn =
un
vn
là nghiệm của phương trình (2.2.6). Thật vậy, ta chứng minh bằng quy
nạp như sau
x1 =
u1
v1
=
a
1
= a ( đúng )
Giả sử xn =
un
vn
là nghiệm của (2.2.6). Khi đó
xn+1 =
un+1
vn+1
=
u2
n +dv2
n
2unvn
=
x2
n +d
2xn
cũng là nghiệm của (2.2.6)
Như vậy để giải phương trình (2.2.6) ta cần giải hệ (2.2.7). Ta có



un+1 = u2
n +dv2
n
√
dvn+1 = 2
√
dunvn, u1 = a, v1 = 1.
(2.2.8)
Cộng vế với vế của các phương trình trong hệ (2.2.8) ta thu được
un+1 +
√
dvn+1 = (un +
√
dvn)2
Do đó
un+1 +
√
dvn+1 = (u1 +
√
dv1)2n
= (a+
√
d)2n
(2.2.9)
Tương tự, trừ vế với vế của các phương trình trong hệ (2.2.8), ta cũng có
un+1 −
√
dvn+1 = (u1 −
√
dv1)2n
= (a−
√
d)2n
. (2.2.10)
Từ (2.2.9) và (2.2.10) ta có



un+1 =
1
2
(a+
√
d)2n
+(a−
√
d)2n
vn+1 =
1
2
√
d
(a+
√
d)2n
−(a−
√
d)2n
.
(2.2.11)
Do xn =
un
vn
nên từ (2.2.11), ta có
xn =
√
d(a+
√
d)2n−1
+(a−
√
d)2n−1
(a+
√
d)2n−1
−(a−
√
d)2n−1
.
Thử lại ( bằng quy nạp ) cho thấy kết quả nhận được thỏa mãn (2.2.6).
43
Xét trường hợp d< 0. Đặt d = -q, q >0. Giả sử un, vn là một nghiệm của hệ
phương trình sai phân



un+1 = u2
n −qv2
n,
vn+1 = 2unvn, u1 = a, v1 = 1,
(2.2.12)
thì vn =
un
vn
là nghiệm của phương trình (2.2.6)
Thật vậy, ta chứng minh bằng quy nạp như sau
x1 =
u1
v1
=
a
1
= a (đúng )
Giả sử xn =
un
vn
là nghiệm của (2.2.6). Khi đó
xn+1 =
un+1
vn+1
=
u2
n −qv2
n
2unvn
=
x2
n +d
2xn
,
cũng là nghiệm của (2.2.6).
Để giải (2.2.6) ta giải hệ (2.2.12). Ta có



un+1 = u2
n −qv2
n
i
√
qvn+1 = 2i
√
qunvn, u1 = a, v1 = 1,
(2.2.13)
Cộng vế với vế của các phương trình trong hệ (2.2.13) ta thu được
un+1 +i
√
qvn+1 = (un +i
√
qvn)2
.
Do đó
un+1 +i
√
qvn+1 = (u1 +i
√
qv1)2n
= (a+i
√
q)2n
. (2.2.14)
Tương tự, trừ vế với vế của các phương trình trong hệ (2.2.13), ta cũng có
un+1 −i
√
qvn+1 = (u1 −i
√
qv1)2n
= (a−i
√
q)2n
. (2.2.15)
Từ (2.2.14) và (2.2.15) ta có



un+1 =
1
2
(a+i
√
q)2n
+(a−i
√
q)2n
vn+1 =
1
2i
√
q
(a+i
√
q)2n
−(a−i
√
q)2n
.
44
Do xn =
un
vn
nên từ hệ nghiệm này, ta thu được
xn =
i
√
q(a+i
√
q)2n−1
+(a−i
√
q)2n−1
(a+i
√
q)2n−1
−(a−i
√
q)2n−1 .
Thử lại ( bằng quy nạp ) cho thấy kết quả nhận được thỏa mãn (2.2.6).
Bài toán 2.2.5.[2] Tìm xn thỏa mãn điều kiện
x1 = a,xn+1 =
2xn
1+dx2
n
,n ∈ N∗
(2.2.16)
Giải. Trước hết ta xét trường hợp d = 0. Khi đó xn+1 = 2xn và xn = 2n−1a.
Xét trường hợp d > 0. Giả sử un,vn là một nghiệm của hệ phương trình sai phân



un+1 = u2
n +dv2
n,
vn+1 = 2unvn ,u1 = 1,v1 = a.
(2.2.17)
thì xn =
un
vn
là nghiệm của phương trình (2.2.16) (chứng minh bằng phương pháp
qui nạp). Như vậy để giải phương trình (2.2.16) ta cần giải hệ (2.2.17). Ta có



un+1 = u2
n +dv2
n,
√
dvn+1 = 2
√
dunvn ,u1 = 1,v1 = a.
(2.2.18)
Cộng vế với vế của các phương trình trong hệ (2.2.18) ta thu được
un+1 +
√
dvn+1 = (un +
√
dvn)2
.
Do đó
un+1 +
√
dvn+1 = (u1 +
√
dv1)2n
= (1+a
√
d)2n
. (2.2.19)
Tương tự, trừ vế với vế của các phương trình trong hệ (2.2.18), ta cũng có
un+1 −
√
dvn+1 = (u1 −
√
dv1)2n
= (1−a
√
d)2n
. (2.2.20)
Từ (2.2.19) và (2.2.20) ta có



un = [(1+a
√
d)2n−1
+(1−a
√
d)2n−1
],
vn =
√
d[(1+a
√
d)2n−1
−(1−a
√
d)2n−1
].
(2.2.21)
45
Do xn =
un
vn
nên từ (2.2.21), ta thu được nghiệm của bài toán có dạng
xn =
[(1+a
√
d)2n−1
+(1−a
√
d)2n−1
]
√
d[(1+a
√
d)2n−1
−(1−a
√
d)2n−1
]
.
Tương tự đối với trường hợp d < 0. Đặt d = −q, q > 0. Giả sử un,vn là một nghiệm
của hệ phương trình sai phân



un+1 = u2
n −qv2
n,
vn+1 = 2unvn ,u1 = 1,v1 = a.
(2.2.22)
thì xn =
un
vn
là nghiệm của phương trình (2.2.16) (chứng minh bằng phương pháp
qui nạp). Như vậy, để giải phương trình (2.2.16) ta cần giải hệ (2.2.22). Ta có



un+1 = u2
n −qv2
n,
i
√
qvn+1 = 2i
√
qunvn ,u1 = 1,v1 = a.
(2.2.23)
Cộng vế với vế của các phương trình trong hệ (2.2.23) ta thu được
un+1 +i
√
qvn+1 = (un +i
√
qvn)2
.
Do đó
un +i
√
qvn = (u1 +i
√
qv1)2n−1
= (1+ai
√
q)2n−1
. (2.2.24)
Tương tự, trừ vế với vế của các phương trình trong hệ (2.2.23), ta cũng có
un −i
√
qvn = (u1 −i
√
qv1)2n−1
= (1−ai
√
q)2n−1
. (2.2.25)
Từ (2.2.24) và (2.2.25), ta có
xn =
[(1+ai
√
q)2n−1
+(1−ai
√
q)2n−1
]
i
√
q[(1+ai
√
q)2n−1
−(1−ai
√
q)2n−1
]
.
Thử lại (bằng qui nạp) cho thấy kết quả nhận được thỏa mãn (2.2.16).
46
Sau đây ta xét một số bài toán cụ thể.
Bài toán 2.2.6. Tìm các dãy xn, yn thỏa mãn hệ phương trình



xn+1 = 2xn +yn, x0 = 1
yn+1 = xn +2yn, y0 = 1
(2.2.26)
Giải. Từ hệ phương trình (2.2.26) ta suy ra
xn+2 = 4xn+1 −3xn, x0 = 1 và x1 = 3
Phương trình đặc trưng λ2 −4λ +3 = 0 có hai nghiệm thực phân biệt λ1 = 1, λ2 = 3.
Do đó xn = A1n +B3n
Mà x0 = 1, x1 = 3 nên ta có A = 0, B = 1
Suy ra 


xn = 3n
yn = 3n
Bài toán 2.2.7. Tìm các dãy xn, yn thỏa mãn hệ phương trình



xn+1 = 5xn +yn, x0 = 1
yn+1 = −4xn +yn, y0 = 1
(2.2.27)
Giải. Từ hệ phương trình (2.2.27) ta suy ra
xn+2 = 6xn+1 −9xn, x0 = 1 và x1 = 6.
Phương trình đặc trưng λ2 −6λ +9 = 0 có nghiệm kép λ = 3.
Do đó xn = (A+Bn)3n.
Mà x0 = 1, x1 = 6 nên ta có A = 1, B = 1.
Suy ra 


xn = 3n +n3n
yn = −12.3n −12n.3n.
Bài toán 2.2.8. Tìm các dãy xn, yn thỏa mãn hệ phương trình



xn+1 = xn −yn, x0 = 1
yn+1 = xn +yn, y0 = 1
(2.2.28)
47
Giải. Từ hệ phương trình (2.2.28) ta suy ra
xn+2 = 2xn+1 −2xn, x0 = 1 và x1 = 0.
Phương trình đặc trưng λ2 −2λ +2 = 0 có hai nghiệm phức phân biệt λ1,2 = 1±i.
Ta có
λ1 = 1+i =
√
2(cos
π
4
+isin
π
4
),
λ2 = 1−i =
√
2(cos
π
4
−isin
π
4
)
Do đó xn =
√
2n(Acos
nπ
4
+Bsin
nπ
4
).
Mà x0 = 1, x1 = 0 nên ta có A = 1, B = −1.
Suy ra



xn =
√
2n+1 cos(n+1)
π
4
yn =
√
2n+1 sin(n+1)
π
4
.
Bài toán 2.2.9. Tìm xn, biết rằng
x0 = 0,xn+1 =
xn −2
xn +4
,n ∈ N. (2.2.29)
Giải. Xét hệ phương trình sai phân



yn+1 = yn +2zn,y0 = 0
zn+1 = yn +4zn,z0 = 1
Khi đó xn =
yn
zn
là nghiệm của phương trình (2.2.29),
suy ra yn+2 = 5yn+1 −6yn, y0 = 0,y1 = −2.
Phương trình đặc trưng λ2 −5λ +6 = 0 có hai nghiệm thực phân biệt λ1 = 2,λ2 = 3.
Do đó yn = A.2n + B.3n. Mà y0 = 0,y1 = −2 nên ta có A = 2,B = −2. Suy ra
yn = 2.2n −2.3n;zn =
1
2
(yn −yn+1) = 2.3n −2n.
Vậy xn =
yn
zn
=
2.2n −2.3n
−2n +2.3n
.
Bài toán 2.2.10. Tìm xn, biết rằng
x0 = 1,xn+1 =
xn −1
xn +3
,n ∈ N. (2.2.30)
Giải. Xét hệ phương trình sai phân
48



yn+1 = yn −zn,y0 = 1
zn+1 = yn +3zn,z0 = 1
Khi đó xn =
yn
zn
là nghiệm của phương trình (2.2.30),
suy ra yn+2 = 4yn+1 −4yn, y0 = 1,y1 = 0.
Phương trình đặc trưng λ2 −4λ +4 = 0 có nghiệm kép λ = 2.
Do đó yn = (A+Bn)2n. Mà y0 = 1,y1 = 0 nên ta có A = 1,B = −1.
Suy ra yn = 2n −n.2n; zn = 2n +n.2n.
Vậy xn =
yn
zn
=
1−n
1+n
.
Bài toán 2.2.11. Tìm xn, biết rằng
x0 = 0,xn+1 =
xn −3
xn +1
,n ∈ N∗
. (2.2.31)
Giải. Xét hệ phương trình sai phân



yn+1 = yn −3zn,y0 = 0
zn+1 = yn +zn,z0 = 1
Khi đó xn =
yn
zn
là nghiệm của phương trình (2.2.31),
suy ra yn+2 = 2yn+1 −4yn,y0 = 0,y1 = −3.
Phương trình đặc trưng λ2 −2λ +4 = 0 có hai nghiệm phức λ1,2 = 1±i
√
3. Ta có
λ1 = 1+i
√
3 = 2(cos
π
3
+isin
π
3
)
λ2 = 1−i
√
3 = 2(cos
π
3
−isin
π
3
)
Do đó yn = 2n(Acosnπ
3 +Bsinnπ
3 ). Mà y0 = 1,y1 = −3 nên ta có A = 0,B = −
√
3.
Suy ra yn = −
√
3.2n.sinnπ
3 ;zn =
1
3
(yn −yn+1) = 2ncosnπ
3
Vậy xn =
yn
zn
= −
√
3tannπ
3 .
49
2.3. Giới hạn của một số dãy truy hồi dạng phân
tuyến tính.
Bài toán 2.3.1.[3] Xét dãy số {xn} được xác định bởi phương trình sau
xn+1 = 1+
1
1+xn
,x0 = 1 (2.3.32)
Chứng minh rằng lim
n→∞
xn =
√
2
Giải. Xét hàm số
f(x) = 1+
1
1+x
.
Ta có nhận xét rằng 1 < f(x) ≤ 2, f(x) là hàm dương, liên tục và nghịch biến trên
[0;+∞).
Từ hệ thức
xn+1 = 1+
1
1+xn
= f(xn),
ta suy ra 1 < xn+1 ≤ 2,∀n ∈ {0,1,2,...}.
Ta cần xét hai trường hợp
Trường hợp 1: x0 < x2
Suy ra f(x0) > f(x2) hay x1 > x3.
Suy ra f(x1) < f(x3) hay x2 < x4.
Suy ra f(x2) > f(x4) hay x3 > x5.
...
Bằng qui nạp ta thu được x2n < x2n+2.
Thật vậy, giả sử x2k < x2k+2 thì f(x2k) > f(x2k+2) hay x2k+1 > x2k+3.
Suy ra f(x2k+1) < f(x2k+3) hay x2k+2 < x2k+4.
Vậy {x2n} là dãy tăng.
Chứng minh tương tự ta được {x2n+1} là dãy giảm.
Từ đó, ta có
- Dãy {x2n} tăng và bị chặn trên nên {x2n} hội tụ. Giả sử x2n → α khi n → ∞.
- Dãy {x2n+1} giảm và bị chặn dưới nên {x2n+1} hội tụ. Giả sử x2n+1 → β khi
n → ∞.
Do hàm f liên tục nên
50
x2n+1
n→∞
−−−−→ β
||
f(x2n)
n→∞
−−−−→ f(α)
Suy ra f(α) = β.
Tương tự, ta cũng có f(β) = α và ta thu được hệ phương trình



β = f(α)
α = f(β)
tương đương với 


β = 1+
1
1+α
α = 1+
1
1+β
từ đó suy ra 


αβ +β = 2+α
αβ +β = 2+β
và do đó ta nhận được α = β =
√
2.
Trường hợp 2: x0 ≥ x2.
Ta thu được
- Dãy {x2n} giảm và bị chặn dưới nên {x2n} hội tụ. Giả sử x2n → α khi n → ∞.
- Dãy {x2n+1} tăng và bị chặn trên nên {x2n+1} hội tụ. Giả sử x2n+1 → β khi n → ∞.
Tương tự như trường hợp 1, ta cũng thu được α = β =
√
2.
Vậy lim
n→∞
xn =
√
2
Bài toán 2.3.2.[3] Cho f là hàm dương, liên tục và nghịch biến trên [0;+∞). Giả
sử rằng hệ phương trình 


β = f(α)
α = f(β)
có nghiệm duy nhất α = β = l. Chứng minh rằng dãy số dương
xn+1 = f(xn)
với x0 > 0 cho trước hội tụ tới l.
51
Giải. Ta xét 2 trường hợp
Trường hợp 1:x0 < x2
Suy ra f(x0) > f(x2) hay x1 > x3.
Suy ra f(x1) < f(x3) hay x2 < x4.
Suy ra f(x2) > f(x4) hay x3 > x5.
...
Bằng quy nạp có thể chứng minh được x2n < x2n+2.
Thật vậy, giả sử x2k < x2k+2
Khi đó thì f(x2k) > f(x2k+2) hay x2k+1 > x2k+3,
Do đó f(x2k+1) < f(x2k+3) hay x2k+2 < x2k+4.
Vậy {x2n} là dãy tăng.
Chứng minh tương tự ta được {x2n+1} là dãy giảm.
Từ đó, ta có
- Dãy {x2n} tăng và bị chặn trên nên {x2n} hội tụ. Giả sử x2n → α khi n → ∞.
- Dãy {x2n+1} giảm và bị chặn dưới nên {x2n+1} hội tụ. Giả sử x2n+1 → β khi
n → ∞.
Do hàm f liên tục nên
x2n+1
n→∞
−−−−→ β
||
f(x2n)
n→∞
−−−−→ f(α)
Suy ra f(α) = β.
Tương tự, ta cũng thu được f(β) = α
Vậy ta có hệ phương trình



β = f(α)
α = f(β)
Theo giả thiết hệ 


β = f(α)
α = f(β)
có nghiệm duy nhất α = β = l.
Trường hợp 2: x0 ≥ x2.
52
Ta dễ dàng kiểm tra
- Dãy {x2n} giảm và bị chặn dưới nên {x2n} hội tụ. Giả sử x2n → α khi n → ∞.
- Dãy {x2n+1} tăng và bị chặn trên nên {x2n+1} hội tụ. Giả sử x2n+1 → β khi n → ∞.
Tương tự như trường hợp 1 ta cũng thu được α = β = l.
Vậy lim
n→∞
xn = l.
Bài toán 2.3.3. Xét dãy số cho bởi phương trình sai phân sau
xn+1 = 1+
2
1+xn
,x0 > 0.
Khảo sát sự hội tụ của dãy {xn}.
Giải. Xét hàm số
f(x) = 1+
2
1+x
.
Dễ thấy hàm số f(x) dương, liên tục và nghịch biến trên [0;+∞) và thỏa mãn hệ
phương trình 


β = f(α)
α = f(β).
Tương đương với hệ phương trình



αβ = α −β +3
αβ = β −α +3
và giải hệ phương trình trên ta nhận được α = β =
√
3.
Do đó theo kết quả của bài toán 2.3.2, ta có dãy {xn} có giới hạn là
√
3 khi n → ∞
Vậy lim
n→∞
xn =
√
3.
Bài toán 2.3.4.[3] Xét dãy số {xn} xác định theo công thức



x0 = 1
xn+1 =
2
xn
+
√
3
x2
n
∀n ∈ N.
Chứng minh rằng dãy số {xn} không có giới hạn hữu hạn.
Giải. Nhận xét rằng nếu dãy số {xn} có giới hạn a thì a là nghiệm của phương trình
x =
2
x
+
√
3
x2
.
53
Do đó a =
√
3.
Ta có x0 = 1 > x2.
Ta chứng minh qui nạp rằng
x2n > x2n+2.
Thật vậy
x2k+1 =
2
x2k
+
√
3
x2
2k
<
2
x2k+2
+
√
3
x2
2k+2
= x2k+3.
Suy ra
x2k+2 =
2
x2k+1
+
√
3
x2
2k+1
>
2
x2k+3
+
√
3
x2
2k+3
= x2k+4.
Do vậy
1 = x0 > x2 > ... > x2n > ... > 0.
suy ra dãy {x2n} là dãy đơn điệu giảm và bị chặn dưới bởi 0, nên tồn tại giới hạn
hữu hạn b = lim
n→∞
x2n
Mặt khác do x0 = 1 nên b ≤ 1.
Vì tồn tại giới hạn hữu hạn a = lim
n→∞
xn =
√
3, nên nói riêng
lim
n→∞
x2n = lim
n→∞
xn =
√
3
Tức là b =
√
3 . Đây là điều vô lý. Vậy dãy số {xn} không có giới hạn hữu hạn.
Bài toán 2.3.5.[6] Xét dãy số {xn} xác định như sau



u0 = 1
un =
−1
3+un−1
với n = 1,2,...
Chứng minh rằng dãy số {un} có giới hạn và hãy tìm lim
n→∞
un .
Giải. Ta có un −un+1 = un +
1
3+un
=
u2
n +3un +1
3+un
.
Bây giờ ta chứng minh rằng un >
−3+
√
5
2
(*) với mọi n = 1,2,... bằng qui nạp
như sau
- Với n = 0, thì u0 = 1 >
−3+
√
5
2
( đúng ).
- Giả sử (*) đúng với n = k (k = 1,2,...), nghĩa là ta có ta có uk >
−3+
√
5
2
.
54
Khi đó: 3+uk >
−3+
√
5
2
=
3+
√
5
2
,
suy ra
1
3+uk
<
2
3+
√
5
=
3−
√
5
2
.
Do đó uk+1 = −
1
3+uk
>
−3+
√
5
2
.
Vậy cũng đúng với n=k+1.
Theo nguyên lí quy nạp suy ra đúng với mọi n.
Vì un >
−3+
√
5
2
với mọi n, nên 3 + un >
3+
√
5
2
, tức là 3 + un > 0 với mọi
n = 0,1,2,...
Do un >
−3+
√
5
2
, nên theo định lí thuận về dấu tam thức bậc hai thì u2
n +3un +1 >
0 với mọi n = 0,1,2,...
Mà un −un+1 =
u2
n +3un +1
3+un
, suy ra un > un+1 với mọi n, nghĩa là dãy {un} là đơn
điệu giảm và bị chặn dưới bởi
−3+
√
5
2
.
Suy ra tồn tại giới hạn của dãy {un} khi n → ∞, và đặt lim
n→∞
un = a.
Từ un =
−1
3+un−1
và lấy giới hạn 2 vế khi n → ∞, ta có:
lim
n→∞
un =
−1
3+ lim
n→∞
un−1
hay
a =
−1
3+a
⇔ a2
+3a+1 = 0 ⇔ a =
−3±
√
5
2
.
Vì un >
−3+
√
5
2
với mọi n, nên a = lim
n→∞
un ≥
−3+
√
5
2
.
Suy ra a =
−3+
√
5
2
.
Vậy lim
n→∞
un =
−3+
√
5
2
.
55
2.4. Bài tập tham khảo.
Bài tập 2.4.1. Tìm xn, biết rằng
x0 = 1,xn+1 =
1−4xn
1−6xn
,n ∈ N.
Bài tập 2.4.2. Tìm xn, biết rằng
x0 = −1,xn+1 =
2xn −3
3xn −4
,n ∈ N.
Bài tập 2.4.3. Tìm xn, biết rằng
x0 = 0,xn+1 =
xn +1
4−xn
,n ∈ N.
Bài tập 2.4.4. Cho a ∈ R{1}. Xét dãy số {xn} được xác định theo công thức



x1 = a
xn+1 =
xn(x2
n +3)
3x2
n +1
∀n ∈ N∗.
Chứng minh rằng dãy số {yn} = {(a−1)xn} có giới hạn và xác định giới hạn đó.
Bài tập 2.4.5. Tính giới hạn của dãy {xn} với
xn+1 =
1
1+xn
,x0 = 1.
Bài tập 2.4.6. Tính giới hạn của dãy {xn} với
xn+1 =
xn
1+x2
n
,x0 > 0.
Bài tập 2.4.7. Tính giới hạn của dãy {xn} với
xn+1 =
x2
n +3
2(xn +1)
,x0 > 0.
Bài tập 2.4.8. Tính giới hạn của dãy {zn} với
zn = yn+1 −yn,yn =
1
x2
n
,x0 > 0.
56
Bài tập 2.4.9. Dãy số {un} xác định như sau



u1 =
√
3
3
un+1 =
un +2−
√
3
1+(
√
3−2)un
; n = 2,3,...
Tìm công thức cho un, sau đó tính u2010.
Bài tập 2.4.10. Cho p,q là hai số nguyên cho trước. Dãy {un} được xác định
như sau



u1 = p+q−1
un+1 = p+q−
pq
un
; n = 1,2,...
Đặt Pn = u1u2 ...un. Chứng minh rằng vối mọi n = 1,2,··· thì Pn là số nguyên.
Bài tập 2.4.11. Dãy số {un},n = 1,2,... được xác định như sau:



u1 =
2
3
un+1 =
un
2(2n+1)un +1
; n = 1,2,...
Tính tổng S = ∑2010
i=1 ui.
Bài tập 2.4.12. Dãy số {un},n = 1,2,... được xác định như sau:



u1 = 2
un+1 = 1+
1
un
; n = 1,2,...
Gọi p là số lẻ và q là số chẵn bất kỳ. Chứng minh Up > Uq.
57
KẾT LUẬN
Sau thời gian học tập hai năm tại khoa Toán-Cơ-Tin học, Trường Đại học Khoa
học Tự nhiên Hà Nội, được sự giúp đỡ chỉ bảo của các thầy cô trong khoa, đặc biệt
là PGS. TS. Nguyễn Minh Tuấn, tác giả đã hoàn thành luận văn với đề tài: " Phép
biến đổi phân tuyến tính và áp dụng giải một số bài toán phổ thông ".
Luận văn đã đạt được một số kết quả sau:
1. Luận văn đã nêu được khá đầy đủ các bài toán tổng quát về phương trình hàm
sinh bởi hàm phân tuyến tính w(x) =
ax+b
cx+d
, c = 0 ,ad −bc = 0 , đồng thời cũng
tập hợp được các bài toán khác về hàm phân tuyến tính trong các kỳ thi Olympic.
2. Luận văn đã trình bày được các bài toán điển hình về phương trình sai phân
dạng phân tuyến tính với hệ số hằng và giới hạn của một số dãy sai phân dạng phân
tuyến tính.
3. Luận văn có thể làm tài liệu tham khảo bổ ích cho quá trình nghiên cứu, giảng
dạy và học tập về phép biến đổi phân tuyến tính ở bậc phổ thông.
Trong mỗi phần của luận văn tác giả đều đưa ra rất nhiều ví dụ minh họa cho
mỗi phần.
Mặc dù đã cố gắng hết sức và nghiêm túc trong quá trình nghiên cứu, nhưng
do thời gian và khả năng còn hạn chế nên chắc chắn luận văn không tránh khỏi
những thiếu sót. Một lần nữa, tác giả rất mong nhận được những ý kiến đóng góp
của các thầy cô giáo và bạn bè đồng nghiệp. Trong thời gian tới, tác giả sẽ cố gắng
tiếp tục nghiên cứu phép biến đổi này trong quá trình giảng dạy và học tập của mình.
58
Tài liệu tham khảo
[1] Nguyễn Văn Mậu, 1997, Phương trình hàm, Nhà xuất bản Giáo dục.
[2] Nguyễn Văn Mậu, 2003, Mốt số bài toán chọn lọc về dãy số, Nhà xuất bản
Giáo dục.
[3] Nguyễn Văn Mậu, Nguyễn Thủy Thanh, 2008, Giới hạn dãy số và hàm số,
Nhà xuất bản Giáo dục.
[4] Nguyễn Trọng Tuấn, 2005, Bài toán hàm số qua các kỳ thi Olympic, Nhà xuất
bản Giáo dục.
[5] Lê Đình Thịnh và Lê Đình Định, 2004, Phương trình sai phân, Nhà xuất bản
Đại học Quốc Gia Hà Nội.
[6] Phan Huy Khải, 2007, Các bài toán về dãy số, Nhà xuất bản Giáo dục .
59

More Related Content

What's hot

bai tap hinh hoc xa anh-pham binh do
bai tap hinh hoc xa anh-pham binh dobai tap hinh hoc xa anh-pham binh do
bai tap hinh hoc xa anh-pham binh doBui Loi
 
Bai7 khai trien_taylor
Bai7 khai trien_taylorBai7 khai trien_taylor
Bai7 khai trien_taylorljmonking
 
Do do tich-phan-thai_thuan_quang mearsure and intergral
Do do tich-phan-thai_thuan_quang mearsure and intergralDo do tich-phan-thai_thuan_quang mearsure and intergral
Do do tich-phan-thai_thuan_quang mearsure and intergralBui Loi
 
kỹ thuật giải phương trình hàm
kỹ thuật giải phương trình hàmkỹ thuật giải phương trình hàm
kỹ thuật giải phương trình hàmljmonking
 
De xstk k11
De xstk k11De xstk k11
De xstk k11dethinhh
 
ứNg dụng tích phân tính diện tích và thể tích
ứNg dụng tích phân tính diện tích và thể tíchứNg dụng tích phân tính diện tích và thể tích
ứNg dụng tích phân tính diện tích và thể tíchThế Giới Tinh Hoa
 
Khong gian vecto (chuong 3)
Khong gian vecto (chuong 3)Khong gian vecto (chuong 3)
Khong gian vecto (chuong 3)Nguyễn Phụng
 
Tính toán khoa học - Chương 4: Giải phương trình phi tuyến
Tính toán khoa học - Chương 4: Giải phương trình phi tuyếnTính toán khoa học - Chương 4: Giải phương trình phi tuyến
Tính toán khoa học - Chương 4: Giải phương trình phi tuyếnChien Dang
 
Bo de-thi-va-loi-giai-xac-xuat-thong-ke
Bo de-thi-va-loi-giai-xac-xuat-thong-keBo de-thi-va-loi-giai-xac-xuat-thong-ke
Bo de-thi-va-loi-giai-xac-xuat-thong-keNam Cengroup
 
Toan a1 -_bai_giang
Toan a1 -_bai_giangToan a1 -_bai_giang
Toan a1 -_bai_giangxuanhoa88
 
Tính toán khoa học - Chương 8: Quy hoạch tuyến tính
Tính toán khoa học - Chương 8: Quy hoạch tuyến tínhTính toán khoa học - Chương 8: Quy hoạch tuyến tính
Tính toán khoa học - Chương 8: Quy hoạch tuyến tínhChien Dang
 
Topo daicuong1[1]
Topo daicuong1[1]Topo daicuong1[1]
Topo daicuong1[1]Bui Loi
 
chuong 4. dai so boole
chuong 4.  dai so boolechuong 4.  dai so boole
chuong 4. dai so boolekikihoho
 
Diophantine equations Phương trình diophant
Diophantine equations Phương trình diophantDiophantine equations Phương trình diophant
Diophantine equations Phương trình diophantBui Loi
 

What's hot (20)

03 ma tran nghich dao
03 ma tran nghich dao03 ma tran nghich dao
03 ma tran nghich dao
 
bai tap hinh hoc xa anh-pham binh do
bai tap hinh hoc xa anh-pham binh dobai tap hinh hoc xa anh-pham binh do
bai tap hinh hoc xa anh-pham binh do
 
Bai7 khai trien_taylor
Bai7 khai trien_taylorBai7 khai trien_taylor
Bai7 khai trien_taylor
 
Chuong01
Chuong01Chuong01
Chuong01
 
Do do tich-phan-thai_thuan_quang mearsure and intergral
Do do tich-phan-thai_thuan_quang mearsure and intergralDo do tich-phan-thai_thuan_quang mearsure and intergral
Do do tich-phan-thai_thuan_quang mearsure and intergral
 
kỹ thuật giải phương trình hàm
kỹ thuật giải phương trình hàmkỹ thuật giải phương trình hàm
kỹ thuật giải phương trình hàm
 
De xstk k11
De xstk k11De xstk k11
De xstk k11
 
ứNg dụng tích phân tính diện tích và thể tích
ứNg dụng tích phân tính diện tích và thể tíchứNg dụng tích phân tính diện tích và thể tích
ứNg dụng tích phân tính diện tích và thể tích
 
Khong gian vecto (chuong 3)
Khong gian vecto (chuong 3)Khong gian vecto (chuong 3)
Khong gian vecto (chuong 3)
 
Tính toán khoa học - Chương 4: Giải phương trình phi tuyến
Tính toán khoa học - Chương 4: Giải phương trình phi tuyếnTính toán khoa học - Chương 4: Giải phương trình phi tuyến
Tính toán khoa học - Chương 4: Giải phương trình phi tuyến
 
Bo de-thi-va-loi-giai-xac-xuat-thong-ke
Bo de-thi-va-loi-giai-xac-xuat-thong-keBo de-thi-va-loi-giai-xac-xuat-thong-ke
Bo de-thi-va-loi-giai-xac-xuat-thong-ke
 
Toan a1 -_bai_giang
Toan a1 -_bai_giangToan a1 -_bai_giang
Toan a1 -_bai_giang
 
Luận văn: Một số lớp bài toán về phương trình hàm, HAY, 9đ
Luận văn: Một số lớp bài toán về phương trình hàm, HAY, 9đLuận văn: Một số lớp bài toán về phương trình hàm, HAY, 9đ
Luận văn: Một số lớp bài toán về phương trình hàm, HAY, 9đ
 
Tính toán khoa học - Chương 8: Quy hoạch tuyến tính
Tính toán khoa học - Chương 8: Quy hoạch tuyến tínhTính toán khoa học - Chương 8: Quy hoạch tuyến tính
Tính toán khoa học - Chương 8: Quy hoạch tuyến tính
 
Topo daicuong1[1]
Topo daicuong1[1]Topo daicuong1[1]
Topo daicuong1[1]
 
Luận văn: Lý thuyết đồ thị với các bài toán phổ thông, HAY, 9đ
Luận văn: Lý thuyết đồ thị với các bài toán phổ thông, HAY, 9đLuận văn: Lý thuyết đồ thị với các bài toán phổ thông, HAY, 9đ
Luận văn: Lý thuyết đồ thị với các bài toán phổ thông, HAY, 9đ
 
Ch3 ma tran
Ch3 ma tranCh3 ma tran
Ch3 ma tran
 
chuong 4. dai so boole
chuong 4.  dai so boolechuong 4.  dai so boole
chuong 4. dai so boole
 
Diophantine equations Phương trình diophant
Diophantine equations Phương trình diophantDiophantine equations Phương trình diophant
Diophantine equations Phương trình diophant
 
Chuong5
Chuong5Chuong5
Chuong5
 

Similar to Luận văn: Phép biến đổi phân tuyến tính, HAY, 9đ

Luận văn: Một số lớp bài toán về loại phương trình hàm, HAY - Gửi miễn phí qu...
Luận văn: Một số lớp bài toán về loại phương trình hàm, HAY - Gửi miễn phí qu...Luận văn: Một số lớp bài toán về loại phương trình hàm, HAY - Gửi miễn phí qu...
Luận văn: Một số lớp bài toán về loại phương trình hàm, HAY - Gửi miễn phí qu...Dịch vụ viết bài trọn gói ZALO: 0909232620
 
Vận dụng giới hạn dãy số trong giải phương trình hàm.pdf
Vận dụng giới hạn dãy số trong giải phương trình hàm.pdfVận dụng giới hạn dãy số trong giải phương trình hàm.pdf
Vận dụng giới hạn dãy số trong giải phương trình hàm.pdfvongoccuong
 
Math educare] toan a1-giai tich ham mot bien_giai tich ham nhieu bien_phuong ...
Math educare] toan a1-giai tich ham mot bien_giai tich ham nhieu bien_phuong ...Math educare] toan a1-giai tich ham mot bien_giai tich ham nhieu bien_phuong ...
Math educare] toan a1-giai tich ham mot bien_giai tich ham nhieu bien_phuong ...Nguyen Vietnam
 

Similar to Luận văn: Phép biến đổi phân tuyến tính, HAY, 9đ (20)

Luận văn: Một số lớp bài toán về loại phương trình hàm, HAY - Gửi miễn phí qu...
Luận văn: Một số lớp bài toán về loại phương trình hàm, HAY - Gửi miễn phí qu...Luận văn: Một số lớp bài toán về loại phương trình hàm, HAY - Gửi miễn phí qu...
Luận văn: Một số lớp bài toán về loại phương trình hàm, HAY - Gửi miễn phí qu...
 
Luận văn: Phương pháp giải bài toán cực trị và ứng dụng, HAY
Luận văn: Phương pháp giải bài toán cực trị và ứng dụng, HAYLuận văn: Phương pháp giải bài toán cực trị và ứng dụng, HAY
Luận văn: Phương pháp giải bài toán cực trị và ứng dụng, HAY
 
Luận văn: Phương pháp giải bài toán cực trị, HAY
Luận văn: Phương pháp giải bài toán cực trị, HAYLuận văn: Phương pháp giải bài toán cực trị, HAY
Luận văn: Phương pháp giải bài toán cực trị, HAY
 
Luận văn: Một số phương pháp giải phương trình hàm, HOT, 9đ
Luận văn: Một số phương pháp giải phương trình hàm, HOT, 9đLuận văn: Một số phương pháp giải phương trình hàm, HOT, 9đ
Luận văn: Một số phương pháp giải phương trình hàm, HOT, 9đ
 
Luận văn: Một số phương pháp giải phương trình hàm, HOT, 9đ
Luận văn: Một số phương pháp giải phương trình hàm, HOT, 9đLuận văn: Một số phương pháp giải phương trình hàm, HOT, 9đ
Luận văn: Một số phương pháp giải phương trình hàm, HOT, 9đ
 
Luận văn: Phương pháp giải phương trình chứa ẩn dưới dấu căn
Luận văn: Phương pháp giải phương trình chứa ẩn dưới dấu cănLuận văn: Phương pháp giải phương trình chứa ẩn dưới dấu căn
Luận văn: Phương pháp giải phương trình chứa ẩn dưới dấu căn
 
Luận văn: Giải một số phương trình tích phân kỳ dị, HAY, 9đ
Luận văn: Giải một số phương trình tích phân kỳ dị, HAY, 9đLuận văn: Giải một số phương trình tích phân kỳ dị, HAY, 9đ
Luận văn: Giải một số phương trình tích phân kỳ dị, HAY, 9đ
 
Luận văn: Giải số phương trình vi phân đại số bằng đa bước, 9đ
Luận văn: Giải số phương trình vi phân đại số bằng đa bước, 9đLuận văn: Giải số phương trình vi phân đại số bằng đa bước, 9đ
Luận văn: Giải số phương trình vi phân đại số bằng đa bước, 9đ
 
Luận văn: Phương trình tích phân kỳ dị với dịch chuyển và phản xạ
Luận văn: Phương trình tích phân kỳ dị với dịch chuyển và phản xạLuận văn: Phương trình tích phân kỳ dị với dịch chuyển và phản xạ
Luận văn: Phương trình tích phân kỳ dị với dịch chuyển và phản xạ
 
Đang thức, bat đang thức tích phân trong l p đa thức và phân thức hữu ty và m...
Đang thức, bat đang thức tích phân trong l p đa thức và phân thức hữu ty và m...Đang thức, bat đang thức tích phân trong l p đa thức và phân thức hữu ty và m...
Đang thức, bat đang thức tích phân trong l p đa thức và phân thức hữu ty và m...
 
Luận văn: Bất đẳng thức trong lớp hàm siêu việt, HAY, 9đ
Luận văn: Bất đẳng thức trong lớp hàm siêu việt, HAY, 9đLuận văn: Bất đẳng thức trong lớp hàm siêu việt, HAY, 9đ
Luận văn: Bất đẳng thức trong lớp hàm siêu việt, HAY, 9đ
 
Vận dụng giới hạn dãy số trong giải phương trình hàm.pdf
Vận dụng giới hạn dãy số trong giải phương trình hàm.pdfVận dụng giới hạn dãy số trong giải phương trình hàm.pdf
Vận dụng giới hạn dãy số trong giải phương trình hàm.pdf
 
Đề tài: Tính ổn định của lớp phương trình hàm với cặp biến tự do
Đề tài: Tính ổn định của lớp phương trình hàm với cặp biến tự doĐề tài: Tính ổn định của lớp phương trình hàm với cặp biến tự do
Đề tài: Tính ổn định của lớp phương trình hàm với cặp biến tự do
 
Luận văn: Bổ đề đạo hàm logarit và ứng dụng, HAY, 9đ
Luận văn: Bổ đề đạo hàm logarit và ứng dụng, HAY, 9đLuận văn: Bổ đề đạo hàm logarit và ứng dụng, HAY, 9đ
Luận văn: Bổ đề đạo hàm logarit và ứng dụng, HAY, 9đ
 
Luận văn: Phương trình tích phân ngẫu nhiên, HOT, 9đ
Luận văn: Phương trình tích phân ngẫu nhiên, HOT, 9đLuận văn: Phương trình tích phân ngẫu nhiên, HOT, 9đ
Luận văn: Phương trình tích phân ngẫu nhiên, HOT, 9đ
 
Luận văn: Phương trình tích phân ngẫu nhiên, HAY
Luận văn: Phương trình tích phân ngẫu nhiên, HAYLuận văn: Phương trình tích phân ngẫu nhiên, HAY
Luận văn: Phương trình tích phân ngẫu nhiên, HAY
 
Luận văn: Kết quả về nghiệm của phương trình Cauchy-Riemann
Luận văn: Kết quả về nghiệm của phương trình Cauchy-RiemannLuận văn: Kết quả về nghiệm của phương trình Cauchy-Riemann
Luận văn: Kết quả về nghiệm của phương trình Cauchy-Riemann
 
Đề tài: Một số phương pháp giải bài toán phương trình đạo hàm riêng biên trị
Đề tài: Một số phương pháp giải bài toán phương trình đạo hàm riêng biên trịĐề tài: Một số phương pháp giải bài toán phương trình đạo hàm riêng biên trị
Đề tài: Một số phương pháp giải bài toán phương trình đạo hàm riêng biên trị
 
Math educare] toan a1-giai tich ham mot bien_giai tich ham nhieu bien_phuong ...
Math educare] toan a1-giai tich ham mot bien_giai tich ham nhieu bien_phuong ...Math educare] toan a1-giai tich ham mot bien_giai tich ham nhieu bien_phuong ...
Math educare] toan a1-giai tich ham mot bien_giai tich ham nhieu bien_phuong ...
 
Luận văn: Giải bài toán Dirichlet đối với phương trình Elliptic, 9đ
Luận văn: Giải bài toán Dirichlet đối với phương trình Elliptic, 9đLuận văn: Giải bài toán Dirichlet đối với phương trình Elliptic, 9đ
Luận văn: Giải bài toán Dirichlet đối với phương trình Elliptic, 9đ
 

More from Dịch vụ viết bài trọn gói ZALO 0917193864

Danh sách 200 đề tài luận văn thạc sĩ tài chính ngân hàng, từ sinh viên giỏi
Danh sách 200 đề tài luận văn thạc sĩ tài chính ngân hàng, từ sinh viên giỏiDanh sách 200 đề tài luận văn thạc sĩ tài chính ngân hàng, từ sinh viên giỏi
Danh sách 200 đề tài luận văn thạc sĩ tài chính ngân hàng, từ sinh viên giỏiDịch vụ viết bài trọn gói ZALO 0917193864
 

More from Dịch vụ viết bài trọn gói ZALO 0917193864 (20)

200 de tai khoa luạn tot nghiep nganh tam ly hoc
200 de tai khoa luạn tot nghiep nganh tam ly hoc200 de tai khoa luạn tot nghiep nganh tam ly hoc
200 de tai khoa luạn tot nghiep nganh tam ly hoc
 
Danh sách 200 đề tài luận văn tốt nghiệp ngành khách sạn,10 điểm
Danh sách 200 đề tài luận văn tốt nghiệp ngành khách sạn,10 điểmDanh sách 200 đề tài luận văn tốt nghiệp ngành khách sạn,10 điểm
Danh sách 200 đề tài luận văn tốt nghiệp ngành khách sạn,10 điểm
 
Danh sách 200 đề tài luận văn thạc sĩ ngân hàng, hay nhất
Danh sách 200 đề tài luận văn thạc sĩ ngân hàng, hay nhấtDanh sách 200 đề tài luận văn thạc sĩ ngân hàng, hay nhất
Danh sách 200 đề tài luận văn thạc sĩ ngân hàng, hay nhất
 
Danh sách 200 đề tài luận văn thạc sĩ ngữ văn, hay nhất
Danh sách 200 đề tài luận văn thạc sĩ ngữ văn, hay nhấtDanh sách 200 đề tài luận văn thạc sĩ ngữ văn, hay nhất
Danh sách 200 đề tài luận văn thạc sĩ ngữ văn, hay nhất
 
Danh sách 200 đề tài luận văn thạc sĩ ô tô, 10 điểm
Danh sách 200 đề tài luận văn thạc sĩ ô tô, 10 điểmDanh sách 200 đề tài luận văn thạc sĩ ô tô, 10 điểm
Danh sách 200 đề tài luận văn thạc sĩ ô tô, 10 điểm
 
Danh sách 200 đề tài luận văn thạc sĩ quản lý giáo dục mầm non, mới nhất
Danh sách 200 đề tài luận văn thạc sĩ quản lý giáo dục mầm non, mới nhấtDanh sách 200 đề tài luận văn thạc sĩ quản lý giáo dục mầm non, mới nhất
Danh sách 200 đề tài luận văn thạc sĩ quản lý giáo dục mầm non, mới nhất
 
Danh sách 200 đề tài luận văn thạc sĩ quản trị rủi ro, hay nhất
Danh sách 200 đề tài luận văn thạc sĩ quản trị rủi ro, hay nhấtDanh sách 200 đề tài luận văn thạc sĩ quản trị rủi ro, hay nhất
Danh sách 200 đề tài luận văn thạc sĩ quản trị rủi ro, hay nhất
 
Danh sách 200 đề tài luận văn thạc sĩ tài chính ngân hàng, từ sinh viên giỏi
Danh sách 200 đề tài luận văn thạc sĩ tài chính ngân hàng, từ sinh viên giỏiDanh sách 200 đề tài luận văn thạc sĩ tài chính ngân hàng, từ sinh viên giỏi
Danh sách 200 đề tài luận văn thạc sĩ tài chính ngân hàng, từ sinh viên giỏi
 
Danh sách 200 đề tài luận văn thạc sĩ tiêm chủng mở rộng, 10 điểm
Danh sách 200 đề tài luận văn thạc sĩ tiêm chủng mở rộng, 10 điểmDanh sách 200 đề tài luận văn thạc sĩ tiêm chủng mở rộng, 10 điểm
Danh sách 200 đề tài luận văn thạc sĩ tiêm chủng mở rộng, 10 điểm
 
danh sach 200 de tai luan van thac si ve rac nhua
danh sach 200 de tai luan van thac si ve rac nhuadanh sach 200 de tai luan van thac si ve rac nhua
danh sach 200 de tai luan van thac si ve rac nhua
 
Kinh Nghiệm Chọn 200 Đề Tài Tiểu Luận Chuyên Viên Chính Trị Hay Nhất
Kinh Nghiệm Chọn 200 Đề Tài Tiểu Luận Chuyên Viên Chính Trị Hay NhấtKinh Nghiệm Chọn 200 Đề Tài Tiểu Luận Chuyên Viên Chính Trị Hay Nhất
Kinh Nghiệm Chọn 200 Đề Tài Tiểu Luận Chuyên Viên Chính Trị Hay Nhất
 
Kho 200 Đề Tài Bài Luận Văn Tốt Nghiệp Ngành Kế Toán, 9 điểm
Kho 200 Đề Tài Bài Luận Văn Tốt Nghiệp Ngành Kế Toán, 9 điểmKho 200 Đề Tài Bài Luận Văn Tốt Nghiệp Ngành Kế Toán, 9 điểm
Kho 200 Đề Tài Bài Luận Văn Tốt Nghiệp Ngành Kế Toán, 9 điểm
 
Kho 200 Đề Tài Luận Văn Ngành Thủy Sản, từ các trường đại học
Kho 200 Đề Tài Luận Văn Ngành Thủy Sản, từ các trường đại họcKho 200 Đề Tài Luận Văn Ngành Thủy Sản, từ các trường đại học
Kho 200 Đề Tài Luận Văn Ngành Thủy Sản, từ các trường đại học
 
Kho 200 đề tài luận văn ngành thương mại điện tử
Kho 200 đề tài luận văn ngành thương mại điện tửKho 200 đề tài luận văn ngành thương mại điện tử
Kho 200 đề tài luận văn ngành thương mại điện tử
 
Kho 200 đề tài luận văn tốt nghiệp ngành điện tử viễn thông, 9 điểm
Kho 200 đề tài luận văn tốt nghiệp ngành điện tử viễn thông, 9 điểmKho 200 đề tài luận văn tốt nghiệp ngành điện tử viễn thông, 9 điểm
Kho 200 đề tài luận văn tốt nghiệp ngành điện tử viễn thông, 9 điểm
 
Kho 200 Đề Tài Luận Văn Tốt Nghiệp Ngành Giáo Dục Tiểu Học
Kho 200 Đề Tài Luận Văn Tốt Nghiệp Ngành Giáo Dục Tiểu HọcKho 200 Đề Tài Luận Văn Tốt Nghiệp Ngành Giáo Dục Tiểu Học
Kho 200 Đề Tài Luận Văn Tốt Nghiệp Ngành Giáo Dục Tiểu Học
 
Kho 200 đề tài luận văn tốt nghiệp ngành luật, hay nhất
Kho 200 đề tài luận văn tốt nghiệp ngành luật, hay nhấtKho 200 đề tài luận văn tốt nghiệp ngành luật, hay nhất
Kho 200 đề tài luận văn tốt nghiệp ngành luật, hay nhất
 
Kho 200 đề tài luận văn tốt nghiệp ngành quản trị văn phòng, 9 điểm
Kho 200 đề tài luận văn tốt nghiệp ngành quản trị văn phòng, 9 điểmKho 200 đề tài luận văn tốt nghiệp ngành quản trị văn phòng, 9 điểm
Kho 200 đề tài luận văn tốt nghiệp ngành quản trị văn phòng, 9 điểm
 
Kho 200 Đề Tài Luận Văn Tốt Nghiệp Ngành Sư Phạm Tin Học
Kho 200 Đề Tài Luận Văn Tốt Nghiệp Ngành Sư Phạm Tin HọcKho 200 Đề Tài Luận Văn Tốt Nghiệp Ngành Sư Phạm Tin Học
Kho 200 Đề Tài Luận Văn Tốt Nghiệp Ngành Sư Phạm Tin Học
 
Kho 200 Đề Tài Luận Văn Tốt Nghiệp Ngành Xuất Nhập Khẩu
Kho 200 Đề Tài Luận Văn Tốt Nghiệp Ngành Xuất Nhập KhẩuKho 200 Đề Tài Luận Văn Tốt Nghiệp Ngành Xuất Nhập Khẩu
Kho 200 Đề Tài Luận Văn Tốt Nghiệp Ngành Xuất Nhập Khẩu
 

Recently uploaded

Sơ đồ tư duy môn sinh học bậc THPT.pdf
Sơ đồ tư duy môn sinh học bậc THPT.pdfSơ đồ tư duy môn sinh học bậc THPT.pdf
Sơ đồ tư duy môn sinh học bậc THPT.pdftohoanggiabao81
 
TỔNG HỢP 30 ĐỀ THI CHỌN HSG CÁC TRƯỜNG THPT CHUYÊN VÙNG DUYÊN HẢI & ĐỒNG BẰNG...
TỔNG HỢP 30 ĐỀ THI CHỌN HSG CÁC TRƯỜNG THPT CHUYÊN VÙNG DUYÊN HẢI & ĐỒNG BẰNG...TỔNG HỢP 30 ĐỀ THI CHỌN HSG CÁC TRƯỜNG THPT CHUYÊN VÙNG DUYÊN HẢI & ĐỒNG BẰNG...
TỔNG HỢP 30 ĐỀ THI CHỌN HSG CÁC TRƯỜNG THPT CHUYÊN VÙNG DUYÊN HẢI & ĐỒNG BẰNG...Nguyen Thanh Tu Collection
 
Kiểm tra chạy trạm lí thuyết giữa kì giải phẫu sinh lí
Kiểm tra chạy trạm lí thuyết giữa kì giải phẫu sinh líKiểm tra chạy trạm lí thuyết giữa kì giải phẫu sinh lí
Kiểm tra chạy trạm lí thuyết giữa kì giải phẫu sinh líDr K-OGN
 
ĐỀ THAM KHẢO THEO HƯỚNG MINH HỌA 2025 KIỂM TRA CUỐI HỌC KÌ 2 NĂM HỌC 2023-202...
ĐỀ THAM KHẢO THEO HƯỚNG MINH HỌA 2025 KIỂM TRA CUỐI HỌC KÌ 2 NĂM HỌC 2023-202...ĐỀ THAM KHẢO THEO HƯỚNG MINH HỌA 2025 KIỂM TRA CUỐI HỌC KÌ 2 NĂM HỌC 2023-202...
ĐỀ THAM KHẢO THEO HƯỚNG MINH HỌA 2025 KIỂM TRA CUỐI HỌC KÌ 2 NĂM HỌC 2023-202...Nguyen Thanh Tu Collection
 
[GIẢI PHẪU BỆNH] Tổn thương cơ bản của tb bào mô
[GIẢI PHẪU BỆNH] Tổn thương cơ bản của tb bào mô[GIẢI PHẪU BỆNH] Tổn thương cơ bản của tb bào mô
[GIẢI PHẪU BỆNH] Tổn thương cơ bản của tb bào môBryan Williams
 
BỘ ĐỀ KIỂM TRA CUỐI KÌ 2 VẬT LÝ 11 - KẾT NỐI TRI THỨC - THEO CẤU TRÚC ĐỀ MIN...
BỘ ĐỀ KIỂM TRA CUỐI KÌ 2 VẬT LÝ 11 - KẾT NỐI TRI THỨC - THEO CẤU TRÚC ĐỀ MIN...BỘ ĐỀ KIỂM TRA CUỐI KÌ 2 VẬT LÝ 11 - KẾT NỐI TRI THỨC - THEO CẤU TRÚC ĐỀ MIN...
BỘ ĐỀ KIỂM TRA CUỐI KÌ 2 VẬT LÝ 11 - KẾT NỐI TRI THỨC - THEO CẤU TRÚC ĐỀ MIN...Nguyen Thanh Tu Collection
 
SÁNG KIẾN “THIẾT KẾ VÀ SỬ DỤNG INFOGRAPHIC TRONG DẠY HỌC ĐỊA LÍ 11 (BỘ SÁCH K...
SÁNG KIẾN “THIẾT KẾ VÀ SỬ DỤNG INFOGRAPHIC TRONG DẠY HỌC ĐỊA LÍ 11 (BỘ SÁCH K...SÁNG KIẾN “THIẾT KẾ VÀ SỬ DỤNG INFOGRAPHIC TRONG DẠY HỌC ĐỊA LÍ 11 (BỘ SÁCH K...
SÁNG KIẾN “THIẾT KẾ VÀ SỬ DỤNG INFOGRAPHIC TRONG DẠY HỌC ĐỊA LÍ 11 (BỘ SÁCH K...Nguyen Thanh Tu Collection
 
cuộc cải cách của Lê Thánh Tông - Sử 11
cuộc cải cách của Lê Thánh Tông -  Sử 11cuộc cải cách của Lê Thánh Tông -  Sử 11
cuộc cải cách của Lê Thánh Tông - Sử 11zedgaming208
 
200 câu hỏi trắc nghiệm ôn tập PLDC.pdf
200 câu hỏi trắc nghiệm ôn tập  PLDC.pdf200 câu hỏi trắc nghiệm ôn tập  PLDC.pdf
200 câu hỏi trắc nghiệm ôn tập PLDC.pdfdong92356
 
10 ĐỀ KIỂM TRA + 6 ĐỀ ÔN TẬP CUỐI KÌ 2 VẬT LÝ 11 - KẾT NỐI TRI THỨC - THEO C...
10 ĐỀ KIỂM TRA + 6 ĐỀ ÔN TẬP CUỐI KÌ 2 VẬT LÝ 11 - KẾT NỐI TRI THỨC - THEO C...10 ĐỀ KIỂM TRA + 6 ĐỀ ÔN TẬP CUỐI KÌ 2 VẬT LÝ 11 - KẾT NỐI TRI THỨC - THEO C...
10 ĐỀ KIỂM TRA + 6 ĐỀ ÔN TẬP CUỐI KÌ 2 VẬT LÝ 11 - KẾT NỐI TRI THỨC - THEO C...Nguyen Thanh Tu Collection
 
Hệ phương trình tuyến tính và các ứng dụng trong kinh tế
Hệ phương trình tuyến tính và các ứng dụng trong kinh tếHệ phương trình tuyến tính và các ứng dụng trong kinh tế
Hệ phương trình tuyến tính và các ứng dụng trong kinh tếngTonH1
 
Slide Webinar Hướng dẫn sử dụng ChatGPT cho người mới bắt đầ...
Slide Webinar Hướng dẫn sử dụng ChatGPT cho người mới bắt đầ...Slide Webinar Hướng dẫn sử dụng ChatGPT cho người mới bắt đầ...
Slide Webinar Hướng dẫn sử dụng ChatGPT cho người mới bắt đầ...Học viện Kstudy
 
CHƯƠNG VII LUẬT DÂN SỰ (2) Pháp luật đại cương.pptx
CHƯƠNG VII LUẬT DÂN SỰ (2) Pháp luật đại cương.pptxCHƯƠNG VII LUẬT DÂN SỰ (2) Pháp luật đại cương.pptx
CHƯƠNG VII LUẬT DÂN SỰ (2) Pháp luật đại cương.pptx22146042
 
BỘ ĐỀ PHÁT TRIỂN THEO CẤU TRÚC ĐỀ MINH HỌA BGD NGÀY 22-3-2024 KỲ THI TỐT NGHI...
BỘ ĐỀ PHÁT TRIỂN THEO CẤU TRÚC ĐỀ MINH HỌA BGD NGÀY 22-3-2024 KỲ THI TỐT NGHI...BỘ ĐỀ PHÁT TRIỂN THEO CẤU TRÚC ĐỀ MINH HỌA BGD NGÀY 22-3-2024 KỲ THI TỐT NGHI...
BỘ ĐỀ PHÁT TRIỂN THEO CẤU TRÚC ĐỀ MINH HỌA BGD NGÀY 22-3-2024 KỲ THI TỐT NGHI...Nguyen Thanh Tu Collection
 
Nhóm 10-Xác suất và thống kê toán-đại học thương mại
Nhóm 10-Xác suất và thống kê toán-đại học thương mạiNhóm 10-Xác suất và thống kê toán-đại học thương mại
Nhóm 10-Xác suất và thống kê toán-đại học thương mạiTruongThiDiemQuynhQP
 
ĐỀ THAM KHẢO THEO HƯỚNG MINH HỌA 2025 KIỂM TRA GIỮA HỌC KÌ + CUỐI HỌC KÌ 2 NĂ...
ĐỀ THAM KHẢO THEO HƯỚNG MINH HỌA 2025 KIỂM TRA GIỮA HỌC KÌ + CUỐI HỌC KÌ 2 NĂ...ĐỀ THAM KHẢO THEO HƯỚNG MINH HỌA 2025 KIỂM TRA GIỮA HỌC KÌ + CUỐI HỌC KÌ 2 NĂ...
ĐỀ THAM KHẢO THEO HƯỚNG MINH HỌA 2025 KIỂM TRA GIỮA HỌC KÌ + CUỐI HỌC KÌ 2 NĂ...Nguyen Thanh Tu Collection
 
50 ĐỀ ĐỀ XUẤT THI VÀO 10 THPT SỞ GIÁO DỤC THANH HÓA MÔN TIẾNG ANH 9 CÓ TỰ LUẬ...
50 ĐỀ ĐỀ XUẤT THI VÀO 10 THPT SỞ GIÁO DỤC THANH HÓA MÔN TIẾNG ANH 9 CÓ TỰ LUẬ...50 ĐỀ ĐỀ XUẤT THI VÀO 10 THPT SỞ GIÁO DỤC THANH HÓA MÔN TIẾNG ANH 9 CÓ TỰ LUẬ...
50 ĐỀ ĐỀ XUẤT THI VÀO 10 THPT SỞ GIÁO DỤC THANH HÓA MÔN TIẾNG ANH 9 CÓ TỰ LUẬ...Nguyen Thanh Tu Collection
 
bài 5.1.docx Sinh học di truyền đại cương năm nhất của học sinh y đa khoa
bài 5.1.docx Sinh học di truyền đại cương năm nhất của học sinh y đa khoabài 5.1.docx Sinh học di truyền đại cương năm nhất của học sinh y đa khoa
bài 5.1.docx Sinh học di truyền đại cương năm nhất của học sinh y đa khoa2353020138
 
Ma trận - định thức và các ứng dụng trong kinh tế
Ma trận - định thức và các ứng dụng trong kinh tếMa trận - định thức và các ứng dụng trong kinh tế
Ma trận - định thức và các ứng dụng trong kinh tếngTonH1
 
30 ĐỀ PHÁT TRIỂN THEO CẤU TRÚC ĐỀ MINH HỌA BGD NGÀY 22-3-2024 KỲ THI TỐT NGHI...
30 ĐỀ PHÁT TRIỂN THEO CẤU TRÚC ĐỀ MINH HỌA BGD NGÀY 22-3-2024 KỲ THI TỐT NGHI...30 ĐỀ PHÁT TRIỂN THEO CẤU TRÚC ĐỀ MINH HỌA BGD NGÀY 22-3-2024 KỲ THI TỐT NGHI...
30 ĐỀ PHÁT TRIỂN THEO CẤU TRÚC ĐỀ MINH HỌA BGD NGÀY 22-3-2024 KỲ THI TỐT NGHI...Nguyen Thanh Tu Collection
 

Recently uploaded (20)

Sơ đồ tư duy môn sinh học bậc THPT.pdf
Sơ đồ tư duy môn sinh học bậc THPT.pdfSơ đồ tư duy môn sinh học bậc THPT.pdf
Sơ đồ tư duy môn sinh học bậc THPT.pdf
 
TỔNG HỢP 30 ĐỀ THI CHỌN HSG CÁC TRƯỜNG THPT CHUYÊN VÙNG DUYÊN HẢI & ĐỒNG BẰNG...
TỔNG HỢP 30 ĐỀ THI CHỌN HSG CÁC TRƯỜNG THPT CHUYÊN VÙNG DUYÊN HẢI & ĐỒNG BẰNG...TỔNG HỢP 30 ĐỀ THI CHỌN HSG CÁC TRƯỜNG THPT CHUYÊN VÙNG DUYÊN HẢI & ĐỒNG BẰNG...
TỔNG HỢP 30 ĐỀ THI CHỌN HSG CÁC TRƯỜNG THPT CHUYÊN VÙNG DUYÊN HẢI & ĐỒNG BẰNG...
 
Kiểm tra chạy trạm lí thuyết giữa kì giải phẫu sinh lí
Kiểm tra chạy trạm lí thuyết giữa kì giải phẫu sinh líKiểm tra chạy trạm lí thuyết giữa kì giải phẫu sinh lí
Kiểm tra chạy trạm lí thuyết giữa kì giải phẫu sinh lí
 
ĐỀ THAM KHẢO THEO HƯỚNG MINH HỌA 2025 KIỂM TRA CUỐI HỌC KÌ 2 NĂM HỌC 2023-202...
ĐỀ THAM KHẢO THEO HƯỚNG MINH HỌA 2025 KIỂM TRA CUỐI HỌC KÌ 2 NĂM HỌC 2023-202...ĐỀ THAM KHẢO THEO HƯỚNG MINH HỌA 2025 KIỂM TRA CUỐI HỌC KÌ 2 NĂM HỌC 2023-202...
ĐỀ THAM KHẢO THEO HƯỚNG MINH HỌA 2025 KIỂM TRA CUỐI HỌC KÌ 2 NĂM HỌC 2023-202...
 
[GIẢI PHẪU BỆNH] Tổn thương cơ bản của tb bào mô
[GIẢI PHẪU BỆNH] Tổn thương cơ bản của tb bào mô[GIẢI PHẪU BỆNH] Tổn thương cơ bản của tb bào mô
[GIẢI PHẪU BỆNH] Tổn thương cơ bản của tb bào mô
 
BỘ ĐỀ KIỂM TRA CUỐI KÌ 2 VẬT LÝ 11 - KẾT NỐI TRI THỨC - THEO CẤU TRÚC ĐỀ MIN...
BỘ ĐỀ KIỂM TRA CUỐI KÌ 2 VẬT LÝ 11 - KẾT NỐI TRI THỨC - THEO CẤU TRÚC ĐỀ MIN...BỘ ĐỀ KIỂM TRA CUỐI KÌ 2 VẬT LÝ 11 - KẾT NỐI TRI THỨC - THEO CẤU TRÚC ĐỀ MIN...
BỘ ĐỀ KIỂM TRA CUỐI KÌ 2 VẬT LÝ 11 - KẾT NỐI TRI THỨC - THEO CẤU TRÚC ĐỀ MIN...
 
SÁNG KIẾN “THIẾT KẾ VÀ SỬ DỤNG INFOGRAPHIC TRONG DẠY HỌC ĐỊA LÍ 11 (BỘ SÁCH K...
SÁNG KIẾN “THIẾT KẾ VÀ SỬ DỤNG INFOGRAPHIC TRONG DẠY HỌC ĐỊA LÍ 11 (BỘ SÁCH K...SÁNG KIẾN “THIẾT KẾ VÀ SỬ DỤNG INFOGRAPHIC TRONG DẠY HỌC ĐỊA LÍ 11 (BỘ SÁCH K...
SÁNG KIẾN “THIẾT KẾ VÀ SỬ DỤNG INFOGRAPHIC TRONG DẠY HỌC ĐỊA LÍ 11 (BỘ SÁCH K...
 
cuộc cải cách của Lê Thánh Tông - Sử 11
cuộc cải cách của Lê Thánh Tông -  Sử 11cuộc cải cách của Lê Thánh Tông -  Sử 11
cuộc cải cách của Lê Thánh Tông - Sử 11
 
200 câu hỏi trắc nghiệm ôn tập PLDC.pdf
200 câu hỏi trắc nghiệm ôn tập  PLDC.pdf200 câu hỏi trắc nghiệm ôn tập  PLDC.pdf
200 câu hỏi trắc nghiệm ôn tập PLDC.pdf
 
10 ĐỀ KIỂM TRA + 6 ĐỀ ÔN TẬP CUỐI KÌ 2 VẬT LÝ 11 - KẾT NỐI TRI THỨC - THEO C...
10 ĐỀ KIỂM TRA + 6 ĐỀ ÔN TẬP CUỐI KÌ 2 VẬT LÝ 11 - KẾT NỐI TRI THỨC - THEO C...10 ĐỀ KIỂM TRA + 6 ĐỀ ÔN TẬP CUỐI KÌ 2 VẬT LÝ 11 - KẾT NỐI TRI THỨC - THEO C...
10 ĐỀ KIỂM TRA + 6 ĐỀ ÔN TẬP CUỐI KÌ 2 VẬT LÝ 11 - KẾT NỐI TRI THỨC - THEO C...
 
Hệ phương trình tuyến tính và các ứng dụng trong kinh tế
Hệ phương trình tuyến tính và các ứng dụng trong kinh tếHệ phương trình tuyến tính và các ứng dụng trong kinh tế
Hệ phương trình tuyến tính và các ứng dụng trong kinh tế
 
Slide Webinar Hướng dẫn sử dụng ChatGPT cho người mới bắt đầ...
Slide Webinar Hướng dẫn sử dụng ChatGPT cho người mới bắt đầ...Slide Webinar Hướng dẫn sử dụng ChatGPT cho người mới bắt đầ...
Slide Webinar Hướng dẫn sử dụng ChatGPT cho người mới bắt đầ...
 
CHƯƠNG VII LUẬT DÂN SỰ (2) Pháp luật đại cương.pptx
CHƯƠNG VII LUẬT DÂN SỰ (2) Pháp luật đại cương.pptxCHƯƠNG VII LUẬT DÂN SỰ (2) Pháp luật đại cương.pptx
CHƯƠNG VII LUẬT DÂN SỰ (2) Pháp luật đại cương.pptx
 
BỘ ĐỀ PHÁT TRIỂN THEO CẤU TRÚC ĐỀ MINH HỌA BGD NGÀY 22-3-2024 KỲ THI TỐT NGHI...
BỘ ĐỀ PHÁT TRIỂN THEO CẤU TRÚC ĐỀ MINH HỌA BGD NGÀY 22-3-2024 KỲ THI TỐT NGHI...BỘ ĐỀ PHÁT TRIỂN THEO CẤU TRÚC ĐỀ MINH HỌA BGD NGÀY 22-3-2024 KỲ THI TỐT NGHI...
BỘ ĐỀ PHÁT TRIỂN THEO CẤU TRÚC ĐỀ MINH HỌA BGD NGÀY 22-3-2024 KỲ THI TỐT NGHI...
 
Nhóm 10-Xác suất và thống kê toán-đại học thương mại
Nhóm 10-Xác suất và thống kê toán-đại học thương mạiNhóm 10-Xác suất và thống kê toán-đại học thương mại
Nhóm 10-Xác suất và thống kê toán-đại học thương mại
 
ĐỀ THAM KHẢO THEO HƯỚNG MINH HỌA 2025 KIỂM TRA GIỮA HỌC KÌ + CUỐI HỌC KÌ 2 NĂ...
ĐỀ THAM KHẢO THEO HƯỚNG MINH HỌA 2025 KIỂM TRA GIỮA HỌC KÌ + CUỐI HỌC KÌ 2 NĂ...ĐỀ THAM KHẢO THEO HƯỚNG MINH HỌA 2025 KIỂM TRA GIỮA HỌC KÌ + CUỐI HỌC KÌ 2 NĂ...
ĐỀ THAM KHẢO THEO HƯỚNG MINH HỌA 2025 KIỂM TRA GIỮA HỌC KÌ + CUỐI HỌC KÌ 2 NĂ...
 
50 ĐỀ ĐỀ XUẤT THI VÀO 10 THPT SỞ GIÁO DỤC THANH HÓA MÔN TIẾNG ANH 9 CÓ TỰ LUẬ...
50 ĐỀ ĐỀ XUẤT THI VÀO 10 THPT SỞ GIÁO DỤC THANH HÓA MÔN TIẾNG ANH 9 CÓ TỰ LUẬ...50 ĐỀ ĐỀ XUẤT THI VÀO 10 THPT SỞ GIÁO DỤC THANH HÓA MÔN TIẾNG ANH 9 CÓ TỰ LUẬ...
50 ĐỀ ĐỀ XUẤT THI VÀO 10 THPT SỞ GIÁO DỤC THANH HÓA MÔN TIẾNG ANH 9 CÓ TỰ LUẬ...
 
bài 5.1.docx Sinh học di truyền đại cương năm nhất của học sinh y đa khoa
bài 5.1.docx Sinh học di truyền đại cương năm nhất của học sinh y đa khoabài 5.1.docx Sinh học di truyền đại cương năm nhất của học sinh y đa khoa
bài 5.1.docx Sinh học di truyền đại cương năm nhất của học sinh y đa khoa
 
Ma trận - định thức và các ứng dụng trong kinh tế
Ma trận - định thức và các ứng dụng trong kinh tếMa trận - định thức và các ứng dụng trong kinh tế
Ma trận - định thức và các ứng dụng trong kinh tế
 
30 ĐỀ PHÁT TRIỂN THEO CẤU TRÚC ĐỀ MINH HỌA BGD NGÀY 22-3-2024 KỲ THI TỐT NGHI...
30 ĐỀ PHÁT TRIỂN THEO CẤU TRÚC ĐỀ MINH HỌA BGD NGÀY 22-3-2024 KỲ THI TỐT NGHI...30 ĐỀ PHÁT TRIỂN THEO CẤU TRÚC ĐỀ MINH HỌA BGD NGÀY 22-3-2024 KỲ THI TỐT NGHI...
30 ĐỀ PHÁT TRIỂN THEO CẤU TRÚC ĐỀ MINH HỌA BGD NGÀY 22-3-2024 KỲ THI TỐT NGHI...
 

Luận văn: Phép biến đổi phân tuyến tính, HAY, 9đ

  • 1. ĐẠI HỌC QUỐC GIA HÀ NỘI TRƯỜNG ĐẠI HỌC KHOA HỌC TỰ NHIÊN Đào Thị Anh Phương PHÉP BIẾN ĐỔI PHÂN TUYẾN TÍNH VÀ ÁP DỤNG GIẢI MỘT SỐ BÀI TOÁN PHỔ THÔNG LUẬN VĂN THẠC SỸ KHOA HỌC Hà Nội - 2011
  • 2. ĐẠI HỌC QUỐC GIA HÀ NỘI TRƯỜNG ĐẠI HỌC KHOA HỌC TỰ NHIÊN Đào Thị Anh Phương PHÉP BIẾN ĐỔI PHÂN TUYẾN TÍNH VÀ ÁP DỤNG GIẢI MỘT SỐ BÀI TOÁN PHỔ THÔNG Chuyên ngành : Phương pháp toán sơ cấp Mã số : 60 46 40 LUẬN VĂN THẠC SỸ KHOA HỌC Người hướng dẫn khoa học PGS.TS. NGUYỄN MINH TUẤN Hà Nội - 2011
  • 3. Mục lục LỜI NÓI ĐẦU . . . . . . . . . . . . . . . . . . . . . . . . . . . . . . . . . . . . . . . . . . . . 2 Chương 1. Một lớp phương trình hàm sinh bởi hàm phân tuyến tính.. . . 4 1.1. Kiến thức chuẩn bị . . . . . . . . . . . . . . . . . . . . . . . . . . . . . . . . . . . . . . . . . . . . . 4 1.1.1. Hàm số. . . . . . . . . . . . . . . . . . . . . . . . . . . . . . . . . . . . . . . . . . . . . . . . . . . . . . . . . . . 4 1.1.2. Hàm số đơn điệu . . . . . . . . . . . . . . . . . . . . . . . . . . . . . . . . . . . . . . . . . . . . . . . . . . 5 1.1.3. Hàm phân tuyến tính . . . . . . . . . . . . . . . . . . . . . . . . . . . . . . . . . . . . . . . . . . . . . . 6 1.2. Phép biến đổi phân tuyến tính trong phương trình hàm . . . . . . . . . . 6 1.2.1. Hàm số xác định bởi các phép biến đổi phân tuyến tính. . . . . . . . . . . . . . . . 6 1.2.2. Một số bài toán khác về hàm phân tuyến tính. . . . . . . . . . . . . . . . . . . . . . . . 23 1.2.3. Bài tập tham khảo . . . . . . . . . . . . . . . . . . . . . . . . . . . . . . . . . . . . . . . . . . . . . . . . 34 Chương 2. Một số bài toán về dãy số. . . . . . . . . . . . . . . . . . . . . . . . . . . . . . . . . 36 2.1. Phương trình và hệ phương trình sai phân. . . . . . . . . . . . . . . . . . . . . . 36 2.1.1. Phương trình sai phân tuyến tính với hệ số hằng.. . . . . . . . . . . . . . . . . . . . . 36 2.1.2. Hệ phương trình sai phân tuyến tính với hệ số hằng . . . . . . . . . . . . . . . . . 40 2.2. Phương trình sai phân dạng phân tuyến tính với hệ số hằng . . . . . 41 2.3. Giới hạn của một số dãy truy hồi dạng phân tuyến tính. . . . . . . . . . 50 2.4. Bài tập tham khảo. . . . . . . . . . . . . . . . . . . . . . . . . . . . . . . . . . . . . . . . . . . . . 56 KẾT LUẬN . . . . . . . . . . . . . . . . . . . . . . . . . . . . . . . . . . . . . . . . . . . . . 58 Tài liệu tham khảo . . . . . . . . . . . . . . . . . . . . . . . . . . . . . . . . . . . . . . . 59 1
  • 4. LỜI NÓI ĐẦU Phép biến đổi phân tuyến tính có nhiều ứng dụng trong môn Toán ở bậc phổ thông. Đặc biệt là ở trường chuyên, lớp chọn và trong các kỳ thi học sinh giỏi Toán trong nước, trong các kỳ thi Olympic các nước trên thế giới thông qua các bài toán về phương trình hàm, các bài toán về dãy số. Để đáp ứng nhu cầu học tập và giảng dạy môn toán ở bậc phổ thông, luận văn Phép biến đổi phân tuyến tính và áp dụng giải một số bài toán phổ thông với mục tiêu tổng hợp và chọn lọc các kiến thức về phép biến đổi phân tuyến tính để giải quyết các bài toán về phương trình hàm và các bài toán về dãy số. Luận văn được chia thành hai chương. Chương 1: Một lớp phương trình hàm sinh bởi hàm phân tuyến tính. Chương này nêu lên một số kiến thức cơ bản về hàm số nói chung và hàm phân tuyến tính nói riêng. Phần trọng tâm của chương là giải quyết các bài toán về phép biến đổi phân tuyến tính trong phương trình hàm. Chương 2: Một số bài toán về dãy số. Chương này nêu lên các kiến thức cơ bản về phương trình và hệ phương trình sai phân tuyến tính với hệ số hằng. Phần trọng tâm của chương là giải quyết các bài toán về hai mảng kiến thức của dãy số: • Phương trình sai phân dạng phân tuyến tính với hệ số hằng. • Giới hạn của một số dãy sai phân dạng phân tuyến tính. Để hoàn thành được luận văn này, trước nhất tác giả xin được gửi lời cảm ơn sâu sắc tới PGS.TS Nguyễn Minh Tuấn đã dành thời gian hướng dẫn, đánh giá, chỉ bảo tận tình giúp đỡ trong quá trình xây dựng đề tài cũng như hoàn thiện luận văn. Tiếp theo, tác giả cũng xin gửi lời cảm ơn chân thành các thầy cô đã đọc, kiểm tra đánh giá và cho những ý kiến quý báu để luận văn được đầy đủ hơn, phong phú hơn. Qua đây, tác giả cũng xin được gửi lời cảm ơn tới Ban giám hiệu, phòng sau Đại học, khoa Toán-Cơ-Tin học trường Đại học Khoa học Tự nhiên và các bạn đồng 2
  • 5. nghiệp đã tạo điều kiện thuận lợi trong suốt quá trình học tập tại trường. Tuy có nhiều cố gắng nhưng do thời gian và khả năng có hạn nên các vấn đề trong luận văn chưa được trình bày sâu sắc và không thể tránh khỏi có những sai sót trong cách trình bày. Rất mong được sự đóng góp ý kiến thêm nữa của thầy cô và các bạn. Tác giả xin chân thành cảm ơn ! Hà Nội, Tháng 02 năm 2011 Đào Thị Anh Phương 3
  • 6. Chương 1 Một lớp phương trình hàm sinh bởi hàm phân tuyến tính. 1.1. Kiến thức chuẩn bị 1.1.1. Hàm số Định nghĩa Cho tập hợp D ⊂ R. Một ánh xạ f : D → R được gọi là một hàm số từ tập D đến tập R và ký hiệu là f : D → R hoặc y = f(x). •D được gọi là tập xác định của hàm số. • f(x0) là giá trị của hàm số tại điểm x0 ∈ D. • Tập hợp T = { f(x)|x ∈ D} được gọi là tập giá trị của hàm số f. Chú ý 1)t ∈ T khi và chỉ khi phương trình f(x) = t có nghiệm x ∈ D. 2)t ∈ T, suy ra t có thể viết dưới dạng t = f(x) với x ∈ D. • Điểm x0 ∈ Dđược gọi là điểm bất động của hàm f nếu như f(x0) = x0. Ví dụ: Ánh xạ x → f(x) = ax+b cx+d c = 0 và ad − bc = 0 xác định một hàm ( gọi là phân tuyến tính trên tập D = R{ −d c }). 4
  • 7. 1.1.2. Hàm số đơn điệu Định nghĩa 1) Hàm số f(x) được gọi là tăng trên khoảng (a;b) nếu với x1 ,x2 ∈ (a;b) mà x1 ≤ x2 thì f(x1) ≤ f(x2). 2) Hàm số f(x) được gọi là giảm trên khoảng (a;b) nếu như với x1 ,x2 ∈ (a;b) mà x1 ≤ x2 thì f(x1) ≥ f(x2). Hàm số tăng hoặc giảm trên một khoảng gọi là hàm số đơn điệu trên khoảng đó. 3) Hàm số f(x) được gọi là tăng thực sự trên khoảng (a;b) nếu như với x1 ,x2 ∈ (a;b) mà x1 < x2 thì f(x1) < f(x2). 4) Hàm số f(x) được gọi là giảm thực sự trên khoảng (a;b) nếu như với x1 ,x2 ∈ (a;b) mà x1 < x2 thì f(x1) > f(x2). 5) Hàm số tăng thực sự hoặc giảm thực sự trên khoảng (a;b) gọi là hàm số đơn điệu thực sự trên khoảng đó. Tính chất: 1) Mọi hàm đơn điệu thật sự trên một khoảng đều là đơn ánh trên khoảng đó. 2) Nếu f : D → R; g : D → R là hai hàm tăng thì f +g tăng. 3) Nếu f : D → R; g : D → R là hai hàm tăng và không âm thì f(x) · g(x) là hàm tăng. 4) Nếu hàm f đơn điệu trên khoảng (a,b) thì phương trình f(x) = m có nhiều nhất một nghiệm trên khoảng đó. 5) Nếu f : Df → R và g : Dg → R tăng và Tf ⊂ Dg thì hàm số hợp g◦ f tăng. Chú ý: Từ kết quả trên suy ra: Nếu hàm f tăng thì hàm số hợp f(f(x)) (nếu được xác định) cũng tăng. Nếu hàm f giảm thì hàm số hợp f(f(x)) (nếu được xác định) cũng giảm. 5
  • 8. 1.1.3. Hàm phân tuyến tính Định nghĩa: Hàm phân tuyến tính là ánh xạ x −→ y = f(x) = ax+b cx+d (1) trong đó a,b,c,d ∈ R và ad −bc = 0, c = 0. Điều kiện ad − bc = 0 để loại trường hợp của vế phải của (1) suy biến thành hằng số vì f (x) = a(cx+d)−c(ax+b) (cx+d)2 = ad −bc (cx+d)2 = 0, ∀x ∈ D. Điều kiện c = 0 để loại trường hợp của vế phải của (1) suy biến thành (mx+n). Tập xác định: D = R{ −d c }. Tập giá trị: T = R{ a c }. Một vài tích chất của hàm phân tuyến tính Định lý 3.1: (i) Hàm ngược của một hàm phân tuyến tính là hàm phân tuyến tính. (ii) Hợp thành của hai hàm phân tuyến tính là hàm phân tuyến tính. Định lý 3.2: Hàm phân tuyến tính luôn tăng thực sự ( hoặc giảm thực sự ) trên mỗi khoảng xác định của nó. 1.2. Phép biến đổi phân tuyến tính trong phương trình hàm 1.2.1. Hàm số xác định bởi các phép biến đổi phân tuyến tính Cho hàm số w(x) = ax+b cx+d , c = 0 ,ad −bc = 0. Trong chương trình này, ta sẽ nghiên cứu các phương trình hàm dạng: f(w(x)) = pf(x)+q, ∀x ∈ R −d c trong đó a, b, c, d, p, q là các hằng số thực và p = 0 6
  • 9. Bài toán 1.2.1. Tìm f : R −d c → R sao cho f ax+b cx+d = p f(x) + q, ∀x ∈ R −d c , ad − bc = 0 ,c = 0 trong đó phương trình w(x) = x có nghiệm. Giải. a) Trường hợp w(x) = x có hai nghiệm phân biệt x1 ,x2. Với x = x2 ta có f(w(x2)) = p f(x2)+q ⇒ f(x2) = p f(x2)+q =⇒ f(x2)(1− p) = q. +)Nếu p = 1thì f(x2) = q 1− p , +)Nếu p = 1thì   Phương trình vô nghiệm nếu q = 0. Phương trình có nghiệm f(x2) bất kỳ nếu q = 0. Với x = x2, đặt x−x1 x−x2 = t ⇒ x = x2 + x2 −x1 t −1 , ∀t = 1, và ax+b cx+d = x2 + x2 −x1 cx2 +d cx1 +d t −1 ,∀t = 1. Khi đó theo giả thiết thì f x2 + x2 −x1 cx2 +d cx1 +d t −1 = p f x2 + x2 −x1 t −1 +q ∀t = 1. Đặt f x2 + x2 −x1 t −1 = g(t). Suy ra g cx2 +d cx1 +d t = pg(t)+q ,∀t = 1. b) Trường hợp w(x) = x có nghiệm kép ( lúc đó (d −a)2 +4ac = 0) và x0 = a−d 2c . Với x = x0 ta có f(x0) = pf(x0)+q hay f(x0)(1− p) = q. +) Nếu p = 1 thì f(x) = q 1−q . 7
  • 10. +) Nếu p = 1 thì   Phương trình vô nghiệm nếu q = 0 Có nghiệmf(x0) tùy ý nếu q = 0 Với x = x0 đặt: 1 x−x0 = t ⇒ x = x0 + 1 t , ∀t = 0, ax+b cx+d = x0 + 1 t + c cx0 +d . Khi đó giả thiết đã cho trở thành f   x0 + 1 t + c cx0 +d    = p f x0 + 1 t +q với t = 0. Đặt f(x0 + 1 t ) = g(t) ta được g t + c cx0 +d = pg(t)+ q, ∀t = 0. Bài toán 1.2.2. Tìm hàm f : R −d c → R sao cho f ax+b cx+d = pf(x) + q, ∀x ∈ R −d c , ad − bc = 0, c = 0, trong đó k ∈ Z+ sao cho wk(x) = x có nghiệm, ở đó w1(x) = w(x) = ax+b cx+d . Giải. Gọi k ∈ Z+ là số bé nhất sao cho wk(x) = x có nghiệm +) Nếu k = 1 thì chính là bài toán 1.2.1 +) Nếu k = 1 thì: f(w(x)) = p· f(x) + q, ⇒ f(w2(x)) = p· f(w(x)) + q = p2 · f(x)+ pq+q, ··· ⇒ f(wk(x)) = pk · f(x)+(pk−1 + pk−2 +···+ p+1)q. Vì phương trình wk(x) = akx+bk ckx+dk = x, 8
  • 11. (với ak ·dk −bk ·ck = 0) có nghiệm. Nếu ck = 0 hoặc dk = 0 thì bài toán sẽ quay về bài toán 1.2.1 vừa xét. Sau đây, ta minh họa cách giải ứng với các trường hợp thông qua bài toán cụ thể. Ta chỉ cần xét các phương trình hàm sinh bởi hàm phân tuyến tính w(x) = ax+b cx+d , c = 0, ad −bc > 0. Bài toán 1.2.3. Tìm tất cả các hàm số f : R → R sao cho f −1 x+2 = 2 f(x)−3, ∀x = −2. (1.2.1) Giải. Nhận xét rằng, phương trình x = −1 x+2 có nghiệm duy nhất x = −1 Thay x = −1 vào (1.2.1), ta được f(−1) = 3. Xét x = −1. Đặt 1 x+1 = t thì t = 0 ,t = −1 và x = −1+ 1 t , −1 x+2 = −1+ 1 t +1 . Khi đó (1.2.1) có dạng f −1+ 1 t +1 = 2 f −1+ 1 t −3, ∀t ∈ R{−1;0}, (1.2.2) hay g(t +1) = 2g(t), ∀t ∈ R{−1;0}, trong đó g(t) = f(−1+ −1 t )−3, ∀t ∈ R{−1;0}. (1.2.3) Suy ra g(t) = 2th(t), với h(t) là hàm tùy ý sao cho h(t +1) = h(t), ∀t ∈ R{−1;0}. 9
  • 12. Từ (1.2.2) và (1.2.3) ta có Kết luận f(x) =    3, khi x = −1 g 1 x+1 +3, khi x = −1, (1.2.4) trong đó g(t) = 2th(t), với h(t) là hàm tùy ý thỏa mãn h(t +1) = h(t), ∀t ∈ R{−1;0}. Bài toán 1.2.4.[1] Cho q ∈ R và cho hàm số w(x) = ax+b x−1 , a = 1, b = −1 4 (a+1)2 . Tìm tất cả các hàm số f : R{1} → R sao cho f(w(x)) = −f(x)+q, ∀x = 1. (1.2.5) Giải. Theo giả thiết thì phương trình w(x) = x có nghiệm thực duy nhất x = a+1 2 . Thay x = a+1 2 vào (1.2.5) ta được: f a+1 2 = q 2 . Xét x = x0, x0 = a+1 2 . Đặt t = 1 x−x0 , khi đó t = 0, và x = x0 + 1 t , w(x) = x0 + 1 t + 2 a−1 . Khi đó có thể viết (1.2.5) dưới dạng f x0 + 1 t + 2 a−1 = −f(x0 + 1 t )+q, ∀t ∈ R{0 ; 2 1−a }, (1.2.6) 10
  • 13. hay g t + 2 a−1 = −g(t), ∀t ∈ R{0 ; 2 1−a }, trong đó g(t) = f(x0 + 1 t )− q 2 , ∀t ∈ R{0 ; 2 1−a }. (1.2.7) Suy ra g(t) = h t + 2 a−1 −h(t) , với h(t) là hàm tùy ý thỏa mãn: h t + 4 a−1 = h(t), ∀t ∈ R{0 ; 2 1−a }. Từ (1.2.6) và (1.2.7) ta có Kết luận: f(x) =    q 2 , khi x = a+1 2 g    1 x− a+1 2   + q 2 , khi∀x ∈ R{1; a+1 2 }, (1.2.8) trong đó g(t) = h t + 2 a−1 −h(t), ∀t ∈ R{0 ; 2 1−a }, với h(t) là hàm tùy ý thỏa mãn h t + 4 a−1 = h(t), ∀t ∈ R{0 ; 2 1−a }. Từ hai bài toán 1.2.3 và 1.2.4 ta xây dựng bài toán khái quát hóa trong trường hợp phương trình hàm sinh bởi hàm phân tuyến tính w(x) = ax+b cx+d , c = 0, ad −bc = 0 thỏa mãn w (x) = x có nghiệm kép x = x0. 11
  • 14. Bài toán 1.2.5.[1] Cho hàm số: w(x) = ax+b cx+d , ad −bc = 0, c = 0, sao cho phương trình w(x) = x có nghiệm kép x = x0. Tìm tất cả các hàm số f : R{ −d c } → R sao cho f(w(x)) = −2f(x)+3, ∀x = −d c . (1.2.9) Giải. +) Theo giả thiết thì phương trình w(x) = x có nghiệm duy nhất x = x0. Thay x = x0 vào (1.2.9), ta được f(x0) = 1. +) Xét x = x0 Đặt 1 x−x0 = t thì t = 0 và x = x0 + 1 t , w(x) = x0 + 1 t + 1 x0 + d c . Khi đó có thể viết (1.2.9) dưới dạng f        x0 + 1 t + 1 x0 + d c        = −2 f(x0 + 1 t )+3, ∀t ∈ R{0}, (1.2.10) hay g t + 1 x0 + d c = −2g(t), ∀t ∈ R{0}, trong đó g(t) = f(x0 + 1 t )−1, ∀t ∈ R{0}. (1.2.11) Suy ra g(t) = 2 t t0 h(t), trong đó hàm h(t) tùy ý thỏa mãn h t + 1 x0 + d c = −h(t), ∀t = 0. 12
  • 15. Từ (1.2.10) và (1.2.11), ta có: Kết luận f(x) =    1 khi x = x0 g 1 x−x0 +1 khi x ∈ R{x0; −d c }, (1.2.12) trong đó g(t) = 2 t t0 h(t), ∀t = 0, với h(t) là hàm tùy ý thỏa mãn: h t + 1 x0 + d c = −h(t), ∀t = 0. Bài toán 1.2.6. Cho hàm số: w(x) = 2 3−x . Tìm tất cả các hàm số f : R{3} → R sao cho f(w(x)) = 2f(x)−3, ∀x = 3. (1.2.13) Giải. Nhận xét rằng phương trình w(x) = x có hai nghiệm phân biệt x = 1 và x = 2. Thay x = 1 hoặc x = 2 vào (1.2.13) ta được f(1) = f(2) = 3 Xét x = 1 và x = 2 Đặt t = x−1 x−2 thì t /∈ {2,1,0}.Do đó x = 2t −1 t −1 = 2+ 1 t −1 , 2 3−x = 2+ 1 t 2 −1 . Viết (1.2.13) dưới dạng sau f 2+ 1 t 2 −1 = 2 f 2+ 1 t −1 −3, ∀t /∈ {2,1,0}, hay g t 2 = 2g(t)−3 13
  • 16. g(t) = f 2+ 1 t −1 , ∀t /∈ {2,1,0}. (1.2.14) Đặt g(t) = 3+t−1 h(t), ∀t /∈ {2,1,0}, và viết (1.2.14) dưới dạng 3+ t 2 −1 h t 2 = 2[3+t−1 h(t)]−3, ∀t /∈ {2,1,0}. Vậy h t 2 = h(t), ∀t /∈ {2,1,0}. (1.2.15) Từ (1.2.14) và (1.2.15), ta có Kết luận f(x) =    3, khi x ∈ {1;2} g x−1 x−2 , khi x /∈ {1;2;3}, (1.2.16) trong đó g(t) = 3+t−1 h(t), với h(t) là hàm tùy ý thỏa mãn h t 2 = h(t), ∀t /∈ {2,1,0}. Bài toán 1.2.7. Xác định hàm số f(x) thỏa mãn điều kiện sau: f 2 x+2 = −3 f(x)+7, ∀x ∈ R{−2}. (1.2.17) Giải. Nhận xét phương trình 2 x−2 = x có hai nghiệm phân biệt x1 = −1− √ 3 và x2 = −1+ √ 3. +)Thay x = x1 hoặc x = x2 vào (2.1.17) ta được f(x1) = f(x2) = 7 4 . +) Xét x = x1 và x = x2. Đặt x−x1 x−x2 = t thì t /∈ { 1 α ,0,1}, α = 1+ √ 3 1− √ 3 . Và x = −1+ √ 3+ 2 √ 3 t −1 , 14
  • 17. 2 x+2 = −1+ √ 3+ 2 √ 3 αt −1 . Khi đó ta có thể viết (1.2.17) dưới dạng sau: f −1+ √ 3+ 2 √ 3 αt −1 = −3 f −1+ √ 3+ 2 √ 3 t −1 +7, ∀t /∈ { 1 α ,0,1}. Hay g(αt) = −3g(t)+7, ∀t /∈ { 1 α ,0,1} , trong đó g(t) = f −1+ √ 3+ 2 √ 3 t −1 , ∀t /∈ { 1 α ,0,1}. (1.2.18) Vì α = 1+ √ 3 1− √ 3 nên α = 0, |α| = 1. Đặt g(t) = |t|log|α|3 h(t)+ 7 4 , ∀t /∈ { 1 α ,0,1}. Khi đó ta có |αt|log|α|3 h(αt)+ 7 4 = −3|t|log|α|3 h(t)− 21 4 +7. Do đó h(αt) = −h(t), ∀t /∈ { 1 α ,0,1}. (1.2.19) Từ (1.2.18) và (1.2.19) ta có Kết luận f(x) =    7 4 , khi x ∈ {−1− √ 3;−1+ √ 3} g x+1+ √ 3 x+1− √ 3 , khi x /∈ {−1− √ 3;−1+ √ 3;−2}, (1.2.20) trong đó    t = x+1+ √ 3 x+1− √ 3 g(t) = |t|log|α|3 h(t)+ 7 4 , (1.2.21) 15
  • 18. với h(t) là hàm tùy ý thỏa mãn h(αt) = −h(t), ∀t /∈ { 1 α ,0,1}. Từ hai bài 1.2.6 và 1.2.7 ta xây dựng bài toán khái quát hóa trong trường hợp phương trình hàm sinh bởi phương trình phân tuyến tính w(x) = ax+b cx+d , c = 0, ad −bc = 0 thỏa mãn w(x) = x có hai nghiệm phân biệt x = x1, x = x2. Bài toán 1.2.8.[1] Cho hàm số: w(x) = ax+b cx+d , ad −bc = 0, c = 0, sao cho phương trình w(x) = x có hai nghiệm phân biệt x1, x2. Tìm tất cả các hàm số f : R{ −d c } → R sao cho f(w(x)) = 2 f(x)−3, ∀x = −d c . (1.2.22) Giải. Theo giả thiết thì phương trình w(x) = x có hai nghiệm phân biệt x = x1, x = x2. +)Thay x = x1 hoặc x = x2 vào (1.2.22),ta được: f(x1) = f(x2) = 3. +)Xét x = x1, x = x2, Đặt x−x1 x−x2 = t thì t /∈ { 1 α ,0,1}, α = cx2 +d cx1 +d , và x = x2 + x2 −x1 t −1 , ax+b cx+d = x2 + x2 −x1 αt −1 . Khi đó ta có thể viết (1.2.22) dưới dạng sau: f x2 + x2 −x1 αt −1 = 2 f x2 + x2 −x1 t −1 −3, ∀t /∈ { 1 α ,0,1}, hay g(αt) = 2g(t)−3, ∀t /∈ { 1 α ,0,1}, 16
  • 19. trong đó g(t) = f x2 + x2 −x1 t −1 , ∀t /∈ { 1 α ,0,1}. (1.2.23) Do x1 = x2 và c = 0 nên α = 1. Nếu α = −1 thì (1.2.23) cho ta f(x) = g(t) ≡ 3. Do x2 = −d c nên α = 0. •) Xét trường hợp α = 0 và |α| = 1. Đặt g(t) = 3+|t|log|α|2 h(t), ∀t /∈ { 1 α ,0,1}, khi đó ta có −3+(|α||t|)log|α|2 ·h(αt) = 2[3+|t|log|α|2 ·h(t)]−3. Do đó h(αt) = h(t), ∀t /∈ { 1 α ,0,1}. (1.2.24) •) Xét −1 = α < 0. Đặt g(t) = 3+|t|log|α|2 ·h(t), ∀t /∈ { 1 α ,0,1}, khi đó ta có 3+(|α||t|)log|α|2 ·h(αt) = 2[3+|t|log|α|2 ·h(t)]−3. Do đó h(αt) = h(t), ∀t /∈ { 1 α ,0,1}. (1.2.25) Từ (1.2.23), (1.2.24) và (1.2.25), ta có Kết luận: Nếu α = −1 thì f(x) = 3. Nếu α = 0 và |α| = 1 thì f(x) =    3, khix = x1, x = x2, g x−x1 x−x2 , khi x /∈ {x1,x2, −d c }, (1.2.26) 17
  • 20. trong đó g(t) = 3+|t|log|α|2 ·h(t), với h(t) là hàm tùy ý thỏa mãn h(αt) = h(t), ∀t /∈ { 1 α ,0,1}. Tiếp theo, ta minh họa một số bài toán cụ thể trong trường hợp phương trình hàm sinh bởi hàm phân tuyến tính w(x) = ax+b cx+d , c = 0, ad −bc = 0 thỏa mãn w(x) = x không có nghiệm thực . Bài toán 1.2.9. Cho hàm số w(x) = x−3 x−1 . Tìm tất cả các hàm số f : R{1} → R sao cho f(w(x))+ f(x) = 5, ∀x = 1. (1.2.27) Giải. Nhận xét rằng phương trình w(x) = x không có nghiệm thực và w(w(x)) ≡ x. Ta chứng minh mọi hàm dạng f(x) = 1 2 [g(w(x))−g(x)]+ 5 2 , (1.2.28) với g(x) tùy ý xác định trên R{1}, đều là nghiệm của (1.2.27). Thật vậy, nếu f(x) có dạng (1.2.27) thì f(w(x))+ f(x) = 1 2 [g(x)−g(w(x))]+ 5 2 + 1 2 [g(w(x))−g(x)]+ 5 2 = 5, ∀x = 1. Ngược lại, khi f(x) thỏa mãn điều kiện (1.2.27) thì chỉ cần chọn g(x) = f(w(x)) ta có ngay biểu diễn (1.2.28). Kết luận: f(x) = 1 2 [g(w(x))−g(x)]+ 5 2 , với g(x) là hàm tùy ý xác định trên R{1}. Bài toán 1.2.10.[1] Cho các hàm số h(x), x ∈ R và w(x) = 2x−5 x−2 . Tìm tất cả các hàm số f : R{2} → R sao cho f(w(x)) = f(x)+h(x), ∀x = 2. (1.2.29) 18
  • 21. Giải. Nhận xét rằng phương trình w(x) = x không có nghiệm thực và w(w(x)) ≡ x. Thay x bởi w(x), từ (1.2.29) ta được h(w(x)) = −h(x), (1.2.30) Vậy điều kiện cần để (1.2.29) có nghiệm là điều kiện (1.2.30) được thỏa mãn Giả sử điều kiện (1.2.30) được thỏa mãn. Khi đó h(x) = 1 2 [h(x)−h(w(x))], ∀x = 2. Ta chứng minh rằng mọi hàm dạng f(x) = 1 2 [g(w(x))+g(x)−h(x)], (1.2.31) với g(x) tùy ý xác định trên R{2}, là nghiệm của (1.2.29). Thật vậy, nếu f(x) có dạng (1.2.31) thì f(w(x)) = 1 2 [g(w(x))+g(x)−h(w(x))] = 1 2 [g(w(x))+g(x)+h(x)] = 1 2 [g(w(x))+g(x)−h(x)+2h(x)] = f(x)+h(x). Ngược lại, khi f(x) thỏa mãn điều kiện (1.2.29) thì chỉ cần chọn g(x) = f(x), ta có ngay biểu diễn (1.2.31). Kết luận f(x) = 1 2 [g(w(x))+g(x)−h(x)], với g(x) là hàm tùy ý xác định trên R{2}. Bài toán1.2.11.[1] Cho hàm số w(x) = −1 x+1 . Tìm tất cả các hàm số f : R{−1;0} → R sao cho f(w(w(x)))+ f(w(x))+ f(x) = 3, ∀x = −1; x = 0. (1.2.32) Giải. Nhận xét rằng phương trình w(x) = x không có nghiệm thực Ta có đẳng thức sau đây: w2(x) := w(w(x)) = −1 −1 x+1 +1 = − x+1 x , 19
  • 22. w3(x) := w(w(w(x))) = −1 − x+1 x +1 = x, ∀x ∈ R{−1;0}. Từ (1.2.32) ta thấy f(x) ≡ 1 là một nghiệm của bài toán. Đặt f(x) = 1+g(x). Khi đó có thể viết (1.2.32) dưới dạng g(w2(x))+g(w(x))+g(x) = 0, ∀x = −1; x = 0. (1.2.33) Ta chứng minh rằng mọi nghiệm của (1.2.33) đều có dạng: g(x) = 1 3 [2h(x)−h(w2(x))−h(w(x))], (1.2.34) với h(x) là hàm tùy ý xác định trên R{−1;0}. Thật vậy, khi g(x) có dạng (1.2.34) thì g(w2(x))+g(w(x))+g(x) = 1 3 [2h(w2(x))−h(w(x))−h(x)]+ 1 3 [2h(w(x))−h(x)−h(w2(x))]+ + 1 3 [2h(x)−h(w2(x))−h(w(x))] = 0 ,∀x ∈ R{−1;0}. Ngược lại, khi g(x) thỏa mãn (1.2.33) thì chỉ cần chọn h(x) = g(x) ta có ngay công thức biểu diễn (1.2.34). Kết luận:   f(x) ≡ 1, ∀x ∈ R{−1;0}, f(x) = 1+ 1 3 [2h(x)−h(w2(x))−h(w(x))], với h(x) là hàm tùy ý xác định trên R{−1;0}. Bài toán 1.2.12.[1] Cho hàm số q(x) xác định trên R và w(x) = −1 x+1 . Tìm tất cả các hàm số f : R{−1;0} → R sao cho f(w(w(x)))+ f(w(x))+ f(x) = q(x), ∀x ∈ R{−1;0}. (1.2.35) Giải. Nhận xét rằng phương trình w(x) = x không có nghiệm thực và w(x) có những tính chất w2(x) := w(w(x)) = −1 −1 x+1 +1 = − x+1 x , w3(x) := w(w(w(x))) = − −1 x+1 +1 −1 x+1 = x, ∀x ∈ R{−1;0}. 20
  • 23. Từ tính chất này của hàm w(x), suy ra điều kiện cần để phương trình (1.2.35) có nghiệm là q(w(x)) = q(x), ∀x ∈ R{−1;0}. (1.2.36) Giả sử điều kiện (1.2.36) thỏa mãn. Khi đó có thể viết: q(x) = 1 3 [q(w2(x))+q(w(x))+q(x)], ∀x ∈ R{−1;0}. (1.2.37) Từ (1.2.37) ta thấy f(x) ≡ 1 3 q(x) là một nghiệm. Đặt f(x) = 1 3 q(x)+g(x). Khi đó có thể viết (1.2.35) dưới dạng: g(w2(x))+g(w(x))+g(x) = 0, ∀x ∈ R{−1;0}. (1.2.38) Ta chứng minh rằng mọi nghiệm của (1.2.38) đều có dạng: g(x) = 1 3 [2h(x)−h(w2(x))−h(w(x))], (1.2.39) với h(x) là hàm tùy ý xác định trên R{−1;0}. Thật vậy, khi g(x) có dạng (1.2.39) thì g(w2(x))+g(w(x))+g(x) = 1 3 [2h(w2(x))−h(w(x))−h(x)]+ + 1 3 [2h(w(x))−h(x)−h(w2(x))]+ 1 3 [2h(x)−h(w2(x))−h(w(x))] = 0. Ngược lại, khi g(x) thỏa mãn điều kiện (1.2.38) thì ta chỉ việc chọn h(x) = g(x) ta có ngay biểu diễn (1.2.39). Kết luận: Điều kiện cần để phương trình (1.2.35) có nghiệm là: q(w(x)) = q(x), ∀x ∈ R{−1;0}. Khi đó mọi nghiệm của (1.2.35) có dạng: f(x) = 1 3 q(x)+ 1 3 [2h(x)−h(w2(x))−h(w(x))], 21
  • 24. với h(x) là hàm tùy ý xác định trên R{−1;0}. Bài toán 1.2.13.[1] Cho hàm số p(x) và q(x) xác định trên R và w(x) = −1 x . Tìm tất cả các hàm số f : R{0} → R sao cho p(x)f(w(x))+ f(x) = q(x), ∀x = 0. (1.2.40) Giải. Nhận xét rằng, phương trình w(x) = x không có nghiệm thực và w(x) có tính chất w2(x) := w(w(x)) = x, ∀x = 0. Thay x bởi w(x) vào (1.2.40), ta được p(w(x))f(x)+ f(w(x)) = q(w(x)), ∀x = 0. (1.2.41) Nhận thấy rằng (1.2.40) và (1.2.41) là hệ hai phương trình tuyến tính đối với hai ẩn là f(w(x)) và f(x). Nếu [1− p(w(x))p(x)] = 0, ∀x = 0 thì: f(x) = q(x)−q(w(x))p(x) 1− p(x)p(w(x)) , (1.2.42) f(w(x)) = q(w(x))− p(w(x))q(x) 1− p(x)p(w(x)) , ∀x = 0. (1.2.43) Các công thức (1.2.42) và (1.2.43) xác định cùng một hàm số f(x) thỏa mãn phương trình (1.2.40). Nếu tồn tại x0 = 0 sao cho [1− p(w(x0))p(x0)] = 0 thì điều kiện cần để phương trình (1.2.40) có nghiệm là: q(x0)− p(x0)q(w(x0)) = 0, và q(w(x0))− p(w(x0))q(x0) = 0. (1.2.44) Giả sử điều kiện (1.2.44) được thỏa mãn tại mọi điểm x0 = 0 sao cho 1− p(w(x0))p(x0) = 0. 22
  • 25. Gọi Zpq là tập hợp các nghiệm x = 0 của phương trình (với ẩn là x ) 1− p(w(x))p(x) = 0. Nhận xét rằng nếu x0 = 0 thuộc Zpq thì w(x0) cũng thuộc Zpq. Khi đó nghiệm của (1.2.40) được xác định theo cách sau. a) Nếu x = 0 và x /∈ Zpq thì: f(x) = q(x)−q(w(x))p(x) 1− p(x)p(w(x)) . b) Nếu x = 0 và x ∈ Zpq thì f(x) được chọn tùy ý sao cho (1.2.40) được thỏa mãn. Kết luận: +) Nếu 1− p(x)p(w(x)) = 0, ∀x = 0 thì f(x) = q(x)−q(w(x))p(x) 1− p(x)p(w(x)) . +) Nếu tồn tại x0 = 0 sao cho 1 − p(w(x0))p(x0) = 0 thì điều kiện để (1.2.40) có nghiệm là q(x)− p(x)p(w(x)) = 0 và q(w(x))− p(w(x))p(x) = 0, ∀x ∈ Zpq với Zpq là tập nghiệm khác 0 của phương trình 1− p(x)q(w(x)) = 0. Khi đó: f(x) =    q(x)−q(w(x))p(x) 1− p(x)p(w(x)) , ∀x ∈ Zpq tùy ý , ∀x /∈ Zpq,x = 0. (1.2.45) 1.2.2. Một số bài toán khác về hàm phân tuyến tính Bài toán 1.2.14.[4] Tìm tất cả các hàm số f : R{0} → R thỏa mãn đồng thời các điều kiện: 1)f(1) = 1; 2)f 1 x+y = f 1 x +f 1 y ; 3)(x+y)f(x+y) = yx f(x)f(y); Với mọi x,y mà xy(x+y) = 0. 23
  • 26. Giải. Trước hết chú ý rằng từ các giả thiết ta suy ra f(x) = 0 với mọi x = 0. Thật vậy, giả sử có x0 = 0 mà f(x0) = 0. Khi đó từ giả thiết 1) ta có x0 = 1. Thay x = 1−x0, y = x0 vào giả thiết 3) ta có: (1−x0 +x0)f(1−x0 +x0) = (1−x0)x0 f(1−x0)f(x0) ⇒ f(1) = 0, vô lý . Vậy với mọi x = 0 thì f(x) = 0. Từ giả thiết 2) thay x = y ta có: f 1 2x = 2 f 1 x , ∀x = 0. (1.2.46) Từ (1.2.46) thay x bởi 1 2y ta có: f(y) = 2 f(2y) ⇒ f(x) = 2 f(2x) ∀x = 0. (1.2.47) Mặt khác, từ giả thiết 3) thay x = y ta có: 2x f(2x) = x2 (f(x))2 , ∀x = 0. (1.2.48) Như vậy (1.2.47) và (1.2.48) cho ta x f(x) = x2 (f(x))2 , ∀x = 0. hay f(x) = x(f(x))2 ⇒ f(x) = 1 x với mọi x = 0 (thỏa mãn). Kết luận: f(x) = 1 x , ∀x = 0. Bài tập 1.2.15.[4] Tìm tất cả các hàm f : R → R thỏa mãn đồng thời các điều kiện 1)f(−x) = −f(x), ∀x ∈ R. 2)f(x+1) = f(x)+1, ∀x ∈ R. 3)f 1 x = f(x) x2 , ∀x = 0. Giải. Từ 1) ta có: f(0) = −f(0) ⇒ f(0) = 0. Vì thế f(1) = f(0)+1 = 1. Từ 2) thay x = −1 ta có: f(0) = f(−1)+1 ⇒ f(−1) = −1. 24
  • 27. Bây giờ ta xác định hàm số với x /∈ {−1;0}. Ta có: f x+1 x = f 1+ 1 x = f(1)+ f 1 x = 1+ f 1 x . (1.2.49) Mặt khác f x+1 x = x+1 x 2 f x x+1 = x+1 x 2 f 1− 1 x+1 = x+1 x 2 1− f 1 x+1 = x+1 x 2 1− 1 (x+1)2 f(x+1) = 1 x2 (x+1)2 − f(x)−1 . Do đó ta có: f(x)+x2 = x2 +2x− f(x) ⇒ f(x) = x. Thử lại ta thấy f(x) = x thỏa mãn các điều kiện 1), 2), 3). Kết luận: f(x) = x, ∀x ∈ R. Bài tập 1.2.16.[4] Tìm tất cả các hàm f : R+ → R+ thỏa mãn f(x+y)+ f(x)f(y) = f(xy)+ f(x)+ f(y), ∀x,y ∈ R+ . (1.2.50) Giải. Cho x = y = 2 ta được: f(4)+(f(2))2 = f(4)+2 f(2) ⇒ (f(2))2 −2 f(2) = 0 ⇒ f(2) = 2( do f(x) > 0). Trong (1.2.50) lại cho x =y =1 có f(2)+[f(1)]2 = f(1)+2 f(1) hay [f(1)]2 −3 f(1)+2 = 0 ⇒ f(1) = 1 hoặc f(1)=2. a) Xét trường hợp f(1) = 2. Trong (1.2.50) thay y = 1 ta được: f(x+1)+2 f(x) = f(x)+ f(x)+ f(1) ⇒ f(x+1) = 2. Từ đây suy ra f(x) = 2, ∀x > 1. Với 0 < x < 1 ta chọn y = 1 x > 1 thì ta có: f(x+ 1 x )+ f(x)f 1 x = f(1)+ f(x)+ f 1 x , 25
  • 28. ⇒ 2+2 f(x) = 2+ f(x)+2 ⇒ f(x) = 2. Như vậy ta có một hàm số cần tìm là f(x) = 2, ∀x ∈ R+. b) Xét trường hợp f(1) = 1. Thay y = 1 vào (1.2.50): f(x+1)+ f(x) = f(x)+ f(x)+ f(1) Như thế: f(x+1) = f(x)+1, ∀x ∈ R+ . (1.2.51) Từ (1.2.51) bằng phép quy nạp đơn giản ta có f(n) = n, ∀n ∈ N∗ và f(x+n) = f(x)+n, ∀x ∈ R+ ,∀n ∈ N∗ . (1.2.52) Từ (1.2.50) thay x = n, y = 1 n ta có: f n+ 1 n +f(n)f 1 n = f(1)+ f(n)+ f 1 n . Chú ý rằng từ (1.2.52) có: f n+ 1 n = n+ f 1 n . Vậy: n+ f 1 n +nf 1 n = 1+ f(n)+ f 1 n ⇒ f 1 n = 1 f(n) = 1 n ,∀n ∈ N∗ . Với mọi số hữu tỷ m n ,∀m,n ∈ N∗, ta thay x = m,y = 1 n thì được: f m+ 1 n +f(m)f 1 n = f m n +f(m)+ f 1 n ⇒ m+ 1 n + m n = f m n +m+ 1 n , suy ra f m n = m n . Như thế đã chứng minh được rằng ∀x ∈ Q+ thì f(x) = x. Ta chứng minh f là hàm đồng biến trên khoảng (0;+∞). Trước hết chú ý rằng ∀x > 1 chọn y = x x−1 > 0 ⇒ x+y = xy ⇒ f(x+y) = f(xy). 26
  • 29. Khi đó (1.2.50) trở thành f(x)f x x−1 = f(x)+ f x x−1 ⇒ f x x−1 (f(x)−1) = f(x). Từ đây suy ra f(x)−1 > 0 ⇒ f(x) > 1 và f x x−1 = f(x) f(x)−1 ,∀x > 1. (1.2.53) Mặt khác từ (1.2.53) có: 1+ f 1 x = f 1+ 1 x = f x+1 x = f(x+1) f(x+1)−1 = f(x)+1 f(x) . Như vậy: f 1 x = 1 f(x) , ∀x ∈ R+ , do đó: Nếu x > 1 thì f(x) > 1, và nếu 0 < x < 1 thì 0 < f(x) < 1. • Xét 0 < x < y < 1. Ta có: f(y−x+x)+ f(y−x)f(x) = f((y−x)x)+ f(y−x)+ f(x). Hay f(y) = f((y−x)x)+ f(y−x)+ f(x)− f(y−x)f(x). Nghĩa là: f(y) = f(x)+ f((y−x)x)+ f(y−x)(1− f(x)). Để ý rằng do 0 < x < 1 nên f(x) < 1, vì thế f(y) > f(x). Do đó hàm f đồng biến trên khoảng (0;1). • Xét 1 < x < y. Suy ra: 0 < 1 y < 1 x < 1 ⇒ f( 1 y ) < f( 1 x ) ⇒ 1 f(y) < 1 f(x) ⇒ f(y) > f(x). Như thế f đồng biến trên (1;+∞). Vậy hàm f là hàm đồng biến trên khoảng (0;+∞). Cuối cùng với mỗi x > 0 ta chọn hai dãy số hữu tỷ (un),(vn) sao cho: un ≤ x ≤ vn và limun = limvn = x. 27
  • 30. Khi đó do f đồng biến nên f(un) ≤ f(x) ≤ f(vn) ⇒ un ≤ f(x) ≤ vn. Cho n → +∞ ta được f(x) = x. Thử lại thấy thỏa mãn. Kết luận: Có hai hàm số thỏa mãn đầu bài là: f(x) = 2 và f(x) = x, ∀x ∈ R+ . Bài toán 1.2.17. (Komal-A328 ) [4] Tìm tất cả các hàm f : R+ → R+ thỏa mãn: f(f(x)+y) = x f(1+xy), ∀x,y > 0. (1.2.54) Giải. Trước hết ta chứng minhf là hàm số giảm. Giả sử có 0 < u < v mà f(u) < f(v). Chọn w = v f(v)−uf(u) v−u . Khi đó uf(u) < uf(v) ⇒ v f(v)−u f(u) > vf(v)−uf(v) = (v−u)f(v). Vậy: w = v f(v)−uf(u) v−u > f(v) > f(u). Ta có: f(w) = f(f(u)+(w− f(u))) = uf 1+ uv(f(v)− f(u)) v−u . Lại có: f(w) = f(f(v)+(w− f(v))) = v f 1+ uv(f(v)− f(u)) v−u . Như thế u = v, vô lý. Vậy ta phải có f(u) ≥ f(v) nghĩa là f hàm không tăng. Thay x = 1 vào (1.2.54) có f(f(1)+y) = f(1+y). (1.2.55) 28
  • 31. Nếu f(1) > 1 thì từ (1.2.55) suy ra f(y) = f(y + f(1) − 1), ∀y > 1, còn nếu f(1) < 1 thì lại có f(y) = f(y+1− f(1)), ∀y > 1. Như thế trong cả hai trường hợp thì hàm f luôn tuần hoàn trên khoảng (1;+∞) với chu kỳ là |f(1)−1|. Như thế f là hàm đơn điệu tuần hoàn nên phải là hàm hằng với x > 1. Từ (1.2.54) ta thấy rằng vế trái là hằng số nhưng vế phải lại lớn tùy ý, vô lý. Do đó f(1) = 1. Xét x > 1. Chọn y = x−1 x ⇒ xy+1 = x, khi đó (1.2.54) trở thành f f(x)+ x−1 x = x f(x). Nếu f(x) > 1 x thì f(x)+ x−1 x = f(x)− 1 x +1 > 1 ⇒ f f(x)− 1 x +1 ≤ f(1) = 1. Vậy x f(x) ≤ 1 ⇒ f(x) ≤ 1 x , muân thuẫn. Tương tự cũng không thể có f(x) < 1 x . Do đó với x > 1 thì f(x) = 1 x . Cuối cùng với x > 0 chọn y = 1 có f(f(x)+1) = x f(1+x). Nhưng vì f(x)+1 > 1 và x+1 > 1 nên f(f(x)+1) = 1 f(x)+1 và f(x+1) = 1 x+1 . Do đó 1 f(x)+1 = x x+1 ⇒ x f(x)+x = x+1 ⇒ f(x) = 1 x . Thử lại thấy f(x) = 1 x thỏa mãn. Kết luận: Hàm số cần tìm là: f(x) = 1 x ,∀x ∈ R+ . Bài toán1.2.18.[4] Tìm tất cả các hàm xác định trên tập các số thực dương và nhận giá trị trong tập đó thỏa mãn f(x f(y))f(y) = f(x+y), ∀x,y > 0. (1.2.56) 29
  • 32. Giải. Giả sử có y>0 mà f(y) >1. Khi đó chọn x = y f(y)−1 > 0 ⇒ x f(y) = x+y Ta có f(x f(y)) = f(x+y) = f(x f(y))f(y). Nhưng f(x f(y)) = f(x+y) > 0 nên ta suy ra f(y)=1, vô lý. Vậy với mỗi y >0 thì 0 < f(y) ≤ 1. Từ đây suy ra kết quả sau f(x+y) = f(x f(y))f(y) ≤ f(y), ∀x,y > 0 hay với 0 <x < y ta có: f(y) = f(y−x+x) = f((y−x)f(x))f(x) ≤ f(x) ⇒ f là hàm số giảm Nếu có a >0 và f(a)=1 thì f(y f(a))f(a) = f(a+y) ⇒ f(a+y) = f(y), ∀y > 0 Bằng phương pháp quy nạp dễ có kết quả f(na+x) = f(x),∀x > 0,∀n ∈ N Ta chứng minh f là hàm số hằng. Thật vậy: Xét 0 < x < y Đặt n = y−x a ⇒ n ≤ y−x a < n+1 ⇒ na+x ≤ y < (n+1)a+x Vì f giảm nên ta có f(na+x) ≥ f(y) ≥ f((n+1)a+x). Mặt khác f(na+x) = f((n+1)a+x) = f(x). Vậy f(x) ≤ f(y) ≤ f(x) ⇒ f(x) = f(y) Thành thử f là hàm hằng mà f(a) =1, do đó f(x) = 1, ∀x > 0. Cuối cùng ta xét trường hợp 0 < f(x) < 1, ∀x > 0. Ta thấy là hàm đơn điệu, giảm thực sự, vì với 0 < x < y ta có: f(y) = f(y−x+x) = f((y−x)f(x))f(x) < f(x) Từ hệ thức (1.2.56) cho y=1 ( đặt f(1) = a < 1) ta được f(ax)a = f(x+1) = f(ax+1+x(1−a)) = f(ax)f((1+x−ax)f(ax)) 30
  • 33. Suy ra f((1+x−ax)f(ax)) = a = f(1) ⇒ 1 = (1+x−ax)f(ax)( do f giảm thực sự ) Vậy f(ax) = 1 1+x−ax . Cuối cùng thay x bởi x a ta được f(x) = 1 1+ x a −a x a = a a+(1−a)x , ∀x > 0 Thử lại: Dễ thấy các hàm số f(x) = 1, ∀x > 0 và f(x) = a a+(1−a)x , ∀x > 0 hoàn toàn thỏa mãn điều kiện của bài toán. Kết luận Các hàm số cần tìm là   f(x) = 1, ∀x > 0, f(x) = a a+(1−a)x , ∀x > 0, a tùy ý thuộc khoảng(0;1). Bài toán 1.2.19. ( IMO -1986 ) [4] Hãy xác định tất cả các hàm f xác định trên tập hợp các số thực không âm và nhận giá trị trong tập đó thỏa mãn điều kiện: f(x f(y))f(y) = f(x+y), ∀x,y ≥ 0. (1.2.57) Giải. Thay x = y = 0 vào (2.2.57), ta được f(0 f(0))f(0) = f(0) ⇒ [f(0)]2 = f(0) ⇒ f(0)(f(0)−1) = 0 ⇒ f(0) = 0hoặc f(0) = 1. Nếu f(0) = 0 thì với mọi x > 0: f(x f(0))f(0) = f(x) ⇒ f(x) = 0, ∀x ≥ 0. Ta xét trường hợp f(0) = 1. Có hai khả năng sau đây: 1) f(x) > 0,∀x > 0 theo bài toán 1.2.18, các hàm số cần tìm là: f(x) = 1, ∀x ≥ 0 và f(x) = a a+(1−a)x , ∀x ≥ 0. 2) Tồn tại a,b > 0 sao cho f(a) > 0 và f(b) = 0. Ta có với 0 < x < a thì f((a−x)f(x))f(x) = f(a−x+x) = f(a) > 0 ⇒ f(x) > 0. 31
  • 34. Vì f(b) = 0 nên suy ra b > a. Lại có với x > b thì f(x) = f(x−b+b) = f((x−b)f(b))f(b) = 0. Vậy với 0 ≤ x ≤ a thì f(x) > 0 và với x ≥ b thì f(x) = 0. Do đó nếu f(x) > 0 thì x < b và nếu f(x) = 0 thì x > a. Ta xét hai tập hợp: A = {x ≥ 0|f(x) > 0}; B = {x ≥ 0|f(x) = 0}. Rõ ràng A,B là hai tập khác rỗng vì a ∈ A và b ∈ B. Mặt khác A là tập bị chặn trên bởi số b vì ∀x ∈ A thì f(x) > 0 suy ra x < b và B là tập bị chặn dưới bởi số a vì ∀x ∈ B thì f(x) = 0 suy ra x > a. Thành thử tồn tại α = Sup A, β = inf B, Ta sẽ chứng minh α = β. Thật vậy, giả sử α < β suy ra có số ϕ sao cho α < ϕ < β. Từ α < ϕ, ta thấy nếu f(ϕ) > 0 thì ϕ ∈ A ⇒ ϕ ≤ α trái với α < ϕ < β. Do đó f(ϕ) = 0 ⇒ ϕ ∈ B ⇒ ϕ ≥ β cũng mâu thuẫn. Nếu α > β thì có số ϕ sao cho α > ϕ > β, khi đó có a ∈ A và b ∈ B sao cho a ≥ ϕ ≥ b. Điều này không đúng với mọi a ∈ A và với mọi b ∈ B ta luôn có a < b. Vậy α = β. Bây giờ ta tìm công thức của hàm f. Xét 0 ≤ x < α. Giả sử f(x) < α α −x ⇒ α −x < α f(x) . Chọn y sao cho α −x < y < α f(x) ⇒ x+y > α và yf(x) < α. Vì yf(x) < α nên có a ∈ A sao cho y f(x) ≤ a ⇒ f(yf(x)) > 0. Do đó f(x+y) = f(yf(x))f(x) > 0 ⇒ x+y ∈ A ⇒ x+y ≤ α, vô lý . Còn nếu f(x) > α α −x ⇒ α −x > α f(x) , 32
  • 35. thì ta chọn số y sao cho α −x > y > α f(x) ⇒ x+y < α và yf(x) > α. Do x+y < α và x < α nên có a ∈ A và a ∈ A mà x+y ≤ a và x ≤ a . Khi đó ta có f(x+y) > 0 và f(x) > 0. Vậy f(yf(x)) > 0 ⇒ y f(x) ∈ A ⇒ y f(x) ≤ α, vô lý. Như vậy ta đã chứng minh được rằng với 0 ≤ x < α thì f(x) = α α −x . Xét x > α = β, khi đó có b ∈ B mà x ≥ b ⇒ f(x) = 0. Cuối cùng ta tính giá trị của hàm số tại điểm α. Gọi x, y > 0 là hai số tùy ý thỏa mãn x+y = α ⇒ 0 < x < α và 0 < y < α ⇒ f(x) = α α −x và f(y) = α α −y Ta có x f(y) = αx α −y = αx x = α, do đó từ f(x f(y))f(y) = f(x+y) ⇒ f(α)f(y) = f(α). Chú ý rằng f(y) = α α −y = 1 nên f(α) = 0. Tóm lại, trong trường hợp thứ hai ta có hàm số: f(x) =    α α −x , khi 0 ≤ x < α 0, khi x ≥ α. (1.2.58) Thử lại thấy thỏa mãn điều kiện của bài toán. Kết luận: Các hàm số cần tìm là f(x) = 0, ∀x ≥ 0, f(x) = 1, ∀x ≥ 0, f(x) = a a+(1−a)x , (0 < a < 1), ∀x ≥ 0. Nếu tồn tại 0 < a < b mà f(a) > 0 và f(b) = 0 thì: f(x) =    α α −x , khi 0 ≤ x < α 0 ,khi x ≥ α, (1.2.59) với α = Sup{x ∈ R+|f(x) > 0} = inf {x ∈ R+|f(x) = 0}. 33
  • 36. 1.2.3. Bài tập tham khảo Bài tập 1.2.20. Cho a,b,c,d, p,q ∈ R, c = 0. Xác định các hàm f(x) sao cho f ax+b cx+d = p f(x)+q, ∀x ∈ R{ −d c }. Bài tập 1.2.21. Tìm hàm f(x) thỏa mãn điều kiện f(x)f( 1 x ) = 1, ∀x = 0. Bài tập 1.2.22. Tìm hàm f(x) thỏa mãn điều kiện f(x)f 1 1−x = 1, ∀x = 0, x = 1. Bài tập 1.2.23. Cho hàm số h(x), x ∈ R{0}. Tìm hàm f(x) thỏa mãn điều kiện f 1 x = x f(x)+h(x), ∀x = 0. Bài tập 1.2.24. Tìm hàm f(x) thỏa mãn điều kiện f(x)f 1 1−x = x−1, ∀x = 0, x = 1. Bài tập 1.2.25. Tìm hàm f(x) thỏa mãn điều kiện f(x)+ f 1 1−x +f x−1 x = −1, ∀x = 0, x = 1. Bài tập 1.2.26. Cho hàm số w(x) = −2 x+1 Ký hiệu w2(x) = w(w(x)) , w3(x) = w(w(w(x))) Tìm tất cả các hàm số f : R{−1;−3;1} → R sao cho    f(w3(x)) = f(x) f(w2(x))+ f(w(x))+ f(x) = 3, ∀x /∈ {−1;−3;1}. (1.2.60) 34
  • 37. Bài tập 1.2.27. Cho hàm số q(x) xác định trên R và w(x) = −2 x+1 . Ký hiệu w2(x) = w(w(x)) , w3(x) = w(w(w(x))). Tìm tất cả các hàm số f : R{−1;−3;1} → R sao cho    f(w3(x)) = f(x) f(w2(x))+ f(w(x))+ f(x) = q(x), ∀x /∈ {−1;−3;1}. (1.2.61) Bài tập 1.2.28. Tìm hàm f : R → R thỏa mãn điều kiện f(x)+ 1 2x f 1 1−x = 1, ∀x ∈ R{0;1}. Bài tập 1.2.29. (Iberoamerican - 1987 ) Tìm hàm f(x) thỏa mãn điều kiện (f(x))2 f 1−x 1+x = 64x, ∀x ∈ R{0;±1}. Bài tập 1.2.30. ( Australian- 1992) Tìm tất cả các hàm số f : R{ 2 3 } → R thỏa mãn điều kiện 2f(x)+ f 2x 3x−2 = 996x, ∀x ∈ R{ 2 3 }. Bài tập 1.2.31.( CAMO - 1991) Chứng minh tồn tại đúng một hàm số f : R{0} → R sao cho f(x) = x f 1 x , và f(x+y) = f(x)+ f(y)−1, ∀x,y ∈ R{0} và x+y = 0. Bài tập 1.2.32. Tìm tất cả các hàm số f xác định trên R{±1} thỏa mãn f x−3 x+1 +f x+3 1−x = x, ∀x ∈ R{±1}. Bài tập 1.2.33. Tìm tất cả các hàm số f xác định trên tập hợp các số thực khác 0 sao cho: 1) f(−x) = −f(x), ∀x = 0; 2) f 1 x+y = f 1 x +f 1 y +2(xy−1000), ∀x,y ∈ R{0} và x+y = 0. Bài tập 1.2.34. Cho a,b,c > 0. Tìm tất cả các hàm f : R+ → R thỏa mãn: f(x)+af b x = cx. 35
  • 38. Chương 2 Một số bài toán về dãy số. 2.1. Phương trình và hệ phương trình sai phân. 2.1.1. Phương trình sai phân tuyến tính với hệ số hằng. • Phương trình sai phân tuyến tính cấp một là phương trình sai phân dạng u1 = α,aun+1 +bun = fn,n ∈ N∗ . trong đó a,b,α là các hằng số, a = 0 và fn là biểu thức của n cho trước. Cách giải. 1) Nếu fn = 0 thì giải phương trình đặc trưng aλ + b = 0 để tìm λ. Khi đó un = qλn (q là hằng số), trong đó q được xác định khi biết u1 = α. 2) Nếu fn là đa thức theo n thì giải phương trình đặc trưng aλ + b = 0 ta tìm được λ. Ta có un = un + un, trong đó un là nghiệm của phương trình thuần nhất aun+1 + bun = 0 và u∗ n là nghiệm riêng tùy ý của phương trình không thuần nhất aun+1 +bun = fn. Vậy un = qλn, q là hằng số sẽ được xác định sau. Ta xác định u∗ n như sau: a) Nếu λ = 1 thì un là đa thức cùng bậc với fn. b) Nếu λ = 1 thì u∗ n = n.gn với gn là đa thức cùng bậc với fn. Thay u∗ n vào phương trình, đồng nhất các hệ số, ta tính được các hệ số của u∗ n. 3) Nếu fn = νµn,n ∈ N∗ thì giải phương trình đặc trưng aλ +b = 0 để tìm λ. 36
  • 39. Ta có un = un + u∗ n, trong đó un = cλn,c là hằng số chưa được xác định, u∗ n được xác định như sau: a) Nếu λ = µ thì u∗ n = Aµn. b) Nếu λ = µ thì u∗ n = Anµn. Thay u∗ n vào phương trình, đồng nhất các hệ số tính được các hệ số của u∗ n. Biết u1, từ hệ thức un = un +u∗ n, tính được c. 4) Nếu fn = f1n + f2n,n ∈ N∗, trong đó f1n là đa thức theo n và f2n = νµn thì ta có un = un + un, trong đó un là nghiệm tổng quát của phương trình thuần nhất aun+1 + bun = 0,u∗ n là một nghiệm riêng của phương trình không thuần nhất aun+1 + bun = f1n,u∗∗ n là nghiệm riêng bất kỳ của phương trình không thuần nhất aun+1 +bun = f2n. • Phương trình sai phân tuyến tính cấp hai là phương trình sai phân dạng u1 = α,u2 = β,aun+1 +bun +cun−1 = fn,n ∈ N∗ trong đó a,b,c,α,β là các hằng số, a = 0 và fn là biểu thức chứa n cho trước. Cách giải. 1) Nếu fn = 0 thì giải phương trình đặc trưng aλ2 +bλ +c = 0, tìm λ. a) Nếu λ1,λ2 là hai nghiệm thực khác nhau thì un = Aλn 1 +Bλn 2 , trong đó A và B được xác định khi biết u1 và u2. b) Nếu λ1,λ2 là nghiệm thực kép, λ1 = λ2 = λ thì un = (A+Bn)λn, trong đó A và B được xác định khi biết u1 và u2. c) Nếu λ là nghiệm phức, λ = x+yi, thì ta đặt r =| λ |= x2 +y2,tanϕ = y x ,ϕ ∈ −π 2 , π 2 Lúc đó λ = r(cosϕ + isinϕ) và un = rn(Acosnϕ + Bsinnϕ), trong đó A và B được xác định khi biết u1 và u2. 2) Nếu fn là đa thức theo n cho trước thì giải phương trình đặc trưng aλ2 + bλ + c = 0 để tìm λ. Ta có un = un + u∗ n, trong đó un là nghiệm tổng quát của phương trình thuần nhất aun+1 +bun +cun−1 = 0 và u∗ n là một nghiệm riêng tùy ý của phương trình không thuần nhất aun+1 +bun +cun−1 = fn. Theo 1) ta tìm được un, trong đó các hệ số A,B chưa xác định, u∗ n được xác định như sau: a) Nếu λ = 1 thì u∗ n là đa thức cùng bậc với fn, 37
  • 40. b) Nếu λ = 1 là nghiệm đơn thì u∗ n = ngn,gn là đa thức cùng bậc với fn, c) Nếu λ = 1 là nghiệm kép thì u∗ n = n2gn,gn là đa thức cùng bậc với fn. Thay u∗ n vào phương trình, đồng nhất các hệ số của u∗ n, biết u1,u2, từ hệ thức un = un +u∗ n tính được A,B. 3) Nếu fn = νµn,n ≥ 2 thì giải phương trình đặc trưng aλ2 + bλ + c = 0, tìm λ. Ta có un = un +u∗ n, trong đó un được tìm như ở 1), hệ số A và B chưa được xác định, u∗ n xác định như sau: a) Nếu λ = µ thì u∗ n = kµn, b) Nếu nghiệm đơn λ = µ thì u∗ n = knµn, c) Nếu nghiệm kép λ = µ thì u∗ n = kn2µn. Thay u∗ n vào phương trình, dùng phương pháp đồng nhất hệ số sẽ tính được hệ số k. Biết u1,u2, từ hệ thức un = un +u∗ n tính được A,B. 4) Nếu fn = f1n + f2n,n ≥ 2 và f1n là đa thức theo n, f2n = νµn thì ta có un = un +u∗ n +u∗∗ n trong đó un là nghiệm tổng quát của phương trình thuần nhất, u∗ n là nghiệm riêng tùy của phương trình không thuần nhất aun+1 +bun +cun−1 = f1n,u∗∗ n là nghiệm riêng tùy ý của phương trình không thuần nhất aun+1 +bun +cun−1 = f2n. 5) Nếu fn = νcosn + µsinn (a = 0),n ≥ 2, thì phương trình đặc trưng aλ2 + bλ + c = 0, tìm λ. Ta có un = un + u∗ n, trong đó un là nghiệm của phương trình thuần nhất, xác định như ở 1), các hệ số A và B chưa được xác định u∗ n = kcosn +lsinn. Thay u∗ n vào phương trình, đồng nhất các hệ số, tính được k,l. Từ hệ thức un = un +u∗ n và u1,u2, ta tính được A và B. • Phương trình sai phân tuyến tính cấp ba là phương trình sai phân dạng u1 = α,u2 = β,u3 = γ,aun+2 +bun+1 +cun +dun−1 = fn,n ≥ 2 trong đó a,b,c,d,α,β,γ là các hằng số, a = 0 và fn là biểu thức của n cho trước. Phương trình sai phân tuyến tính cấp ba luôn giải được vì phương trình bậc ba luôn giải được. Nghiệm tổng quát của phương trình sai phân tuyến tính cấp ba có dạng un = un +u∗ n, trong đó u∗ n là nghiệm của phương trình tuyến tính không thuần nhất, un là nghiệm tổng quát của phương trình tuyến tính thuần nhất. Cách giải. 38
  • 41. 1) Xét phương trình đặc trưng aλ3 +bλ2 +cλ +d = 0. (∗) a) Nếu (∗) có 3 nghiệm thực phân biệt λ1,λ2,λ3 thì un = β1λn 1 +β2λn 2 +β3λn 3 . b) Nếu (∗) có một nghiệm thực bội 2 và một nghiệm đơn (λ1 = λ2 = λ3) thì un = (β1 +β2n)λn 1 +β3λn 3 . c) Nếu (∗) có một nghiệm thực bội 3 (λ1 = λ2 = λ3)thì un = (β1 +β2n+β3n2 )λn 1 . d) Nếu (∗) có một nghiệm thực λ1 và 2 nghiệm phức liên hợp λ2,3 = r(cosϕ ± isinϕ) thì un = β1λn 1 +rn (β2cosnϕ +β3sinnϕ). 2) Gọi u∗ n là một nghiệm riêng của phương trình tuyến tính không thuần nhất. Xét fn là đa thức của n. Ta có Nếu λ = 1 thì u∗ n là đa thức cùng bậc với fn, Nếu λ = 1 (nghiệm đơn) thì u∗ n = ngn,gn là đa thức cùng bậc với fn, Nếu λ = 1 (bội 2) thì u∗ n = n2gn,gn là đa thức cùng bậc với fn, Nếu λ = 1 (bội 3) thì u∗ n = n3gn,gn là đa thức cùng bậc với fn. Xét fn = νµn, ta có Nếu λ = µ thì u∗ n = k.n.µn, Nếu nghiệm đơn λ = µ thì u∗ n = k.µn, Nếu nghiệm bội λ = µ (bội s) thì u∗ n = k.ns.µn. 39
  • 42. 2.1.2. Hệ phương trình sai phân tuyến tính với hệ số hằng Là hệ phương trình sai phân dạng    xn+1 = pxn +qyn ,x1 = a, yn+1 = rxn +syn ,y1 = b. Cách giải. Trong hệ phương trình, thay n bởi n+1, ta nhận được xn+2 = pxn+1 +qyn+1 = pxn+1 +q(rxn +syn) = pxn+1 +qrxn +s(qyn) = pxn+1 +qrxn +s(xn+1 − pxn). Suy ra xn+2 −(p+s)xn+1 +(ps−qr)xn = 0, trong đó x1 = a. Từ hệ phương trình, ta lại có x2 = px1 + qy1 = pa + qb. Như vậy ta được phương trình sai phân tuyến tính cấp hai thuần nhất x1 = a,x2 = pa+qb,xn+1 −(p+s)xn +(ps−qr)xn−1 = 0,n ≥ 2. Giải phương trình này, ta tìm được xn. Thay xn vào hệ phương trình, ta tìm được yn. 40
  • 43. 2.2. Phương trình sai phân dạng phân tuyến tính với hệ số hằng Bài toán 2.2.1. Tìm xn, biết x1 = a > 0 và xn+1 = xn 2+xn (n ∈ N) (2.2.1) Giải. Nhận xét rằng xn > 0 với mọi n ∈ N. Đặt 1 xn = yn. Khi đó ta có thể viết (2.2.1) dưới dạng yn+1 −2yn = 1. Suy ra yn = (a+1)2n−1 −a a và xn = a (a+1)2n−1 −a . Bài toán 2.2.2.[2] Tìm xn, biết x1 = a > 0 và xn+1 = anxn +bn (2.2.2) trong đó an = 0 với mọi n ∈ N. Giải. Đặt dãy phụ xn = yn n−1 ∏ k=0 ak. Khi đó ta có y1 = a a0 và (2.2.2) được viết dưới dạng yn+1 −yn = bn ∏n k=0 ak . (2.2.3) Từ đẳng thức (2.2.3) ta nhận được yn = y1 + n−1 ∑ k=1 bk ∏k i=0 ai = a a0 + n−1 ∑ k=1 bk ∏k i=0 ai Vậy nên ta có xn = a a0 + n−1 ∑ k=1 bk ∏k i=0 ai n−1 ∏ k=0 ak. 41
  • 44. Bài toán 2.2.3.[2] Xác định số hạng tổng quát của dãy số {xn}, nếu xn thỏa mãn phương trình dãy sau đây x0 = a,xn+1 = pxn +q rxn +s ,n ∈ N (2.2.4) trong đó p,q,r,s ∈ R cho trước. Giải. Giả sử un,vn là một nghiệm của hệ phương trình sai phân    un+1 = pun +qvn ,u0 = a, vn+1 = run +svn ,v0 = 1, (2.2.5) thì xn = un vn là nghiệm của phương trình (2.2.4). Thật vậy, ta chứng minh bằng quy nạp như sau x0 = u0 v0 = a 1 = a,(đúng). Giả sử xn = un vn là nghiệm của (2.2.4). Khi đó xn+1 = un+1 vn+1 = pun +qvn run +svn = un vn +q r un vn +s = pxn +q rxn +s cũng là nghiệm của (2.2.4). Giải hệ (2.2.5) bằng cách đưa về phương trình sai phân thuần nhất yn+2 = (p+s)yn+1 +(qr −sp)yn. Bài toán 2.2.4.[2] Tìm xn thỏa mãn điều kiện x1 = a, xn+1 = x2 n +d 2xn , n ∈ N∗ (2.2.6) Giải. Khi d =0 thì ta có ngay xn+1 = 1 2 xn và xn = ( 1 2 )n−1a Xét trường hợp d > 0. Giả sử un, vn là nghiệm của hệ phương trình sai phân    un+1 = u2 n +dv2 n, vn+1 = 2unvn, u1 = a, v1 = a (2.2.7) 42
  • 45. thì xn = un vn là nghiệm của phương trình (2.2.6). Thật vậy, ta chứng minh bằng quy nạp như sau x1 = u1 v1 = a 1 = a ( đúng ) Giả sử xn = un vn là nghiệm của (2.2.6). Khi đó xn+1 = un+1 vn+1 = u2 n +dv2 n 2unvn = x2 n +d 2xn cũng là nghiệm của (2.2.6) Như vậy để giải phương trình (2.2.6) ta cần giải hệ (2.2.7). Ta có    un+1 = u2 n +dv2 n √ dvn+1 = 2 √ dunvn, u1 = a, v1 = 1. (2.2.8) Cộng vế với vế của các phương trình trong hệ (2.2.8) ta thu được un+1 + √ dvn+1 = (un + √ dvn)2 Do đó un+1 + √ dvn+1 = (u1 + √ dv1)2n = (a+ √ d)2n (2.2.9) Tương tự, trừ vế với vế của các phương trình trong hệ (2.2.8), ta cũng có un+1 − √ dvn+1 = (u1 − √ dv1)2n = (a− √ d)2n . (2.2.10) Từ (2.2.9) và (2.2.10) ta có    un+1 = 1 2 (a+ √ d)2n +(a− √ d)2n vn+1 = 1 2 √ d (a+ √ d)2n −(a− √ d)2n . (2.2.11) Do xn = un vn nên từ (2.2.11), ta có xn = √ d(a+ √ d)2n−1 +(a− √ d)2n−1 (a+ √ d)2n−1 −(a− √ d)2n−1 . Thử lại ( bằng quy nạp ) cho thấy kết quả nhận được thỏa mãn (2.2.6). 43
  • 46. Xét trường hợp d< 0. Đặt d = -q, q >0. Giả sử un, vn là một nghiệm của hệ phương trình sai phân    un+1 = u2 n −qv2 n, vn+1 = 2unvn, u1 = a, v1 = 1, (2.2.12) thì vn = un vn là nghiệm của phương trình (2.2.6) Thật vậy, ta chứng minh bằng quy nạp như sau x1 = u1 v1 = a 1 = a (đúng ) Giả sử xn = un vn là nghiệm của (2.2.6). Khi đó xn+1 = un+1 vn+1 = u2 n −qv2 n 2unvn = x2 n +d 2xn , cũng là nghiệm của (2.2.6). Để giải (2.2.6) ta giải hệ (2.2.12). Ta có    un+1 = u2 n −qv2 n i √ qvn+1 = 2i √ qunvn, u1 = a, v1 = 1, (2.2.13) Cộng vế với vế của các phương trình trong hệ (2.2.13) ta thu được un+1 +i √ qvn+1 = (un +i √ qvn)2 . Do đó un+1 +i √ qvn+1 = (u1 +i √ qv1)2n = (a+i √ q)2n . (2.2.14) Tương tự, trừ vế với vế của các phương trình trong hệ (2.2.13), ta cũng có un+1 −i √ qvn+1 = (u1 −i √ qv1)2n = (a−i √ q)2n . (2.2.15) Từ (2.2.14) và (2.2.15) ta có    un+1 = 1 2 (a+i √ q)2n +(a−i √ q)2n vn+1 = 1 2i √ q (a+i √ q)2n −(a−i √ q)2n . 44
  • 47. Do xn = un vn nên từ hệ nghiệm này, ta thu được xn = i √ q(a+i √ q)2n−1 +(a−i √ q)2n−1 (a+i √ q)2n−1 −(a−i √ q)2n−1 . Thử lại ( bằng quy nạp ) cho thấy kết quả nhận được thỏa mãn (2.2.6). Bài toán 2.2.5.[2] Tìm xn thỏa mãn điều kiện x1 = a,xn+1 = 2xn 1+dx2 n ,n ∈ N∗ (2.2.16) Giải. Trước hết ta xét trường hợp d = 0. Khi đó xn+1 = 2xn và xn = 2n−1a. Xét trường hợp d > 0. Giả sử un,vn là một nghiệm của hệ phương trình sai phân    un+1 = u2 n +dv2 n, vn+1 = 2unvn ,u1 = 1,v1 = a. (2.2.17) thì xn = un vn là nghiệm của phương trình (2.2.16) (chứng minh bằng phương pháp qui nạp). Như vậy để giải phương trình (2.2.16) ta cần giải hệ (2.2.17). Ta có    un+1 = u2 n +dv2 n, √ dvn+1 = 2 √ dunvn ,u1 = 1,v1 = a. (2.2.18) Cộng vế với vế của các phương trình trong hệ (2.2.18) ta thu được un+1 + √ dvn+1 = (un + √ dvn)2 . Do đó un+1 + √ dvn+1 = (u1 + √ dv1)2n = (1+a √ d)2n . (2.2.19) Tương tự, trừ vế với vế của các phương trình trong hệ (2.2.18), ta cũng có un+1 − √ dvn+1 = (u1 − √ dv1)2n = (1−a √ d)2n . (2.2.20) Từ (2.2.19) và (2.2.20) ta có    un = [(1+a √ d)2n−1 +(1−a √ d)2n−1 ], vn = √ d[(1+a √ d)2n−1 −(1−a √ d)2n−1 ]. (2.2.21) 45
  • 48. Do xn = un vn nên từ (2.2.21), ta thu được nghiệm của bài toán có dạng xn = [(1+a √ d)2n−1 +(1−a √ d)2n−1 ] √ d[(1+a √ d)2n−1 −(1−a √ d)2n−1 ] . Tương tự đối với trường hợp d < 0. Đặt d = −q, q > 0. Giả sử un,vn là một nghiệm của hệ phương trình sai phân    un+1 = u2 n −qv2 n, vn+1 = 2unvn ,u1 = 1,v1 = a. (2.2.22) thì xn = un vn là nghiệm của phương trình (2.2.16) (chứng minh bằng phương pháp qui nạp). Như vậy, để giải phương trình (2.2.16) ta cần giải hệ (2.2.22). Ta có    un+1 = u2 n −qv2 n, i √ qvn+1 = 2i √ qunvn ,u1 = 1,v1 = a. (2.2.23) Cộng vế với vế của các phương trình trong hệ (2.2.23) ta thu được un+1 +i √ qvn+1 = (un +i √ qvn)2 . Do đó un +i √ qvn = (u1 +i √ qv1)2n−1 = (1+ai √ q)2n−1 . (2.2.24) Tương tự, trừ vế với vế của các phương trình trong hệ (2.2.23), ta cũng có un −i √ qvn = (u1 −i √ qv1)2n−1 = (1−ai √ q)2n−1 . (2.2.25) Từ (2.2.24) và (2.2.25), ta có xn = [(1+ai √ q)2n−1 +(1−ai √ q)2n−1 ] i √ q[(1+ai √ q)2n−1 −(1−ai √ q)2n−1 ] . Thử lại (bằng qui nạp) cho thấy kết quả nhận được thỏa mãn (2.2.16). 46
  • 49. Sau đây ta xét một số bài toán cụ thể. Bài toán 2.2.6. Tìm các dãy xn, yn thỏa mãn hệ phương trình    xn+1 = 2xn +yn, x0 = 1 yn+1 = xn +2yn, y0 = 1 (2.2.26) Giải. Từ hệ phương trình (2.2.26) ta suy ra xn+2 = 4xn+1 −3xn, x0 = 1 và x1 = 3 Phương trình đặc trưng λ2 −4λ +3 = 0 có hai nghiệm thực phân biệt λ1 = 1, λ2 = 3. Do đó xn = A1n +B3n Mà x0 = 1, x1 = 3 nên ta có A = 0, B = 1 Suy ra    xn = 3n yn = 3n Bài toán 2.2.7. Tìm các dãy xn, yn thỏa mãn hệ phương trình    xn+1 = 5xn +yn, x0 = 1 yn+1 = −4xn +yn, y0 = 1 (2.2.27) Giải. Từ hệ phương trình (2.2.27) ta suy ra xn+2 = 6xn+1 −9xn, x0 = 1 và x1 = 6. Phương trình đặc trưng λ2 −6λ +9 = 0 có nghiệm kép λ = 3. Do đó xn = (A+Bn)3n. Mà x0 = 1, x1 = 6 nên ta có A = 1, B = 1. Suy ra    xn = 3n +n3n yn = −12.3n −12n.3n. Bài toán 2.2.8. Tìm các dãy xn, yn thỏa mãn hệ phương trình    xn+1 = xn −yn, x0 = 1 yn+1 = xn +yn, y0 = 1 (2.2.28) 47
  • 50. Giải. Từ hệ phương trình (2.2.28) ta suy ra xn+2 = 2xn+1 −2xn, x0 = 1 và x1 = 0. Phương trình đặc trưng λ2 −2λ +2 = 0 có hai nghiệm phức phân biệt λ1,2 = 1±i. Ta có λ1 = 1+i = √ 2(cos π 4 +isin π 4 ), λ2 = 1−i = √ 2(cos π 4 −isin π 4 ) Do đó xn = √ 2n(Acos nπ 4 +Bsin nπ 4 ). Mà x0 = 1, x1 = 0 nên ta có A = 1, B = −1. Suy ra    xn = √ 2n+1 cos(n+1) π 4 yn = √ 2n+1 sin(n+1) π 4 . Bài toán 2.2.9. Tìm xn, biết rằng x0 = 0,xn+1 = xn −2 xn +4 ,n ∈ N. (2.2.29) Giải. Xét hệ phương trình sai phân    yn+1 = yn +2zn,y0 = 0 zn+1 = yn +4zn,z0 = 1 Khi đó xn = yn zn là nghiệm của phương trình (2.2.29), suy ra yn+2 = 5yn+1 −6yn, y0 = 0,y1 = −2. Phương trình đặc trưng λ2 −5λ +6 = 0 có hai nghiệm thực phân biệt λ1 = 2,λ2 = 3. Do đó yn = A.2n + B.3n. Mà y0 = 0,y1 = −2 nên ta có A = 2,B = −2. Suy ra yn = 2.2n −2.3n;zn = 1 2 (yn −yn+1) = 2.3n −2n. Vậy xn = yn zn = 2.2n −2.3n −2n +2.3n . Bài toán 2.2.10. Tìm xn, biết rằng x0 = 1,xn+1 = xn −1 xn +3 ,n ∈ N. (2.2.30) Giải. Xét hệ phương trình sai phân 48
  • 51.    yn+1 = yn −zn,y0 = 1 zn+1 = yn +3zn,z0 = 1 Khi đó xn = yn zn là nghiệm của phương trình (2.2.30), suy ra yn+2 = 4yn+1 −4yn, y0 = 1,y1 = 0. Phương trình đặc trưng λ2 −4λ +4 = 0 có nghiệm kép λ = 2. Do đó yn = (A+Bn)2n. Mà y0 = 1,y1 = 0 nên ta có A = 1,B = −1. Suy ra yn = 2n −n.2n; zn = 2n +n.2n. Vậy xn = yn zn = 1−n 1+n . Bài toán 2.2.11. Tìm xn, biết rằng x0 = 0,xn+1 = xn −3 xn +1 ,n ∈ N∗ . (2.2.31) Giải. Xét hệ phương trình sai phân    yn+1 = yn −3zn,y0 = 0 zn+1 = yn +zn,z0 = 1 Khi đó xn = yn zn là nghiệm của phương trình (2.2.31), suy ra yn+2 = 2yn+1 −4yn,y0 = 0,y1 = −3. Phương trình đặc trưng λ2 −2λ +4 = 0 có hai nghiệm phức λ1,2 = 1±i √ 3. Ta có λ1 = 1+i √ 3 = 2(cos π 3 +isin π 3 ) λ2 = 1−i √ 3 = 2(cos π 3 −isin π 3 ) Do đó yn = 2n(Acosnπ 3 +Bsinnπ 3 ). Mà y0 = 1,y1 = −3 nên ta có A = 0,B = − √ 3. Suy ra yn = − √ 3.2n.sinnπ 3 ;zn = 1 3 (yn −yn+1) = 2ncosnπ 3 Vậy xn = yn zn = − √ 3tannπ 3 . 49
  • 52. 2.3. Giới hạn của một số dãy truy hồi dạng phân tuyến tính. Bài toán 2.3.1.[3] Xét dãy số {xn} được xác định bởi phương trình sau xn+1 = 1+ 1 1+xn ,x0 = 1 (2.3.32) Chứng minh rằng lim n→∞ xn = √ 2 Giải. Xét hàm số f(x) = 1+ 1 1+x . Ta có nhận xét rằng 1 < f(x) ≤ 2, f(x) là hàm dương, liên tục và nghịch biến trên [0;+∞). Từ hệ thức xn+1 = 1+ 1 1+xn = f(xn), ta suy ra 1 < xn+1 ≤ 2,∀n ∈ {0,1,2,...}. Ta cần xét hai trường hợp Trường hợp 1: x0 < x2 Suy ra f(x0) > f(x2) hay x1 > x3. Suy ra f(x1) < f(x3) hay x2 < x4. Suy ra f(x2) > f(x4) hay x3 > x5. ... Bằng qui nạp ta thu được x2n < x2n+2. Thật vậy, giả sử x2k < x2k+2 thì f(x2k) > f(x2k+2) hay x2k+1 > x2k+3. Suy ra f(x2k+1) < f(x2k+3) hay x2k+2 < x2k+4. Vậy {x2n} là dãy tăng. Chứng minh tương tự ta được {x2n+1} là dãy giảm. Từ đó, ta có - Dãy {x2n} tăng và bị chặn trên nên {x2n} hội tụ. Giả sử x2n → α khi n → ∞. - Dãy {x2n+1} giảm và bị chặn dưới nên {x2n+1} hội tụ. Giả sử x2n+1 → β khi n → ∞. Do hàm f liên tục nên 50
  • 53. x2n+1 n→∞ −−−−→ β || f(x2n) n→∞ −−−−→ f(α) Suy ra f(α) = β. Tương tự, ta cũng có f(β) = α và ta thu được hệ phương trình    β = f(α) α = f(β) tương đương với    β = 1+ 1 1+α α = 1+ 1 1+β từ đó suy ra    αβ +β = 2+α αβ +β = 2+β và do đó ta nhận được α = β = √ 2. Trường hợp 2: x0 ≥ x2. Ta thu được - Dãy {x2n} giảm và bị chặn dưới nên {x2n} hội tụ. Giả sử x2n → α khi n → ∞. - Dãy {x2n+1} tăng và bị chặn trên nên {x2n+1} hội tụ. Giả sử x2n+1 → β khi n → ∞. Tương tự như trường hợp 1, ta cũng thu được α = β = √ 2. Vậy lim n→∞ xn = √ 2 Bài toán 2.3.2.[3] Cho f là hàm dương, liên tục và nghịch biến trên [0;+∞). Giả sử rằng hệ phương trình    β = f(α) α = f(β) có nghiệm duy nhất α = β = l. Chứng minh rằng dãy số dương xn+1 = f(xn) với x0 > 0 cho trước hội tụ tới l. 51
  • 54. Giải. Ta xét 2 trường hợp Trường hợp 1:x0 < x2 Suy ra f(x0) > f(x2) hay x1 > x3. Suy ra f(x1) < f(x3) hay x2 < x4. Suy ra f(x2) > f(x4) hay x3 > x5. ... Bằng quy nạp có thể chứng minh được x2n < x2n+2. Thật vậy, giả sử x2k < x2k+2 Khi đó thì f(x2k) > f(x2k+2) hay x2k+1 > x2k+3, Do đó f(x2k+1) < f(x2k+3) hay x2k+2 < x2k+4. Vậy {x2n} là dãy tăng. Chứng minh tương tự ta được {x2n+1} là dãy giảm. Từ đó, ta có - Dãy {x2n} tăng và bị chặn trên nên {x2n} hội tụ. Giả sử x2n → α khi n → ∞. - Dãy {x2n+1} giảm và bị chặn dưới nên {x2n+1} hội tụ. Giả sử x2n+1 → β khi n → ∞. Do hàm f liên tục nên x2n+1 n→∞ −−−−→ β || f(x2n) n→∞ −−−−→ f(α) Suy ra f(α) = β. Tương tự, ta cũng thu được f(β) = α Vậy ta có hệ phương trình    β = f(α) α = f(β) Theo giả thiết hệ    β = f(α) α = f(β) có nghiệm duy nhất α = β = l. Trường hợp 2: x0 ≥ x2. 52
  • 55. Ta dễ dàng kiểm tra - Dãy {x2n} giảm và bị chặn dưới nên {x2n} hội tụ. Giả sử x2n → α khi n → ∞. - Dãy {x2n+1} tăng và bị chặn trên nên {x2n+1} hội tụ. Giả sử x2n+1 → β khi n → ∞. Tương tự như trường hợp 1 ta cũng thu được α = β = l. Vậy lim n→∞ xn = l. Bài toán 2.3.3. Xét dãy số cho bởi phương trình sai phân sau xn+1 = 1+ 2 1+xn ,x0 > 0. Khảo sát sự hội tụ của dãy {xn}. Giải. Xét hàm số f(x) = 1+ 2 1+x . Dễ thấy hàm số f(x) dương, liên tục và nghịch biến trên [0;+∞) và thỏa mãn hệ phương trình    β = f(α) α = f(β). Tương đương với hệ phương trình    αβ = α −β +3 αβ = β −α +3 và giải hệ phương trình trên ta nhận được α = β = √ 3. Do đó theo kết quả của bài toán 2.3.2, ta có dãy {xn} có giới hạn là √ 3 khi n → ∞ Vậy lim n→∞ xn = √ 3. Bài toán 2.3.4.[3] Xét dãy số {xn} xác định theo công thức    x0 = 1 xn+1 = 2 xn + √ 3 x2 n ∀n ∈ N. Chứng minh rằng dãy số {xn} không có giới hạn hữu hạn. Giải. Nhận xét rằng nếu dãy số {xn} có giới hạn a thì a là nghiệm của phương trình x = 2 x + √ 3 x2 . 53
  • 56. Do đó a = √ 3. Ta có x0 = 1 > x2. Ta chứng minh qui nạp rằng x2n > x2n+2. Thật vậy x2k+1 = 2 x2k + √ 3 x2 2k < 2 x2k+2 + √ 3 x2 2k+2 = x2k+3. Suy ra x2k+2 = 2 x2k+1 + √ 3 x2 2k+1 > 2 x2k+3 + √ 3 x2 2k+3 = x2k+4. Do vậy 1 = x0 > x2 > ... > x2n > ... > 0. suy ra dãy {x2n} là dãy đơn điệu giảm và bị chặn dưới bởi 0, nên tồn tại giới hạn hữu hạn b = lim n→∞ x2n Mặt khác do x0 = 1 nên b ≤ 1. Vì tồn tại giới hạn hữu hạn a = lim n→∞ xn = √ 3, nên nói riêng lim n→∞ x2n = lim n→∞ xn = √ 3 Tức là b = √ 3 . Đây là điều vô lý. Vậy dãy số {xn} không có giới hạn hữu hạn. Bài toán 2.3.5.[6] Xét dãy số {xn} xác định như sau    u0 = 1 un = −1 3+un−1 với n = 1,2,... Chứng minh rằng dãy số {un} có giới hạn và hãy tìm lim n→∞ un . Giải. Ta có un −un+1 = un + 1 3+un = u2 n +3un +1 3+un . Bây giờ ta chứng minh rằng un > −3+ √ 5 2 (*) với mọi n = 1,2,... bằng qui nạp như sau - Với n = 0, thì u0 = 1 > −3+ √ 5 2 ( đúng ). - Giả sử (*) đúng với n = k (k = 1,2,...), nghĩa là ta có ta có uk > −3+ √ 5 2 . 54
  • 57. Khi đó: 3+uk > −3+ √ 5 2 = 3+ √ 5 2 , suy ra 1 3+uk < 2 3+ √ 5 = 3− √ 5 2 . Do đó uk+1 = − 1 3+uk > −3+ √ 5 2 . Vậy cũng đúng với n=k+1. Theo nguyên lí quy nạp suy ra đúng với mọi n. Vì un > −3+ √ 5 2 với mọi n, nên 3 + un > 3+ √ 5 2 , tức là 3 + un > 0 với mọi n = 0,1,2,... Do un > −3+ √ 5 2 , nên theo định lí thuận về dấu tam thức bậc hai thì u2 n +3un +1 > 0 với mọi n = 0,1,2,... Mà un −un+1 = u2 n +3un +1 3+un , suy ra un > un+1 với mọi n, nghĩa là dãy {un} là đơn điệu giảm và bị chặn dưới bởi −3+ √ 5 2 . Suy ra tồn tại giới hạn của dãy {un} khi n → ∞, và đặt lim n→∞ un = a. Từ un = −1 3+un−1 và lấy giới hạn 2 vế khi n → ∞, ta có: lim n→∞ un = −1 3+ lim n→∞ un−1 hay a = −1 3+a ⇔ a2 +3a+1 = 0 ⇔ a = −3± √ 5 2 . Vì un > −3+ √ 5 2 với mọi n, nên a = lim n→∞ un ≥ −3+ √ 5 2 . Suy ra a = −3+ √ 5 2 . Vậy lim n→∞ un = −3+ √ 5 2 . 55
  • 58. 2.4. Bài tập tham khảo. Bài tập 2.4.1. Tìm xn, biết rằng x0 = 1,xn+1 = 1−4xn 1−6xn ,n ∈ N. Bài tập 2.4.2. Tìm xn, biết rằng x0 = −1,xn+1 = 2xn −3 3xn −4 ,n ∈ N. Bài tập 2.4.3. Tìm xn, biết rằng x0 = 0,xn+1 = xn +1 4−xn ,n ∈ N. Bài tập 2.4.4. Cho a ∈ R{1}. Xét dãy số {xn} được xác định theo công thức    x1 = a xn+1 = xn(x2 n +3) 3x2 n +1 ∀n ∈ N∗. Chứng minh rằng dãy số {yn} = {(a−1)xn} có giới hạn và xác định giới hạn đó. Bài tập 2.4.5. Tính giới hạn của dãy {xn} với xn+1 = 1 1+xn ,x0 = 1. Bài tập 2.4.6. Tính giới hạn của dãy {xn} với xn+1 = xn 1+x2 n ,x0 > 0. Bài tập 2.4.7. Tính giới hạn của dãy {xn} với xn+1 = x2 n +3 2(xn +1) ,x0 > 0. Bài tập 2.4.8. Tính giới hạn của dãy {zn} với zn = yn+1 −yn,yn = 1 x2 n ,x0 > 0. 56
  • 59. Bài tập 2.4.9. Dãy số {un} xác định như sau    u1 = √ 3 3 un+1 = un +2− √ 3 1+( √ 3−2)un ; n = 2,3,... Tìm công thức cho un, sau đó tính u2010. Bài tập 2.4.10. Cho p,q là hai số nguyên cho trước. Dãy {un} được xác định như sau    u1 = p+q−1 un+1 = p+q− pq un ; n = 1,2,... Đặt Pn = u1u2 ...un. Chứng minh rằng vối mọi n = 1,2,··· thì Pn là số nguyên. Bài tập 2.4.11. Dãy số {un},n = 1,2,... được xác định như sau:    u1 = 2 3 un+1 = un 2(2n+1)un +1 ; n = 1,2,... Tính tổng S = ∑2010 i=1 ui. Bài tập 2.4.12. Dãy số {un},n = 1,2,... được xác định như sau:    u1 = 2 un+1 = 1+ 1 un ; n = 1,2,... Gọi p là số lẻ và q là số chẵn bất kỳ. Chứng minh Up > Uq. 57
  • 60. KẾT LUẬN Sau thời gian học tập hai năm tại khoa Toán-Cơ-Tin học, Trường Đại học Khoa học Tự nhiên Hà Nội, được sự giúp đỡ chỉ bảo của các thầy cô trong khoa, đặc biệt là PGS. TS. Nguyễn Minh Tuấn, tác giả đã hoàn thành luận văn với đề tài: " Phép biến đổi phân tuyến tính và áp dụng giải một số bài toán phổ thông ". Luận văn đã đạt được một số kết quả sau: 1. Luận văn đã nêu được khá đầy đủ các bài toán tổng quát về phương trình hàm sinh bởi hàm phân tuyến tính w(x) = ax+b cx+d , c = 0 ,ad −bc = 0 , đồng thời cũng tập hợp được các bài toán khác về hàm phân tuyến tính trong các kỳ thi Olympic. 2. Luận văn đã trình bày được các bài toán điển hình về phương trình sai phân dạng phân tuyến tính với hệ số hằng và giới hạn của một số dãy sai phân dạng phân tuyến tính. 3. Luận văn có thể làm tài liệu tham khảo bổ ích cho quá trình nghiên cứu, giảng dạy và học tập về phép biến đổi phân tuyến tính ở bậc phổ thông. Trong mỗi phần của luận văn tác giả đều đưa ra rất nhiều ví dụ minh họa cho mỗi phần. Mặc dù đã cố gắng hết sức và nghiêm túc trong quá trình nghiên cứu, nhưng do thời gian và khả năng còn hạn chế nên chắc chắn luận văn không tránh khỏi những thiếu sót. Một lần nữa, tác giả rất mong nhận được những ý kiến đóng góp của các thầy cô giáo và bạn bè đồng nghiệp. Trong thời gian tới, tác giả sẽ cố gắng tiếp tục nghiên cứu phép biến đổi này trong quá trình giảng dạy và học tập của mình. 58
  • 61. Tài liệu tham khảo [1] Nguyễn Văn Mậu, 1997, Phương trình hàm, Nhà xuất bản Giáo dục. [2] Nguyễn Văn Mậu, 2003, Mốt số bài toán chọn lọc về dãy số, Nhà xuất bản Giáo dục. [3] Nguyễn Văn Mậu, Nguyễn Thủy Thanh, 2008, Giới hạn dãy số và hàm số, Nhà xuất bản Giáo dục. [4] Nguyễn Trọng Tuấn, 2005, Bài toán hàm số qua các kỳ thi Olympic, Nhà xuất bản Giáo dục. [5] Lê Đình Thịnh và Lê Đình Định, 2004, Phương trình sai phân, Nhà xuất bản Đại học Quốc Gia Hà Nội. [6] Phan Huy Khải, 2007, Các bài toán về dãy số, Nhà xuất bản Giáo dục . 59